Вы находитесь на странице: 1из 38

Envi Reviewer (MIDTERMS) Page |1

CRUZ V. SECRETARY OF ENVIRONMENT AND NATURAL RESOURCES came into the possession of Simeon T. Dagdag. On the other hand, the lease to De Vera signed in June
1916 covered adjoining land of a bigger area. It was transferred by him to Regino Nepomuceno. After
Facts: Petitioners Isagani Cruz and Cesar Europa brought suit for prohibition and mandamus assailing the purchasing the land, Simeon T. Dagdag had it relocated and the portion in question turned out to be in
constitutionality of certain provisions of RA 8371 otherwise known as the Indigenous Peoples Rights Act of possession of the heirs of Regino Nepomuceno, allegedly by virtue of the lease. The latter refused to
1997 (IPRA) and its Implementing Rules and Regulations. They also assailed Section 3(a) and (b), 5, 6, 7, surrender it, even in the face of Dagdag's patent and title, and despite the Director of Lands' administrative
8, 57, and 58 of the IPRA and its IRR on the ground that they amounted to an unlawful deprivation of the determination holding that their contract of lease did not, could not and should not extend to the area
State’s ownership over lands of the public domain as well as minerals and other natural resources in granted to Dagdag's predecessors.
violation of the regalian doctrine embodied in Section 2, Article XII of the Constitution. In addition, they
questioned Sections 51 to 53 and 59, 52(i), 63, 65, and 66 of the said RA defining the powers and Issue: WON a registered contract of lease can be considered a "title".
jurisdiction of the NCIP and making customary law applicable to the settlement of disputes involving
Held: No. The sales patent issued in the name of Margarita Juanson having been registered with the office
ancestral domains and ancestral lands on the ground that these provisions violate the due process clause
of the Register of Deeds, and title having been issued by the Register of Deeds in the name of Margarita
of the Constitution. Finally, petitioners assailed the validity of Rule VII, Part II, Section 1 of the NCIP
Juanson, Lot 3786 was thereafter brought under the operation of the Land Registration Act. The plaintiffs
Administrative Order No. 1, series of 1998 providing the administrative relationship of the NCIP to the
herein cannot, therefore, be bound by the fact that Lot 3786 is within the lease of Andres de Vera which
Office of the President is characterized as a lateral but autonomous relationship for purposes of policy and
had been transferred to Regino Nepomuceno, the father and predecessor of the defendants. The fact that
program coordination, contending that the said Rule infringes upon the President’s power of control over
the lease in favor of Andres de Vera had been registered, cannot bind and prejudice the plaintiff for Lot
executive departments under Section 17, Article VII of the Constitution.
3786 being a registered land, he need not go farther than the title." Public land patents when registered
in the corresponding Register of Deeds Office are regarded as veritable Torrens Titles subject to no
Issue: Whether or not the IPRA and its IRR are valid.
encumbrances except those stated therein, plus those specified by the statute (lease is not one of them).
In addition to that, it should be remembered that when the lease was renewed in 1949, the portion in
Held: The petition is dismissed pursuant to Rule 56, Section 7 of the Rules of Civil Procedure that as the
question was no longer public land subject to the disposition of the Director of Lands because it had already
votes were equally divided (7 to 7) and the necessary majority was not obtained, the case was
been granted to Margarita Juanson and had become private property; therefore, it could not have been
redeliberated upon. However, after the redeliberation, the voting remained the same.
included in the renewal of such lease of public land. The defendants also averred that, since the titles of
the parties have come under the operation of the Land Registration Act, in case of overlapping titles, the
Ancestral Domains are all areas belonging to ICCs/IPs held under a claim of ownership, occupied or
older title should prevail. The title of the defendants was issued and registered on June 14, 1916. The title
possessed by ICCs/IPs by themselves or through their ancestors, communally or individually since time
of the plaintiff was registered on August 5, 1927. According to them, the title of the defendants should,
immemorial, continuously until the present, except when interrupted by war, force majeure or displacement
therefore prevail, and they should have been declared the owners of the land in question. The flaw in their
by force, deceit, stealth or as a consequence of government projects or any other voluntary dealings with
argument lies in the assumption that their lease contract constituted a "title", or deed or conveyance within
government and/or private individuals or corporations. It comprises lands, inland waters, coastal areas,
the meaning of section 122. The documents mentioned, wherein lands are "alienated, granted, or
and natural resources therein and includes ancestral lands, forests, pasture, residential, agricultural, and
conveyed", are documents transferring ownership—not documents of lease, transferring mere possession.
other lands individually owned whether alienable or not, hunting grounds, burial grounds, worship areas,
bodies of water, mineral and other natural resources. They also include lands which may no longer be
exclusively occupied by ICCs/IPs but from which they traditionally had access to for their subsistence and
traditional activities, particularly the home ranges of ICCs/IPs who are still nomadic and/or shifting REPUBLIC VS. NAGUIAT
cultivators. Facts: Celestina Naguiat filed for registration of title to four parcels of land
Ancestral lands are lands held by the ICC/IPs under the same conditions as ancestral domains except Celestina Naguiat filed an application for registration of title to four parcels of land located in Panan,
that these are limited to lands and that these lands are not merely occupied and possessed but are also Botolan, Zambales. The applicant alleges that she is the owner of the said parcels of land having acquired
utilized by the ICCs/IPs under claims of individual or traditional group ownership. These lands include but them by purchase from its previous owners and their predecessors-in-interest who have been in possession
are not limited to residential lots, rice terraces or paddies, private forests, swidden farms and tree lots. thereof for more than thirty (30) years; and that to the best of her knowledge, said lots suffer no mortgage
or encumbrance of whatever kind nor is there any person having any interest, legal or equitable, or in
possession thereof.
SIMEON T. DAGDAG VS. VICENTE NEPOMUCENO, ET AL.,
Petitioner Republic opposed on the ground that neither the applicant nor her predecessors-in interest have
Facts: A portion of Lot No. 3786, is covered by Sales Patent No. 251 issued to Margarita Juanson, and also been in open, continuous, exclusive and notorious possession and occupation of the lands in
by lease No. 49 executed by the Bureau of Lands in favor of Andres de Vera. The overlapping was recently question since 12 June 1945 or prior thereto, considering the fact that she has not established that the
discovered, and their successors in interest now litigate for possession and/or ownership. The Sales Patent lands in question have been declassified from forest or timber zone to alienable and disposable property.
was inscribed in the office of the Register of Deeds on July 11, 1927, and Original Certificate of Title No.
ISSUE:
68 was accordingly issued in the name of Margarita Juanson. Juanson sold it to several persons until it
Envi Reviewer (MIDTERMS) Page |2

Whether or not the said parcels of land are alienable. The Court ruled in the negative.

HELD The first sentence of Section 2 embodies the Regalian Doctrine which in its simplest meaning, the capacity
of the State to own or acquire property. It was introduced by Spain and was carried over until the present
No, the said areas are still classified as forest land. The issue of whether or not respondent and her Constitution.
predecessors-in-interest have been in open, exclusive and continuous possession of the parcels of land in
question is of little moment. For, unclassified land cannot be acquired by adverse occupation or possession; The Regalian doctrine extends not only to land but also to allnatural wealth that may be found in the
occupation thereof in the concept of owner, however long, cannot ripen into private ownership and be bowels of the earth. Conspicuously absent in the 1987 Constitution is the provision in the 1935 and 1973
registered as title. Constitutions authorizing the State to grant licenses, concessions, or leases for the exploration exploitation,
development or utilization of natural resources.
A forested area classified as forest land of the public domain does not lose such classification simply
because loggers or settlers have stripped it of its forest cover. Parcels of land classified as forest land may However, Section 2 offers the State an option among others, for the President to enter into an agreement
actually be covered with grass or planted to crops by kaingin cultivators or other farmers. The classification with foreign- owned corporations involving either technical or financial assistance for large scale
is merely descriptive of its legal nature or status and does not have to be descriptive of what the land exploration, development and utilization of minerals, petroleum, and other mineral oils.
actually looks like. Moreover, the duty of classifying lands rests in the government and unless a positive
act from the government was done through the DENR to reclassify the land into inalienable, a land will still It should be noted though that the FTAA’s based on Section 2 SHOULD BE LIMITED to technical and
be part of the public domain. financial assistance only. However, contrary to the language of the constitution, the WMCP FTAA allows
WMCP to extend more than mere financial or technical assistance to the State, for it permits WMCP to
manage and operate every aspect of the mining activity.
LA BUGAL- B’LAAN TRIBAL ASSOC. ET. AL. V. SEC. VICTOR RAMOS (DENR) ET. AL. It should be noted that the phrase “management or other form of assistance” which appears in the 1973
constitution was deleted in the 1987 constitution. The management or operation of mining activities by
FACTS:
foreign contractors, which is the primary feature of service contracts, was precisely the evil that the drafters
This case in relation to the Public Land Act discusses the Regalian Doctrine that was embodied in our of the 1987 constitution sought to eradicate. It was a subterfuge to get around the nationality requirement
present Constitution specifically under Section 2, Article XII. of the constitution. The contractual stipulations grant WMCP beneficial ownership over natural resources
that properly belong to the State and are intended for the benefit of its citizens.
The petition herein assails the constitutionality of RA 7942 or the Philippine Mining Act of 1995, along with
its IRR and the FTAA or the Financial and Technical Assistance Agreement entered into by the Republic of
the Philippines and WMCP or the Western Mining Corporation Philippines. RA 7942 governs the exploration, Republic of the Philippines v. Court of Appeals and Jose Y. De La Rosa
development, utilization and processing of all mineral resources. While FTAA is defined as a contract
involving technical or financial assistance for large-scale exploration, development, and utilization of natural Facts:
resource
The case arose when Private Respondent Jose Y. De La Rosa on his own behalf and in behalf of his children
Shortly before the effectivity of RA 7942, the President entered into an FTAA with WMCP covering 99, 387 (“De La Rosa” for brevity) applied for the registration of a parcel of land on February 11, 1965. The land
hectares of land in South Cotabato, Sultan Kudarat, Davao Del Sur, and North Cotabato. WMCP is a was situated in Tuding, Itogon, Benguet Province, and was divided into 9 lots. The Lots 1-5 were bought
subsidiary of a major Australian mining and exploration company. Thereafter, DENR Secretary Ramos from Mamaya Balbalio, and Lots 6-9 were purchased from Jaime Alberto. Said lots were acquired by the
issued the IRR for RA 7942. Herein petitioners filed a petition for prohibition and mandamus to stop the sellers by virtue of prescription. However, this application was opposed by Benguet Consolidated, Inc., as
implementation of RA 7942 and its IRR. to Lots 1-5, Atok Big Wedge Corporation, as to portions of Lots 1-5 and all of Lots 6-9, and the Republic,
through the Bureau of Forestry Development as to Lots 1-9. Benguet Consolidated and Atok Big Wedge
Petitioners contend that the DENR Secretary acted without or in excess of his jurisdiction in signing and claim that they have acquired said lots from their respective locator who registered the same as mineral
promulgating the IRR, the latter being unconstitutional in that it allows fully foreign owned corporations to lands. For the Republic, it contended that the land sought to be registered was covered by the Central
explore, develop, utilize, and exploit mineral resources in a manner contrary toArt. XII of the Constitution. Cordillera Forest Reserve under Proclamation No. 217, hence, not subject to alienation.
They also prayed to declare the FTAA with WMCP as unconstitutional, null and void.
The trial court denied the application. On appeal by the applicants, the Court of Appeals reversed the
ISSUE: decision of the trial court and recognized the claims of the applicant, but subject to the rights of Benguet
and Atok respecting their mining claims. In other words, CA affirmed the surface rights of the De La Rosas
The issue is WON RA 7942 and its IRR together with the FTAA entered into by the President is constitutional
over the land while at the same time reserving the sub-surface rights of Benguet and Atok by virtue of
and valid.
their mining claim.
RULING:
Issue:
Envi Reviewer (MIDTERMS) Page |3

Whether or not the CA is correct when it ruled that the applicant, and Benguet and Atok have their rights registration proceedings over the said parcels of land. Unfortunately, these documents were burned during
on the same land, the former is the owner of the surface rights, while the latter are the owners of the sub- the last World War and no official copy could be obtained from the Register of Deeds of Mabitac, Laguna.
surface rights.
ISSUE:
Held:
WETHER OR NOT THE APPLICANT HAS RIGHT OVER THE SUBJECT PARCEL OF LAND.
The Supreme Court held that the CA erred in justifying the respective rights of the parties. The CA was
wrong when it said that the rights over the land could be used for both mining and non-mining purposes RULING:
simultaneously. The correct interpretation is that once minerals are discovered in the land whatever the
THE APPLICANT DO NOT HAVE RIGHT OVER THE SUBJECT PARCEL OF LAND
use to which it is being devoted at the time, such use may be discontinued by the State to enable it to
extract the minerals therein in the exercise of its prerogative. Further, the SC ruled that Benguet and Atok From the enactment of Act No. 190 to the present Rules of the Court, the rule governing the sale of real
have exclusive rights to the property in question by virtue of their respective mining claims which they property has remained unchanged: It can be proved only by the very instrument reciting the transaction,
validly acquired before the Constitution of 1935 prohibited the alienation of all lands of the public domain duly subscribed by the proper party or his authorized agent, or else by secondary evidence of the contents
except agricultural land, subject to vested rights existing at the time of its adoption. of such document. However, before the terms of a transaction in reality may be established by secondary
evidence, it is necessary that the due execution and subsequent loss of the original instrument evidencing
the transaction be proved. For it is due execution, and loss thereafter, of the document that would warrant
THE DIRECTOR OF LANDS, and ADRIANO CARPIO, MARTIN AGUILAR and PEDRO AGUILAR, v. or constitute basis for the introduction of the secondary evidence to prove the contents of such document.
C.A and MARIANO RAYMUNDO. And the due execution of the document should be proved through the testimony of (1) the person or
persons who executed it; (2) the person before whom its execution was acknowledge; or (3) any person
FACTS: who was present and saw it executed and delivered, or who, after its execution and delivery, saw it and
recognized the signatures, or by a person to whom the parties to the instruments had previously confessed
The applicant, having failed to establish his right or title over the northern portion of Lot No. 463 involved
the execution thereof. Thus, in one case, the admission of the certified copy of the record of a deed in a
in the present controversy, and there being no showing that the same has been acquired by any private
public registry as secondary evidence of the terms of the deed of sale, was declared improper and invalid,
person from the Government, either by purchase or by grant, the property is and remains part of the public
the Court pointing to the party’s failure to present the notary and those persons who must have seen the
domain.
signing of the document as witnesses to testify on its due execution.
Mariano B. Raymundo filed in the Court of First Instance of Laguna an application for registration of his
imperfect or incomplete title over five parcels of land (Lots Nos. 461, 462, 463, 480, and 480 and 483,
Mabitac Cadastre) situated in Mabitac, Laguna, allegedly acquired by actual, open, adverse and continuos CARIÑO VS INSULAR GOVERNMENT
occupation of the properties, by himself and by his predecessors-n-interest since time immemorial.
FACTS:
The application for registration was opposed by several parties, specifically, (a) by the Director of Lands,
on the ground of applicant’s lack of registrable title; and (b) by Adriano Carpio, Martin Aguilar and Pedro On, June 22, 1903, Mateo Cariño by his attorney, Metcalf A. Clarke, filed a petition in Court of Land
Aguilar, as regards the northern portion of Lot No. 463, for the reason that they were the actual possessors Registration, asking that he be inscribed as the owner of a tract of land in the municipality of Baguio, in
thereof and had filed homestead applications thereof since 1935. the Province of Benguet, containing 146 hectares.

After hearing, the registration court rendered judgement declaring applicant Raymundo to have established The Governmet of the Philippine Islands opposed the petition. The Government of the United States also
proprietary rights over Lots No. 461, 462, 480, 483 and the southern portion of Lot No. 463; and ruling appeared and opposed the petition on the ground that the land was part of the military reservation of
oppositors Carpio and Aguilar brothers to have likewise proved their title as regards the northern portion Baguio. Court of Land Registration rendered a judgment in favor of the petitioner. Respondents appealed
of Lot No. 463. to the Court of First Instance of the Province of Benguet. Judgment was entered dismissing the petition.

Both Raymundo and the Director of lands appealed to the Court of Appeals. In its decision of 11 July 1968, The petitioner presented no documentary evidence of title, except a possessory information obtained in
the appellate court modified the judgement of the trial court, by recognizing Raymundo’s registrable title, 1901 by the provisions of the Mortgage Law. The petitioner, however insists that although the statute of
not only over Lots. 461, 162, 480, 483 and the southern portion of Lot No. 463 but even over the northern limitations did not run against the Government of Spain in the Philippine Islands, yet a grant is to be
part of Lot No. 463 adjudicated to oppositors Carpio and Aguilar brothers. Raymundo’s claim over the conclusively presumed form immemorial use and occupation.
whole Lot No. 463 was declared proved by a Deed of Absolute Sale.
ISSUES:
The Court of Appeals said that applicant Raymundo satisfactorily explained the presentation of said copy
in lieu of the original by proving that the original document together with other pertinent paper were Whether or not Mateo Cariño is the rightful owner of the tract of land.
entrusted by him to his lawyer, Judge Mariano C. Melendres, sometime before the war in connection with
HELD:
Envi Reviewer (MIDTERMS) Page |4

The possession of the land has not been of such character as to require the presumption of a grant. No However, the respondent judge on October 12, 1987, issued an order authorizing Mr. Palomares to sell,
one has lived upon it for many years. It was never used for anything but pasturage of animals, except lease, dispose any area or portion of the property subject of OCT no. 291. Eleven days after or on October
insignificant portions thereof. And since the insurrection against Spain, it has apparently not been used by 23, 1987, he issued another order affirming the power of the plaintiff Domingo C. Palomares to execute
the petitioner for any purpose. The plaintiff is not entitled to the benefits of paragraph 6 of section 54 of and perform the acts authorized in the said first order without the need of Writ of Execution.
Act No. 926, the Public Land Act, because the act is not applicable to the Province of Benguet.The judgment
of the courts below is AFFIRMED. The petitioners filed a motion for reconsideration however the same was denied by the respondent judge
…………………………………………………………………………………………………. as the same was against a interlocutory order. Hence, the petitioner filed the present petition for certiorari
against Judge Velez for issuing the orders dated October 12 and 23, 1987.
In the opinion of Justice Kapunan, the Regalian Theory does not negate title to lands held in private
ownership since time immemorial, adverting to the landmark case of Cariño v. Insular Government where ISSUE
the United States Supreme Court, reversing the decision of the pre-war Philippine Supreme Court, made
1. Whether or not OCT No 291 is still valid and subsisting
the following pronouncement:
2. Whether or not respondent Judge Velez commit a grave abuse of discretion amounting
to lack or excess of jurisdiction in issuing the Orders dated October 12 and 23, 1987.
“ Every presumption is and ought to be taken against the Government in a case like the present. It might,
perhaps, be proper and sufficient to say that when, as far back as testimony or memory goes, the land RULING
has been held by individuals under a claim of private ownership, it will be presumed to have been held in
the same way from before the Spanish conquest, and never to have been public land. “ 1. The court take judicial notice that the hectarage embraced in OCT No. 291/TCT No. 192 consist of
government property. As mentioned by the court, Proclamation No. 192 and Proc. No. 423 have the
The above ruling institutionalized the recognition of the existence of native title to land, or ownership of character of official assertions of ownership and a valid exercise of the State’s dominical authority and
land by Filipinos by virtue of possession under a claim of ownership since time immemorial and independent right of sovereignty. The same is a valid declaration of regalian right of the government over the
of any grant from the Spanish Crown, as an exception to the theory of jura regalia. property. The court find hard evidence that indeed OCT No. 291 had been conveyed to the USA and
later surrendered to the Republic of the Philippines and as a consequence, OCT No. 291 was cancelled
and the land covered therein has been conveyed to the national government. In absence of any
ACTING REGISTRARS OF LAND TITLES AND DEEDS OF PASAY, PASIG AND MAKATI VS JUDGE concrete and convincing evidence on the part of the respondent to prove that indeed the landholding
FRANCISCO X. VELEZ AND THE INTESTATE ESTATE OF THE LATE DELFIN CASAL is still under their ownership, the presumption is that the said property belongs to the state.
FACTS:

Private respondent Domingo Palorames filed a case before the Regional Trial Court Br. 132 of Makati for 2. The court also found Judge Velez guilty of grave abuse of discretion amounting to lack or excess of
declaratory relief, quieting of title, cancellation of TCT No. 192 and cancellation of entries in OCT No. 291. jurisdiction because what he gave away, by virtue of the Order he issued, was the property that
Prior to the filing of the said case, Mr. Palomares had earlier filed a petition for issuance of owner’s duplicate inalienably belongs to the government. Worse, he gave it away without notice to its present owner.
copy of OCT No. 291 embracing Hacienda de Maricaban, however the said petition was denied for lack of Aside from that, he denied the SolGen’s notice to appeal. The order he issued was in the nature of a
merit. final judgement thus the proper remedy is to appeal the same. The court also note the improper action
of the respondent Judge when he filed his answer on the petition because according to Rule 65 of the
On June 2, 1986, private respondent Palomares filed a motion to admit amended complaint impleading the Rules, such as the present cases, the judge is included only as a nominal party and should have
Republic of the Philippines and the Registry of Deeds of Pasay, Pasig and Marikina, alleging that the Court remained a neutral magistrate. Thus, he was directed by the court to show cause why he should not
of Land Registration acknowledged or confirmed the title of Dolores Casal which covers the 2,574 hectare- be administratively dealt with for giving away the property that inalienably belongs to the Government,
parcel of land and that the Registry of Deeds of Rizal issued OCT No. 291 to Dolores Casal, and that after without notice to the registered owner, and for blocking Government efforts to defend what rightfully
her death, the said property was managed by her grandchildren, that there were no record of conveyances belongs to it. The petition in G.R. No. 81564 is GRANTED.
or dispositions of any kind have been allegedly made upon the parcel, he also alleged that the State itself,
by placing 27,213,255 square meters thereof under a military reservation (now Fort Bonifacio), by
Proclamation No. 423, and fifty hectares thereof pursuant to Proclamation No. 192, had been guilty of FAUSTINA JAMISOLA VDA. DE CALIBO, and HIPOLITO, ALEJANDRO, AURORA, VlCENTE,
landgrabbing and that TCT No. 192 which covers the same landholding is inexistent, and that any titles PACIFICO and PAZ, surnamed LIBUTAN, HEIRS OF OLIVA JAMISOLA, petitionersappellants,
coming from TCT No. 192 is null and void. vs. TIBURCIO BALLESTEROS, THE DIRECTOR OF LANDS and THE SECRETARY OF
AGRICULTURE & NATURAL RESOURCES, respondents-appellees.
The petitioners in this case, filed their answer, refuting the claims of private respondents and alleging that
OCT No. 291 which is according to Palomares is the one valid and should be upheld by the court has long The sales application of Tiburcio Ballesteros is in conflict with the sales application No. 10969 of one
been cancelled and the property covered by the said title had been delivered and conveyed to the Barbara Andoy on which the Director of Lands rendered a decision on April 10, 1930, the dispositive part
government. They also filed a memorandum alleging that Dolores Casay had conveyed the property to the of which is as follows:
government of US and Judge Ostrand even placed or stamped his imprimatur.
Envi Reviewer (MIDTERMS) Page |5

'Upon investigation it was found that Barbara Andoy entered a portion of the land in dispute with the the Secretary of Agriculture and Commerce is made the executive officer charged with carrying out the
knowledge that the premises had already been applied for by Tiburcio Ballesteros. As Andoy's entry was provisions of the Public Land Law, and he performs this duty through the Director of Lands (sec. 3). Subject
not made in good faith, her Sales Application No. 10960 is hereby rejected, and that of Ballesteros, given to the control of the executive head, the Director of Lands is by law vested with direct executive control
due course." over land matters, "and his decisions as to questions of fact shall be conclusive when approved by the
Secretary of Agriculture and Commerce." (Sec. 4). quasi-judicial officer. As such officer he makes findings
The said Barbara Andoy, defeated party in that decision and now dead, was the mother of the petitioner. of fact, even passes upon questions of mixed fact and law, and considers and decides the qualifications of
In order to determine whether or not to give due course to the sales application of the respondent, an applicants for the purchase of public lands. A discretion is lodged by law in the Director of Lands which
investigation was ordered of the land applied for. Public Land Inspector held a hearing at which Tiburcio should not be interfered with. The decisions of the Director of Lands on the construction of the Public Land
Ballesteros, on one hand, and the Jamisolas, on the other, presented their respective evidence. Director of Law are entitled to great respect by the courts.
Lands dismissed the claim of the Jamisolas and gave due course to the application of Tiburcio Ballesteros.
However, his power is not absolute and his decisions can be reviewed by the Courts through a direct
The Jamisolas appealed to the Secretary of Agriculture and Natural Resources who modified the decision
proceeding in showing that such decision is rendered in consequence of fraud, mistake, imposition, other
of the Director of Lands, holding that the petitioners herein were entitled to acquire by means of the sales
than error of judgment in estimating the value or effect of evidence. Since the Director of Lands erred in
patent application. The decision of the Department Head would appear to be based on the ground that the
its decision, it can be subjected to review by the Court.
Jamisolas were landless people, and pursuant to the policy of the Government to give land to the landless.

Ballesteros, moved the reconsideration of the said decision on the ground that the Jamisolas were not
landless people because on the contrary, they own and possess considerable landed properties. So the Vicente Villaflor vs. Court of Appeals
Secretary of Agriculture and Natural Resources reversed his decision.
FACTS
ISSUE: In 1940, Cirilo Piencenaves, in a Deed of Absolute Sale, sold to Vicente Villafor, a parcel of agricultural land
(planted with Abaca) containing an area of 50 hectares. The deed states that the land was sold to Villaflor
W/N the Director acted with grave abuse of discretion in rendering his decision.
in 1937, but no formal document was then executed, and since then until the present time, Villaflor has
W/N the Secretary had acted in excess of jurisdiction and with grave abuse of discretion in issuing his been in possession and occupation of the same. Before the sale of said property, Piencenaves inherited
orders said property from his parents and was in adverse possession of such without interruption for more than
50 years. On the same day, Claudio Otero, in a Deed of Absolute Sale sold to Villaflor a parcel of agricultural
The Director, who is the officer charged with carrying out the provisions of the Public Land Law has control land (planted with corn), containing an area of 24 hectares. Hermogenes Patete, in a Deed of Absolute
over the survey, classification, lease, sale or any other form of concession or disposition and management Sale sold to Villaflor, a parcel of agricultural land (planted with abaca and corn), containing an area of 20
of the lands of the public domain, His decision as to questions of fact, when approved by the Secretary of has., more or less. Both deed state the same details or circumstances as that of Piencenaves’. In 1940,
Agriculture and Natural Resources, is conclusive. He clearly acted within his jurisdiction. If he had erred in Fermin Bocobo, in a Deed of Absolute Sale sold to Villaflor, a parcel of agricultural land (planted with
appraising the evidence, the error is one of judgment, but not an act of grave abuse of discretion annullable abaca), containing an area of 18 hectares, more or less. In 1946, Villaflor leased to Nasipit Lumber Co.,
by writ of certiorari. Likewise the Secretary had not overreached his jurisdiction, nor were his acts Inc. a parcel of land, containing an area of 2 hectares, together with all the improvements existing thereon,
tainted by grave abuse of discretion. Concededly, as head of the Department of Agriculture and Natural for a period of 5 years at a rental of P200.00 per annum to cover the annual rental of house and building
Resources, he has the power to review, reverse, modify or affirm the decision of the Director. sites for 33 houses or buildings. In 1948, in an “Agreement to Sell” Villaflor conveyed to Nasipit Lumber, 2
parcels of land. From said day, the parties agreed that Nasipit Lumber shall continue to occupy the property
not anymore in concept of lessee but as prospective owners.
ORTUA VS. ENCARNACION
On 7 December 1948, Villaflor and Nasipit Lumber executed an “Agreement,” confirming an Agreement to
Facts: Petitioner Ortua filed an application to purchase a tract of public land located in Camarines Sur. It Sell, but with reference to the Sales Application filed with the Bureau of Land. Sales Application of Villaflor
was rejected by the Director of Land on the ground that petitioner is neither a Filipino citizen or an American were rejected for having leased the property to another even before he had acquired transmissible rights
citizen. Public Land Law, Act No. 2874, in its sections 23 and 88, provides that the purchaser shall be a thereto. In August 1950, Villaflor executed a document, denominated as a “Deed of Relinquishment of
citizen of lawful age of the Philippine Islands or of the United States. It was alleged by the respondent that Rights,” in favor on Nasipit Lumber, in consideration of the amount of P5,000 that was to be reimbursed
petitioner is a Chinese citizen. to the former representing part of the purchase price of the land, the value of the improvements Villaflor
introduced thereon, and the expenses incurred in the publication of the Notice of Sale; in light of his
Issue: whether or not Ortua is a Chinese citizen and not entitled to purchase the tract of public land. difficulty to develop the same as Villaflor has moved to Manila. Pursuant thereto Nasipit Lumber filed a
Sales Application over the 2 parcels of land. “Order of Award” was then issued in favor of Nasipit Lumber.
Ruling: The Court ruled in negative. Circumstances shows that Ortua is a Filipino citizen and thus entitled In 1973, Villafor wrote a letter to Nasipit Lumber, reminding the latter of their verbal agreement in 1955;
to purchase the tract of land. Further, the Court emphasize the powers lodged to the Director of Lands. but the new set of corporate officers refused to recognize Villaflor’s claim. In a formal protest dated 31
He performs his functions pursuant to the provisions of the Public Land Law. In accordance with this law, January 1974 which Villaflor filed with the Bureau of Lands, he protested the Sales Application of Nasipit
Envi Reviewer (MIDTERMS) Page |6

Lumber, claiming that the company has not paid him P5,000.00 as provided in the Deed of Relinquishment All told, the only disqualification that can be imputed to private respondent is the prohibition in the 1973
of Rights dated 16 August 1950. The Director of Lands found that the payment P5,000.00 in the Deed and Constitution against the holding of alienable lands of the public domain by corporations.54 However, this
the consideration in the Agreement to Sell were duly proven, and ordered the dismissal Court earlier settled the matter, ruling that said constitutional prohibition had no retroactive effect and
of Villaflor’s protest. He also contended that NASIPIT is disqualified to own lands due to the prohibition of could not prevail over a vested right to the land. In Ayog vs. Cusi, Jr.,55 this Court declared:
the 1973 Constitution. In 1978, Villaflor filed a complaint in the trial court for “Declaration of Nullity of “We hold that the said constitutional prohibition has no retroactive application to the sales application of
Contract (Deed of Relinquishment of Rights), Recovery of Possession (of two parcels of land subject of the Biñan Development Co., Inc. because it had already acquired a vested right to the land applied for at the
contract), and Damages”. In 1983, he died. The trial court ordered his widow, Lourdes D. Villaflor, to be time the 1973 Constitution took effect.
substituted as petitioner. CFI dismissed the complaint. The heirs of petitioner appealed to the Court of That vested right has to be respected. It could not be abrogated by the new Constitution. Section 2, Article
Appeals which, however, rendered judgment against them. Hence this petition. XIII of the 1935 Constitution allows private corporations to purchase public agricultural lands not exceeding
one thousand and twenty-four hectares. Petitioner’s prohibition action is barred by the doctrine of vested
ISSUE rights in constitutional law.
Whether or not CA erred in adopting the findings of the Bureau of Lands; and
Whether or not NASIPIT is qualified to acquire title over the disputed land.
PAAT v CA
HELD:
First Issue: Doctrine of Primary Jurisdiction.—Underlying the rulings of the trial and appellate courts is the FACTS: The truck of private respondent (Sps De Guzman) was seized by DENR personnel because it was
doctrine of primary jurisdiction; i.e., courts cannot and will not resolve a controversy involving a question transporting forest products without the required permit of the DENR which is in manifest contravention
which is within the jurisdiction of an administrative tribunal, especially where the question demands the of sec. 68 of P.D. 705 as amended by E.O. 277 (The Revised Forestry Code)
exercise of sound administrative discretion requiring the special knowledge, experience and services of the
administrative tribunal to determine technical and intricate matters of fact. Reliance by the trial and the As a result, private respondent filed a suit for replevin for the return of the truck. Petitioners filed a motion
appellate courts on the factual findings of the Director of Lands and the Minister of Natural Resources is to dismiss with the trial court contending, inter alia, that private respondents had no cause of action for
not misplaced. By reason of the special knowledge and expertise of said administrative agencies over their failure to exhaust administrative remedies.
matters falling under their jurisdiction, they are in a better position to pass judgment thereon; thus, their Private respondents contend that that their case falls within the exception of the doctrine upon justification
findings of fact in that regard are generally accorded great respect, if not finality, by the courts. The that (1) due process was violated because they were not given the chance to be heard (2)the seizure and
findings of fact of an administrative agency must be respected as long as they are supported by substantial forfeiture was unlawful (a) the secretary of DENR and his representative has no authority to confiscate and
evidence, even if such evidence might not be overwhelming or even preponderant. It is not the task of an forfeit conveyances utilized in transporting illegal forest products (b) that the truck was not used in the
appellate court to weigh once more the evidence submitted before the administrative body and to commission of the crime.
substitute its own judgment for that of the administrative agency in respect of sufficiency of evidence.
RTC denied the motion to dismiss and ruled in favor of the private respondent. CA likewise affirmed the
Second Issue: Petitioner asserts that private respondent was legally disqualified from acquiring the parcels RTC.
of land in question because it was not authorized by its charter to acquire disposable public agricultural
lands under Sections 121, 122 and 123 of the Public Land Act, prior to its amendment by P.D. No. 763. ISSUE: WoN without violating the Principle of Exhaustion of Administrative Remedies, may an action for
We disagree. The requirements for a sales application under the Public Land Act are: (1) the possession replevin prosper.
of the qualifications required by said Act (under Section 29) and (2) the lack of the disqualifications
HELD: NEGATIVE. The contention of the private respondent was without merit. The exhaustion of
mentioned therein (under
Administrative remedies may be disregarded when: 1) there is a violation of due process 2) when the issue
Sections 121, 122, and 123). However, the transfer of ownership via the two agreements dated July 7 and
involved is purely question of law 3) when the administrative action is patently illegal amounting to lack/
December 7, 1948 and the relinquishment of rights, being private contracts, were binding only between
excess of jurisdiction 4)when there is estoppel on the part of the administrative agency concerned 5) when
petitioner and private respondent. The Public Land Act finds no relevance because the disputed land was
there is irreparable injury 6) when the respondent is department secretary whose acts as an alter ego of
covered by said Act only after the issuance of the order of award in favor of private respondent. Thus, the
the President bears the implied and assumed approval of the latter 7) when to require exhaustion of
possession of any disqualification by private respondent under said Act is immaterial to the private contracts
administrative remedies would be unreasonable 8) when it would amount to the nullification of the claim
between the parties thereto. (We are not, however, suggesting a departure from the rule that laws are
9) when the subject matter is a private land in land case proceedings 10) when the rule does not provide
deemed written in contracts.) Consideration of said provisions of the Act will further show their
a plain, speedy and adequate remedy 11) when there are circumstances indicating the urgency of judicial
inapplicability to these contracts. Section 121 of the Act pertains to acquisition.s of public land by a
intervention.
corporation from a grantee, but petitioner never became a grantee of the disputed land. On the other
hand, private respondent itself was the direct grantee. Sections 122 and 123 disqualify corporations, which However, the case at bar does not fall under the aforementioned exceptions for they were given the
are not authorized by their charter, from acquiring public land; the records do not show that private opportunity to be heard and the DENR and its representative are authorized to order the confiscation and
respondent was not so authorized under its charter. forfeiture.
Envi Reviewer (MIDTERMS) Page |7

Also, it is important to point out that the enforcement of forestry laws, rules and regulations and the of Forestry then came to this Court in a petition for review on certiorari claiming that the land in dispute
protection, development, and management of forest lands falls within the primary and special was forestal in nature and not subject to private appropriation. He asks that the registration be reversed.
responsibilities of DENR. By the very nature of its function, the DENR should be given a free hand
unperturbed by judicial instruction to determine a controversy which is well within its jurisdiction. The Issue:
assumption by the trial court, therefore, of a replevin suit filed by private respondent constitutes an Whether or not Mangrove swamps/manglares form part of public forests and are therefore not alienable.
unjustified encroachment into the domain of administrative agency’s prerogative.
Held :
Yes. Mangrove swamps or manglares should be understood as comprised within the public forests of the
ALBA VS. CA Philippines as defined in Section 1820 of the Administrative Code of 1917. The legislature having so
Facts: An application was filed for title to land by Jose Lachica claiming that the land was purchased by determined, The judiciary cannot ignore or modify its decision, and in effect veto it, in the exercise of its
him and his wife from one Eulalio Raz. An order of general default was issued but opposed by Octabella own discretion. The statutory definition remains unchanged to date and, no less noteworthy, is accepted
de Raz, Spouses Braulio, Jose Rago representing Apolonia Rebeco and the Alba’s. and invoked by the executive department. More importantly, the said provision has not been challenged
as arbitrary or unrealistic or unconstitutional assuming the requisite conditions, to justify our judicial
Spouses Braulio filed their opposition in the registration alleging that they are the owners but the deed of intervention and scrutiny. The law is thus presumed valid and so must be respected. We repeat our
sale cannot be found in the records and theya have not presented any evidence. Their opposition was statement in the Amunategui case that the classification of mangrove swamps as forest lands is descriptive
dismissed. Jose rago representing Apolonia Rebeco has not represented evidence and manifested to file a of its legal nature or status and does not have to be descriptive of what the land actually looks like. That
motion for withdrawal of opposition so their petition was likewise dismissed. The remaining oppositors are determination having been made and no cogent argument having been raised to annul it, we have no duty
the Alba’s and Octabella De Raz. as judges but to apply it. And so we shall.

Rtc rendered judgement in favor of respondent applicant Lachica on the basis of the testimonial and
documentary evidence and the continuous payment of taxes in the said land. Remanining oppositors are CENON MATEO, vs. HON. FLORENCIO MORENO, in his capacity as SECRETARY OF PUBLIC
dissatisfied and interposed an appeal with CA in which the CA affirmed the decision of the same basis; tax WORKS AND COMMUNICATIONS
declaration, continuous possession and acquired the land by prescription.
FACTS:
Issue: W/N private respondent applicant is entitled to the confirmation of his ownership?
Sometime in 1959 a number of residents of Guiguinto, Bulacan, sent a letter-complaint to the Highway
District Engineer of that province asking that the Sapang Cabay, a public navigable stream, which had
Ruling: The controlling statute when private respondent applicant filed his application for registration is
been blocked by means of dikes and dams and converted into fishponds, be ordered reopened and restored
Sec 48 of CA 141 but the assailed decisions of CA affirming the ruling of RTC relied on the provisions of
to its original condition.
Sec 19 of Act 496 in relation to the Civil Code provisions on prescription on the assumption that the subject
land is private land, while the application of for registration of applicant is for judicial confirmation of an
Secretary rendered his decision on August 10, 1959, finding that the Sapang Cabay was a public navigable
imperfect title considering that the land is presumed part of public domain under Regalian Doctrine.
stream and ordering Cenon Mateo, the herein petitioner-appellant, who had in the meantime acquired the
property inside which the said creek is situated, to remove the dikes and dams therein constructed within
The land sought to be registered hardly falls under any of the classification of lands referred by ART 496.
thirty days from notice; otherwise they would be removed at his expense. Mateo moved to reconsider but
Prescription invoked by both lower courts cannot be pleaded to bolster applicants claim because no land
was turned down, whereupon he filed the basic petition to restrain the respondent Secretary from enforcing
can be acquired by private persons without any grant express or implied from the government. The
his decision.
possession of public agricultural land however long the period may have extended never confers title upon
the possession. Also Tax Declaration is not a conclusive evidence of ownership unless supported by other
ISSUE:
effective proof.
W/N the Secretary is correct in holding that Sapang Cabay is a public navigable steam
HELD:
YES.
THE DIRECTOR OF FORESTRY, petitioner vs. RUPERTO A. VILLAREAL In the present case the evidence shows that ocular inspection of the entire length of the creek up to
Guiguinto, shows that portions of the creek are closed by dikes, that traces of the path of the Cabay Creek
Facts : within petitioner-appellant's fishpond are still visible by the nature of the mud, that a man-made canal
Ruperto Villareal applied for the registration of a land which consists of 178,113 square meters of detours the creek from its original path, that during the flood season the creek is waistdeep and even
mangrove swamps located in the municipality of Sapian, Capiz. He was opposed by several persons, more, and surrounding ricelands are overflooded, that the surrounding ricelands are flooded during heavy
including the petitioner on behalf of the Republic of the Philippines. After trial, the application was approved rains because the man-made canal is not sufficient to contain the volume of water coming from the creek.
by the Court of First Instance. of Capiz. 1 The decision was affirmed by the Court of Appeals. 2 The Director
Envi Reviewer (MIDTERMS) Page |8

In addition, the documentary evidence shows that as long ago as 1941 there were already complaints Upon investigation, he discovered that the property is not an alienable or disposable land susceptible of
against the closure of the Sapang Cabay by the petitioner-appellant's predecessor-in-interest, Modesto private ownership.
Pascual; that the municipal council of Guiguinto passed a resolution on November 22 of the same year,
requesting the Secretary of Public Works and Communications to order the removal of the obstruction; ISSUE:
that the administrative proceedings for that purpose were interrupted by the war, but reopened in 1948,
and again in 1952, pursuant to similar resolutions of the same municipal council; and that in 1954 the W/N The land in question was validly subjected to alienation
Secretary of Public Works and Communications rendered a decision ordering Encarnacion Jacobo, who was
then the owner from whom the petitioner-appellant subsequently bought the property, to remove the dikes RULING:
she had constructed. It is true that Encarnacion Jacobo was able to get her free patent application approved
in 1953 and to secure the corresponding certificate of title, but said title did not change the public character
In the case at bar, as reflected above, complainant presented certifications from the DENR that the property
of the Sapang Cabay.
is part of the public domain and not disposable as it is within the Bataan National Park. Indeed, by virtue
of Proclamation No. 2445 issued on December 1, 1945, all properties of the public domain therein
designated as part of the Bataan National Park were withdrawn from sale, settlement or other disposition,
WILSON PO CHAM, Complainant, v. ATTY. EDILBERTO D. PIZARRO, Respondent. subject to private rights.
FACTS:
On the other hand, respondent has utterly failed to substantiate his documented claim of having irrevocable
Sometime in July 1995, Emelita Canete Elenita Alipio and now deceased Mario Navarro who was then the rights and interests over the property which he could have conveyed to complainant. E.g., he could have
Municipal Assessor of Morong, Bataan, offered for sale to him a parcel of land with an area of approximately presented any document issued by the government conferring upon him and his alleged co-owners, or
forty (40) hectares, identified as Lot 1683 of Cad. Case No. 262, situated at Sitio Gatao, Nagbalayong, even upon his alleged predecessors-in-interest, with any such right or interest, but he presented none. He
Morong, Bataan (the property). merely presented a Deed of Absolute Sale purportedly executed by a certain Jose R. Monzon in his,
Banzon's and Zabala's favor on July 25, 1995, a month shy of the execution on August 21, 1995 of the
He having expressed interest in the offer, Canete and Navarro arranged a meeting between him and Deed of Absolute Sale in favor of complainant.
respondent at the latter's residence in Balanga, Bataan1 where respondent categorically represented to
him that the property being offered for sale was alienable and disposable. 2 Respondent in fact presented The tax declaration and receipt which respondent presented do not help his cause any as neither tax
to him 1) Real Property Tax Order of Payment3 dated July 10, 1995 covering the property signed by Edna receipts nor realty tax declarations are sufficient evidence of the right of possession over realty unless
P. Pizarro as Municipal Treasurer and Navarro as Municipal Assessor; 2) a Deed of Absolute Sale 4 dated supported by other effective proof.46 The presentation of a tax declaration must indeed have been a
July 25, 1995 purportedly executed by the alleged previous actual occupant of the property, one Jose R. "pretext," as observed by the PENR in its earlier-quoted portion of its letter-directive to the Balanga
Monzon (Monzon), transferring all his rights, interest and possession thereover in favor of Virgilio Banzon Municipal Assessor "that the area occupied . . . [is] within alienable and disposable land." Respondent must
(Banzon), Rolando B. Zabala (Zabala) and respondent for an agreed consideration of P500,000.00; and 3) thus be faulted for fraudulently inducing complainant to purchase, for P3,372,533.00, non-existent
Special Power of Attorney5 dated July 25, 1995 executed by Banzon and Zabala authorizing him "irrevocable rights, interest and participation" over an inalienable property.
(respondent) to:
REPUBLIC OF THE PHILIPPINES, petitioner, vs. CELESTINA NAGUIAT, respondent.
a) offer to sell [their] rights over a certain parcel of land
b) negotiate and enter into a contract for the consumation (sic) of sale of the subject property; FACTS:
and to sign the same.
Celestina Naguiat filed an application for registration of title to four parcels of land located in Panan,
Botolan, Zambales. The applicant alleges that she is the owner of the said parcels of land having acquired
On August 21, 1995, respondent executed a Deed of Absolute Sale11 over the property in his favor. them by purchase from its previous owners and their predecessors-in-interest who have been in possession
Respondent thereafter furnished him with a copy of Tax Declaration No. 5010 13 with Property Index No. thereof for more than thirty (30) years; and that to the best of her knowledge, said lots suffer no mortgage
018-08-004-05-126 issued in his (respondent's) name and his alleged co-owners, and Real Property Tax or encumbrance of whatever kind nor is there any person having any interest, legal or equitable, or in
Receipt No. 02520114 dated August 17, 1995 issued in his (respondent's) name. possession thereof.

He thus gave respondent two checks dated August 21, 1995 representing the purchase price of the rights Petitioner Republic opposed on the ground that neither the applicant nor her predecessors-in interest have
over the property, Asian Bank Corporation Check No. GA06321015 in the amount of P168,627.00 payable been in open, continuous, exclusive and notorious possession and occupation of the lands in question since
to respondent, and Asian Bank Manager's Check No. 004639GA16 in the amount of P3,193,906.00 payable 12 June 1945 or prior thereto, considering the fact that she has not established that the lands in question
to respondent, Banzon and Zabala. He subsequently took possession of the property and installed a barbed have been declassified from forest or timber zone to alienable and disposable property.
wire fence at its front portion. Soon after, however, a forest guard approached him and informed him that
the property could not be fenced as it was part of the Bataan National Park.17 ISSUE:
Envi Reviewer (MIDTERMS) Page |9

Did the areas in question cease to have the status of forest or other inalienable lands of the public domain? There is no question that the lands in the case at bar were not alienable lands of the public domain. As
testified by the District Forester, records in the Bureau of Forestry show that the subject lands were never
HELD: declared as alienable and disposable and subject to private alienation prior to 1913 up to the present.
Moreover, as part of the reservation for provincial park purposes, they form part of the forest zone.
No, the said areas are still classified as forest land.The issue of whether or not respondent and her
predecessors-in-interest have been in open, exclusive and continuous possession of the parcels of land in
It is elementary in the law governing natural resources that forest land cannot be owned by private persons.
question is of little moment. For, unclassified land cannot be acquired by adverse occupation or possession;
It is not registrable and possession thereof, no matter how lengthy, cannot convert it into private property,
occupation thereof in the concept of owner, however long, cannot ripen into private ownership and be
unless such lands are reclassified and considered disposable and alienable.
registered as title.
Since 1,976 square meters of the 3,384 square meters covered by TCT 3913 fall within the reservation,
A forested area classified as forest land of the public domain does not lose such classification simply TCT 3913 should be annulled only with respect to the aforesaid area. It has been held that where a
because loggers or settlers have stripped it of its forest cover. Parcels of land classified as forest land may certificate of title covers a portion of land within the area reserved for park purposes, the title should be
actually be covered with grass or planted to crops by kaingin cultivators or other farmers. "Forest lands" annulled with respect to said portion.
do not have to be on mountains or in out of the way places. The classification is merely descriptive of its
legal nature or status and does not have to be descriptive of what the land actually looks like.
IGNACIO MESINA, VS. EULALIA PINEDA VDA. DE SONZA, ET AL

PALOMO V. COURT OF APPEALS FACTS:


P l a i n t i f f a l l e g e s t h a t h e i s t h e o w n e r i n f e e s i m p l e o f t h e Lot No. 3259, w i t h
Facts: William Cameron Forbes, then Governor General of the Philippines, issued Executive Order No. 40 t h e improvements thereon, situated in San Antonio, Nueva Ecija, and that he has been in actual
which reserved for provincial park purposes some 440, 530 square meters of land situated in Barrio Naga, possession thereof since 1914, publicly, openly, peacefully and against the whole world, and that up to
Municipality of Tiwi, Province of Albay pursuant to the provisions of Act 648 of the Philippine Commission. the present time he is the only one who benefits from the pro duce thereof. The said lot is also
The CFI of Albay ordered the registration of 15 parcels of land covered by EO No. 40 in the name of Diego the subject of registration proceedings pending in the same court known as Registration Case
Palomo. These parcel of lands allegedly covered by OCT Nos. 513, 169, 176, and 173 were donated to the No. 372, L. R.C. Cad Record No. 12238. The Director of Lands issued a homestead patent in
herein petitioners, Ignacio and Carmen Palomo. Claiming that these OCTs were lost during the Japanese favor of the defendants but did not exercise due care, in spite of his knowledge that defendants
occupation, Ignacio Palomo filed a petition for reconstitution with the CFI of Albay on May 30, 1950. The had not complied with the requirements set forth in Commonwealth Act 141. Plaintiff claims
Register of Deeds issued TCT Nos. 3911, 3912, 3913, and 3914 in October 1953. President Ramon that title was procured by defendants through fraud, deception and misrepresentation since
Magsaysay issued Proclamation No. 47 on July 10, 1954 converting the area embraced by EO No. 40 into they knew that the lot belonged to the plaintiff.
the “Tiwi Hot Spring National Park.” The area was never released as alienable and disposable portion of
the public domain and, therefore, neither susceptible to disposition under the provisions of the Public Land Issue: WON the requirement set forth in Republic Act No. 1942, which took effect on June 22, 1957
Law (CA 141) nor registrable under the Land Registration Act (Act No. 496). However, the Palomos amending Section 48-b of Commonwealth Act 141 has been complied with.
continued in possession of the property, paid real estate taxes, and introduced improvements on those
lands. On May 1974, the petitioners filed a civil case against the private respondents who entered the land Held: Yes. Republic Act No. 1942, which took effect on June 22, 1957 amending Section 48-b of
covered by TCT No. 3913 and/or TCT No. 3914 and cut down bamboos thereat. On October 1974, the Commonwealth Act 141 provides:
Republic of the Philippines filed a civil case for annulment and cancellation of Certificates of Title involving
the 15 parcels of land registered in the name of the petitioners. The trial court ruled in favor of the Republic (b) Those who by themselves or through their predecessors in interest have been in open, continuous,
as they found no sufficient proof that the Palomos have established property rights over the parcels of land exclusive and notorious possession and occupation of agricultural lands of the public domain, under a bona
in question. The Palomos obtained no right at all over the properties because these were issued only when fide claim of acquisition of ownership, for at least thirty years immediately preceding the filing of the
EO No. 40 was already in force. The petitioners appealed to the CA which affirmed in toto the findings of application for confirmation of title except when prevented by war or force majeure. These shall be
the lower court. Hence, this petition. conclusively presumed to have performed all the conditions essential to a Government grant and shall be
entitled to a certificate of title under the provisions of this chapter.
Issue: Whether or not the alleged OCTs issued pursuant to the order of the CFI and the subsequent TCTs
issued pursuant to the petition for reconstitution are valid. In the case of Susi v. Razon, all the necessary requirements for a grant by the Government are complied
with through actual physical possession openly, continuously and publicly. The possessor is deemed to
Held: No proof was presented that the petitioners’ predecessors in interest derived title from an old have already acquired by operation of law not only a right, but a grant from the government.
Spanish grant. The aforesaid “decisions” of the CFI were not signed by the judge but were merely certified
copies of notification to Diego Palomo bearing the signature of the clerk of court. Plaintiff is said to have acquired the lot by the grant of the State, it follows that same had ceased to be a
part of the public domain and had become private property and, therefore, is beyond the control of the
Director of Lands. Consequently the homestead patent and the original certificate of title covering said land
E n v i R e v i e w e r ( M I D T E R M S ) P a g e | 10

issued by the Director of Lands in favor of the defendants can be said to be null and void, for having been The Director of Lands seasonably filed an opposition on the ground that neither the applicants nor their
issued through fraud, deceit and misrepresentation. predecessor-in-interest possess sufficient title to acquire ownership in fee simple of the parcels of land
applied for; that they have not been in open, continuous, exclusive and notorious possession and
occupation of the land in question for at least thirthy (30) years immediately preceding the filing of the
Director of Lands vs. IAC and ACME present application; and that these parcels of land are portions of the public domain belonging to the
Republic of the Philippines, and therefore, not subject to appropriation
Facts:
In order to establish thirty (30) years of open and continuous possession over the subject property, private
Respondent Ace Plywood and Veneer Co. INC. ordered registration of fiver parcels of land which it acquired respondents presented Crisanto Angeles and Monico Balila, Crisanto Angeles claimed that he first took
from members of the Dumagat Tribe on October 29, 1962. It applied for registration on July 17, 1981. possession of these two (2) parcels of land in the year 1931 and subsequently sold it to respondents in
Herein petitioner is contending that the 1973 Constitution is the applicable law in the case at bar which 1972. Private respondent Silvestre Manlapaz also testified that upon their acquisition of the two (2) parcels
prohibits private corporations or associations from holding alienable lands of the public domain except by of land designated as Lots 2855 and 2856, they immediately took possession of the same, planted
lease no to exceed 1,000 hectares. This prohibition was not found in the 1935 constitution which is the coconuts, camotes and other vegetables and expanded the portion planted to palay.
applicable law in 1962 when the respondent corporation acquired the land.
The Trial Court ruled in favor of the spouses granting the two prcels of lands under their name. On appeal,
Issue: the Court of Appeals affirmed the decision of the RTC since the Dir of Lands did not present any evidence
to support his contention. Respondent Court erred in ruling that petitioner failed to raise the defense of res
Whether or not the title acquired by ACME in 1962 could be confirmed in their favor in proceedings instated judicata in the trial court and, hence, waived the same.
in 1982 when the 1973 Constitution was already in effect.

Held: ISSUE:
The issue is whether or not the Spouses met the requirements to be awarded as the owners of the subject
Yes. Land registration is deemed as a judicial confirmation proceeding that should at most be limited to parcels of land.
ascertaining whether the possession claimed is of the required character and length of time as it is not so
much one to confer title as it is to recognize a title already vested. Applying it to the case, the respondent
RULING:
corporation already acquired ownership and title to the land in 1962 and that registration is merely to
The Court said NO.
establish the ownership. Furthermore, the 1973 Constitution cannot impair vested rights so the prohibition
on private corporations cannot be applicable.
The Supreme Court ruled that a decision in a cadastral proceeding declaring a lot public land is not the final decree
contemplated in Section 38 and 40 of the Land Registration Act.

DIR. OF LANDS V. CA AND SPOUSES MANLAPAZ AND PIZARRO


A Judicial declaration that a parcel of land is public, does not preclude even the same applicant from subsequently
FACTS: seeking a judicial confirmation of his title to the same land, provided he thereafter complies with the provisions. of
Section 48 of Commonwealth Act No. 141, as amended, and as long as said public land remains alienable and disposable
This is a review on certiorari seekig the reversal of the Decision of Court of Appeals affirming the Decision (now section 3 and 4, PD No. 1073,)
of the then Court of First Instance of Bataan adjudicating in favor of the herein private respondents the
subject two parcels of land. Crisanto Angeles He claimed that he was in possession of the land way back in 1930. Yet he declared the
same for taxation purposes only in 1966. Although tax receipts are not incontrovertible evidence of
The facts are as follows: ownership, they constitute at least proof that the holder had a claim of title over the property. 24 He stated
that he knew the owners of the adjoining properties, but during the cross-examination, he was unable to
On January 1973, herein respondents filed an application before the CFI of Bataan seeking the registration
give their names. Nor was he able to explain how he came into possession of the parcel of land and there
and confirmation of titles to two parcels of land , designated as Lot No. 2855 and Lot No. 2856
is no showing of any title, perfect or imperfect, granted by the state to him or his predecessors.
Trial court ordered the Land RegComm. To submit reports whether not the parcels of land in question had
been issued patents or whether the same are subject of pending decrees The documents introduced by the applicants merely evidenced the fact that the parcels of land applied for
were alienable and disposable lands of the public domain,25 but no document has been presented that
are portions of Lot 2749, Cad. 241, Orion Cadastre and that the same have been the subject of registration would clearly establish the length of time of the possession of their predecessors-in-interest
proceedings in Court Cadastral Case No. 15, LRC (GLRO) Cadastral Record No. 1021 wherein a decision
has been rendered although there is no existing record of the same on file because it was among those
records lost or destroyed due to the ravages of the last global war Undoubtedly, the private respondents have failed to submit convincing proof of their predecessors-in-
interest's actual, peaceful and adverse possession in the concept of owner of the lots in question during
E n v i R e v i e w e r ( M I D T E R M S ) P a g e | 11

the period required, by law. This is of utmost significance in view of the basic presumption that lands of Heirs of Marasigan V Intermediate Appellate Court and Maria Marron
whatever classification belong to the State and evidence of a land grant must be "well-nigh incontrovertible
CA reversed and set aside. Facts:

The case involves a disputed property identified as Lot 2-A owned by Spouses Fe and Felicisimo Bazar
The Roman Catholic Archbishop of Manila, applicant and appellee, v. The Director of Land,
Gregoria Fajardo, et al., objectors and appellants. On April 24, 1975 a civil case was filed by Maria Marron which seeks to compel the spouses Bazar to
execute a registrable Deed of Absolute Sale in favor of her. On January 27, 1976, while the case was still
Facts: pending, Maria Marron caused the annotation of a notice of lis pendens at the back of the Transfer
Certificate of Title of the spouses Bazar. On February 24, 1976 the Court of First Instance rendered
On November 30, 1909, counsel for the Roman Catholic Archbishop of Manila, a corporation sole judgement in favor of Maria Marron
represented by Mgr. J. J. Harty, applied to the Court of Land Registration for the inscription in conformity
with law of four parcels of land, to wit: one parcel situated in the sitio of Mabolo, barrio of Santa Isabel; *A notice of lis pendent means that a certain property is involved in litigation and serves as a notice
another parcel in the sitio of Daquila of the same barrio of Santa Isabel; another parcel in the sitio of to the world that one who buys the same does it at his own risk.
Gatmuala of the barrio of Ligas; and, finally, another parcel situated in the sitio of Murasan of the barrio
of Ticay, all of which parcels of land being in the district of the pueblo of Malolos, Province of Bulacan. Prior to the Civil Case instituted by Maria Marron, on December 18, 1974, a Deed of Absolute Sale of Lot
Against this application adverse claims were filed by Gregoria Fajardo and 32 other objectors who claim 2-A was executed by Fe Bazar in fav or of Maria Marasigan for the amount of Php. 15,000. However, it
that they have been in quiet and peaceable possession for years of the said land. All the adverse claims was only on July 5, 1977 that the said deed was registered with the Registry of Deeds of Manila.
filed by the occupants of these lands were disallowed and the Court of Land Registration on April 29, 1911, Consequently, the Transfer Certificate of Title of the spouses were cancelled and a new title was issued in
decreed the registration and adjudication of the four parcels of land applied for registration and Marasigan’s name and likewise the notice of lis pendens caused to be annotated by Marron was carried
adjudication of the four parcels of land applied for. An appeal was taken, and the SC, in a decision rendered over to the new title.
on August 8, 1913, affirmed the judgment of the lower court in so far as it decreed the registration of the
two parcels of land situated in Gatmuala and Murasan, and reversed the said judgment in so far as it On May 26, 1977, the Bazarz filed a petition for relief from judgement dated February 24, 1976 and while
decreed the registration of the two parcels situated in the sitios of Mabolo and Daquila of the barrio of their petition was pending, the spouses moved to set aside said judgement on June 22, 1979 on the ground
Santa Isabel. Counsel for the petitioner on March 14, 1914, filed a new application in the same Court of of lack of jurisdiction. On September 6, 1979, Marron filed a case in the Court of First Instance of Manila
Land Registration for the registration in conformity with law, of the two parcels of land situated in which seeks to nullify Marasigan’s title over the subject lot.
the sitios of Mabolo and Daquila - the same two parcels of land whose registration in favor of the petitioner
had been denied by this court in its decision of August 8, 1913. The Director of Lands and the 33 objectors However, the case was dismissed on the ground that Marron’s complaint was premature since the
opposed the said application setting up the plea of res judicata. judgement rendered by court of First Instance had not yet become final and executory. On appeal,
Intermediate Appellate Court ruled that Marron is entitled to the subject property by virtue of the notice
Issue: of lis pendens and that the decision of CFI had become final and executory because the petition for relief
from judgement by the spouses Bazar was filed out of time.
Whether or not the applicant is barred from applying for registration again the said two (2) parcels of land
by reason of res judicata. Issue:

Held: Who between Maria Maron and Maria Marasigan has the better right over the subject lot.

Ruling:
The decree of the court, denying the registration sought, means nothing more than the respect
recommended by the law for the possession which the occupant enjoys, inasmuch as the applicant has not The court ruled that Maria Marron has the better right over the subject property
shown a better right or title to the real properties in question; but the said decree, denying the registration,
does not declare the possessors of the lands sought to be registered are the owners and proprietors It was shown that Maria Marasigan acquired the subject property 4 months before Maria Marron filed a
thereof, and therefore that order denying the registration does not produce the effects of res judicata on civil case. However, the transaction between Marasigan and spouses Bazar became effective only as
the new petition filed by the plaintiff for the registration of the two parcels of land in litigation, as the said against third person on July 5, 1977 when it was registered in the registry of deeds. Thus, there is no
decree does not contain any acknowledgment or declaration of ownership which would bar perpetually and question that when the Registry of Deeds issued the new Certificate of Title to Marasigan, the notice of lis
absolutely a third person, who is the applicant, from proving, as it has done, that it is the actual and lawful pendens was carried over to such title Because of the lis pendens, it was a clear notice to Marasigan that
owner of the lands in dispute, and that the objectors are no more than tenants and precarious holders of there was a court case affecting her rights to the property she purchased. It is an established rule that the
a major part of the lands in question, a portion of which is not held or occupied by anyone at the present filing of notice of lis pendens charges all strangers with a notice of the particular litigation referred to
time except by the applicant itself and no objector came forward to lay claim to it. therein and therefore, any right they may thereafter acquire on the property is subject to the eventuality
E n v i R e v i e w e r ( M I D T E R M S ) P a g e | 12

of the suit. In the present case, Maria Marron was granted with the right over Lot 2-A therefore Maria good faith is devoid of merit. In the first place, there is nothing in the records tending to show that they
Marasigan lost her right over the lot. took steps to contest the issuance of TCT No. 268893 in the name of petitioner Gonzales after the
cancellation of TCT No. 83217/T-146 in the name of Esteban Raterta as well as TCT No. 135516 in their
name which emanated from Raterta's certificate of title. Private respondents cannot herein assail the title
GONZALES VS COURT OF APPEALS of petitioner Gonzales.

FACTS: As a rule, a Torrens Title is irrevocable and indefeasible, and the duty of the court is to see to it that it is
maintained and respected unless challenged in a direct proceeding. By express provision of Sec. 48, P.D.
People's Homesite and Housing Corporation (PHHC) awarded the subject property, Lot 29, Block E-418, 1529, it is clear that a certificate of title cannot be subject to collateral attack. The respondent Santos
covered by TCT No. 1356 of the Register of Deeds of Quezon City, to petitioner Venancio Gonzales. brothers and their alleged tenant Julian Aguilar have no more right to remain in the promises. The trial
However, it was subsequently cancelled and the lot was "re-awarded" to Esteban Raterta. Petitioner court is directed to enforce immediately the writ of possession issued in favor of herein petitioner Venancio
Gonzales commenced Civil Case No. Q-5550 in the Court of First Instance of Rizal, at Quezon City, praying Gonzales in Civil Case No Q-5550 against persons in occupancy.
that PHHC be ordered to execute the corresponding contract of sale in his favor.

The trial court dismissed the case but directed PHHC to reimburse petitioner Gonzales the amount of
REPUBLIC VS UMALI
P1,576.00 paid on the lot. Petitioner Gonzales elevated the case to the Court of Appeals, docketed as CA-
G.R. No. 31757 R, which on January 31, 1969, rendered a decision in his favor. Ordering PHHC to cancel FACTS:
or revoke the award made toEsteban Raterta being null and void, And to execute the corresponding
Contract to Sell over the lot in favor of Gonzales.6 During the pendency of Civil Case No. Q-5550, the lot Petitioner Republic seek the reversion of a parcel of land as the original sale of the same from the
was sold by Esteban Raterta to Dalmacio Raterta who in turn sold the property to private respondents government was tainted with fraud and done with forgery.
Rafael, Mario and Vicente, all surnamed Santos.
The subject land is located in Tanza, Cavite with an area of 78, 865 sq.m. In the year 1910, Florentina
Thereafter, petitioner Gonzales sought execution of the judgment of the Court of Appeals in CA-G.R. No. Bobadilla purchased the said land on installment from the government, and subsequently, OCT No. 180
31757-R. The move was opposed by PHHC CFI of Rizal issued an Order stating inter alia: was issued in her favor. Florentina Bobadilla then transferred her rights in favor of Martina, Maria, Gregorio,
. . . the plaintiff pointed out that the records of this case have long been forwarded to this court by the Tomasa and Julio, all surnamed Cenizal. Later on, Tomasa and Julio assigned their shares to the remaining
Court of Appeals, however, it is unfortunate and regrettable that said records cannot be found among the three, Martina, Maria and Gregorio. In the year 1971, the three assignees allegedly signed a Joint Affidavit
records of cases of this court despite diligent efforts to locate the same . . . Considering that the records which they filed with the Bureau of Lands claiming that they’re entitled to the issuance of Certificate of
of this case was (sic) lost or misplaced through no fault of the herein plaintiff, in counting the reglementary Title considering that they had already paid in full for the subject property. The Secretary of Agriculture
period during which the writ could not he served, that is when the record of this case cannot be found, executed Deed No. V-10910 in favor of the three assignees. In the same year, TCT No 55044 was issued
shall be deducted or subtracted. 10 replacing OCT No. 180 by the Register of Deeds of Cavite in favor of Maria and Gregorio Cenizal, and
Rosalina, Luz and Enrique, in lieu of Martina Cenizal.
In the meantime, after securing the legal opinion of the Government Corporate Counsel.The PHHC
In 1985, a complaint for reversion was filed by the petitioner asking the present owners of the property,
voluntarily executed the judgment in CA-G.R. No. 31757-R by cancelling and revoking the award in favor
namely, Remedios, Juan, Luz and Enrique, to return the property to the State for forgery and fraud. The
of Esteban Raterta and executing a deed of sale in favor of petitioner Gonzales. On May 26, 1980,
petitioner contended that Gregorio died in 1943 while Maria on 1959, therefore, they could not have signed
petitioner Gonzales filed in Civil Case No. Q-5530 a motion for writ of possession.Respondent Court set
the Joint Affidavit on 1971 which Deed No. V-10910 was based.
aside the writ of execution issued by the trial court. Two motion for consideration of Gonzales was denied.
Hence, this petition for review on certiorari. In their answer, private respondents filed their motion to dismiss and argued that they acquired the
property in good faith and for value and that the government is not the real party in interest as the subject
ISSUE: land was covered by Torrens System.
Whether petitioner is entitled to a writ of possession.
RTC granted the motion to dismiss, hence this present petition.
HELD:
The Court ruled that if the delays were through no fault of the prevailing party, the same should not be ISSUE:
included in computing the 5-year period to execute a judgment by motion. The motion for execution was
WON the sale should be considered null and void ab initio given such deception
filed within the period.
HELD
Therefore, it is thus clear that petitioner had never slept on his rights as the delay in the execution was
beyond his control. Regarding the contention of private respondents that they are entitled to the property The court agreed that there was obviously forgery as the two supposed applicants were already dead at
in question on the ground that they are not parties to said case because they acquired the property in the time the Joint Affidavit was executed and that the 3 signatures were written by one and same hand.
E n v i R e v i e w e r ( M I D T E R M S ) P a g e | 13

Nonetheless, considering that the status of the private respondents as innocent purchasers were never Gregorio Agunoy, Sr. has not occupied and cultivated the land in the manner and for the length of time
raised nor having been disapproved, under the protection of Torrens System, the title is considered required by law
conclusive and indefeasible, and its status will not change despite the flaw in TCT No.55044. Furthermore,
Section 39 provides that if a person acquired or received a Certificate of Title and subsequent purchaser The Court of Appeals declared that Agunoy Sr, had validly and properly acquired Free Patent No. 314450
takes Certificate of Title FOR VALUE IN GOOD FAITH, the same shall hold free from all encumbrances. In and the corresponding Original Certificate of Title No. P-4522
addition to that, the private respondents acquired the land not by direct grant but by several transfers
following the original sale to Florentina Bobadilla in 1910. Considering that the land was registered under
ISSUE: W/N WHETHER OR NOT THE COURT OF APPEALS ERRED IN DECLARING THAT PETITIONER IS
Torrens System, the government has no more control and jurisdiction over it as the same ceases to become
NOT THE REAL PARTY-IN-INTEREST IN THIS CASE AND THAT GREGORIO AGUNOY, SR. HAD VALIDLY
a public land. Petition is denied.
ACQUIRED FREE PATENT NO. 314450 AND ORIGINAL CERTIFICATE OF TITLE NO. P-4522.

RULING:
REPUBLIC OF THE PHILIPPINES, petitioner, vs. GREGORIO AGUNOY, SR., et al., SPOUSES
EDUARDO and ARCELITA MARQUEZ and RURAL BANK OF GAPAN, NUEVA ECIJA, respondents.
It bears stressing that, by petitioner’s own judicial admission, the lots in dispute are no longer part of the
public domain, and there are numerous third, fourth, fifth and more parties holding Torrens titles in their
FACTS: favor and enjoying the presumption of good faith. The doctrine to follow is simple enough: a fraudulent or
forged document of sale may become the ROOT of a valid title if the certificate of title has already been
Gregorio Agunoy, Sr. filed his application for Free Patent covering two parcels of land identified as Lot Nos. transferred from the name of the true owner to the name of the forger or the name indicated by the forger.
1341 and 1342, containing an aggregate area of 18.6486 hectares with the Bureau of Lands. On he was The nullification of defendant’s title would not result in the reversion of the land to the state but remains
issued Free Patent No. 314450 by the Director of Lands. The Register of Deeds of Nueva Ecija registered in private property. Consequently, the government, not being the real party-in-interest, is without
the Free Patent and issued the corresponding Original Certificate of Title (OCT) No. P-4522 in the name of personality to institute the action for reversion.
respondent.

The heirs of Eusebio Perez filed a formal protest with the Bureau of Lands alleging that Lot 1341 of the Sabas H. Homena and Iluminada Juaneza, plaintiffs-appellant, v. Dimas Casa and Maria Castor
covered by Original Certificate of Title No-P4522 is identical to Lots 1 and 2 of Plan Psu-47200 which had and the Register of Deeds for the Province of Cotabato, defendants-appellees.
been adjudicated as private property of said protestant. Bureau of Lands, conducted a formal investigation
and ocular inspection of the premises and it was ascertained that Free Patent No. 314450 and its Facts:
corresponding OCT No. P-4522 were improperly and fraudulently issued.
Plaintiffs-appellants filed a complaint for alleged unlawful acts of dispossession disturbing plaintiffs’
peaceful, continuous, open, uninterrupted, adverse and public possession against spouses Dimas Casa and
Upon the death of the wife of Gregorio Agunoy, Sr., the heirs, executed a Deed of Extrajudicial Partition Maria Castor. Likewise, plaintiffs sought to annul the original certificate of title issued by the Register of
with Sale in favor of Joaquin Sangabol for and in consideration of the sum of Twenty Thousand Pesos. The Deeds for the province of Cotabato in favour of defendant spouses pursuant to a Homestead Patent on
Original Certificate of Title No. P-4522 was cancelled by the Register of Deeds of Nueva Ecija and Transfer the ground that said patent was obtained by defendant spouses through fraud and misrepresentation by
Certificate of Title (TCT) No. 166270 was issued in favor of the aforenamed heirs. Said TCT No. 166270 stating, among others, in their application that the lot was not claimed and occupied by another person.
was again cancelled by reason of the concurrent sale to Joaquin Sangabol in whose favor TCT No. NT - Plaintiffs alleged that on June 15, 1967, they purchased from the defendants two (2) hectares of the
166271 was issued. aforementioned parcel of land, it being agreed in the deed of sale that the said portion would be reconveyed
to plaintiffs after the five-year prohibitory period, as provided for in the Homestead Patent Law, shall have
The Chief of the Legal Division recommended to the Director of Lands that court action be instituted for elapsed, and that defendants failed to abide by said agreement.
the cancellation of Free Patent No. 314450 and its corresponding Original Certificate of Title No. P-4522 in
the name Gregorio Agunoy, Sr., as well as other subsequent transfer certificates of title issued therefrom The CFI issued an order dismissing the complaint. Thereafter, the CA certified the case to the SC as it
based on the foregoing findings. Petitioner, Republic of the Philippines, thru the Office of the Solicitor involved only questions of law.
General, filed the complaint in this case against several defendants, among whom are the herein Issue:
respondents. petitioner Republic alleged that Free Patent No. 314450 and its corresponding Original
Certificate of Title No. P-4522 were procured by defendant Gregorio Agunoy, Sr., through fraud, deceit Whether or not the CFI erred in dismissing the complaint filed by plaintiffs-defendants.
and misrepresentation. The fraudulent acts and misrepresentation of defendant Gregorio Agunoy, Sr. had
misled the then Bureau of Lands in issuing said patent. Since the property in question was no longer a Held:
disposable public land, Free Patent No. 314450 and its corresponding Original Certificate of Title No. P-
4522 issued to defendant Gregorio Agunoy, Sr. are null and void and should be cancelled. Moreover,
E n v i R e v i e w e r ( M I D T E R M S ) P a g e | 14

The Court held in the negative. Basically, the plaintiffs' supposed cause of action rests upon the deed of Lee Chuy Realty Corporation vs CA and Marc Realty And Development Corporation
sale executed by defendants in their favor on June 15, 1962 wherein the latter sold a two-hectare portion GR no. 104114
of the homestead which they were applying for to the plaintiffs on the understanding that the actual
conveyance of the said portion to plaintiffs would be made only after the lapse of the five-year period FACTS:
during which, under the Public Land Act, the homestead owner was prohibited from transferring his rights. A valuable piece of land located at Meycauyan, Bulacan, with an area of 24,576 sq. m. and covered by
The agreement is clearly illegal and void ab initio; it is intended to circumvent and violate the law. As OCT No. 0-5290 is disputed by Lee Chuy Realty Corporation and Marc Realty and Development Corp. Such
parties to a void contract, the plaintiffs have no rights which they can enforce and the court cannot lend land was originally co-owned by Ruben Jacinto(one-sixth), Dominador, Arsenio, Liwayway all surnamed
itself to its enforcement. Plaintiffs can neither invoke the doctrine of implied trust based on an illegal Bascara and Ernesto jacinto(collectively owned the remaining five-sixths).
contract. The issue of prescription or laches becomes irrelevant in a case such as this, where plaintiffs On Feb. 4, 1981, Ruben Jacinto sold his one-sixth pro-indiviso share to LEE CHUY REALTY which was
clearly have no cause of action. “What cannot be done directly, cannot be done indirectly.”-Judge Sawali registered 30 April 1981. On 5 May 1989 the Bascaras and Ernesto Jacinto also sold their share to MARC
REALTY which was registered on 16 October 1989.

Republic vs. Garcia Lee Chuy Realty claims that it was never informed of the existence of the sale between Marc Realty and
Facts: the Bascaras/Jacinto. Marc Realty insists that Lee Chuy verbally notified of the sale and was given a copy
of the deed of sale. On 13 November 1989 LEE CHUY REALTY filed a complaint for legal redemption against
MARC REALTY and consigned in court a manager's check for 614,400. MARC REALTY insisted that the
Isabelo Garcia and his wife Tagumpay Dumaguindin purchased the homestead rights of Lingasa
complaint be dismissed for failure to state a cause of action there being no allegation of prior valid tender
Bapanialag to a parcel of homestead land situated at Mabay, Kiamba, Cotabato, containing an area of
of payment or a prior valid notice of consignation.
23.21 hectares. The transfer was approved by the Secretary of Agriculture and Natural Resources .Patent
No. V-532 was thus issued in favor of the appellant Isabelo Garcia and his wife.
On Dec 26, 1990, the trial court ruled in favour of Lee Chuy Realty which stated that there was a valid
tender of payment and consignation. It also stated that neither a separate offer to redeem nor a formal
Three years and three months after the issuance of the homestead patent, or on 14 April 1950, the spouses notice of consignation is necessary for the reason that the filing of the action itself, within the period of
sold to Domingo Colorado, Raymundo de Guzman, Inocencio Padama, Ignacio Ramos and Leon de Guzman redemption, is equivalent to a formal offer to redeem. On 1 February 1991 MARC REALTY filed a Petition
19 hectares of the homestead land. The new owners took possession of the property. The deed of sale for Certiorari, Prohibition with Temporary Restraining Order and/or Writ of Preliminary Injunction which
was, however, not submitted to the Secretary of Agriculture and Natural Resources for approval nor was referred to the Court of Appeals. The CA reversed the decision of the lower court and ruled that "a
presented to the Registrar of Deeds in and for the province of Cotabato for registration. Alienation within prior tender or offer of redemption is a prerequisite or precondition to the filing of an action for legal
5 years from the issuance of the patent is prohibited. Since the sale was made within the prohibitive period redemption” and that "there must be tender of the redemption price within the required period because
of 5 years, the sale is void and is a cause for reversion of the homestead to the state. The spouses and the policy of the law is not to leave the purchaser's title in uncertainty beyond the established 30-day
buyers contended that it was intended only as a mortgage on the improvements and crops on the 19 period.
hectares and was drawn up as a deed of sale by mistake. They also claimed that since the deed of sale
was not registered, there was no conveyance made of the 19hectares.The CFI ruled in favor of the state. MARC REALTY contends that prior tender of payment is a condition precedent to the filing of an action in
court in order to validly exercise the right of legal redemption. LEE CHUY REALTY however argues that the
Issue: Whether the ruled that the land should revert to the state as public domain. filing of the action itself is equivalent to a formal offer to redeem, which is a condition precedent to the
valid exercise of the right of legal redemption. Lee Chuy filed a motion for reconsideration but was denied.
Ruling:
ISSUE:
WON a formal offer to redeem accompanied with tender of payment a condition precedent to the filing of
The judgement was affirmed. The land should revert to the state. It is clear that the appelants intended an action for the valid exercise of the right of legal redemption; is the filing of the action with consignation
the mortgage to be a deed of sale. Moreover, the improvements on the land only consist of 50 coconut, 5 equivalent to a formal offer to redeem.
mango trees, 1 nangka tree, bananas and other fruit trees. It is unbelievable that the defendants would
grant a loan with such meager collateral. There was also no evidence of installment payments presented HELD:
to support the contention that it was a mortgage. It is also not necessary for the encumbrance to be No. The Court of Appeals erroneously concluded that a prior tender or offer of redemption is a prerequisite
registered ; it is enough that it be encumbered or alienated within 5 years. To hold otherwise would defeat or precondition to the filing of the action for legal redemption. To avail of the right of redemption what is
the purpose of the prohibition since nobody would have such illegal document registered. The buyers have essential is to make an offer to redeem within the prescribed period. There is no prescribed form for an
also taken possession of the land.Even if only 19 of the 23.21 hectares had been alienated, this is still offer to redeem to be properly effected. It can either be the formal tender with consignation or the filing
sufficient cause for reversion of the land to the state. of a complaint in court. What is paramount is the availment of the fixed and definite period within which
to exercise the right of legal redemption. The filing of the action itself is equivalent to a formal offer to
redeem. What constitutes a condition precedent is either a formal offer to redeem or the filing of an action
E n v i R e v i e w e r ( M I D T E R M S ) P a g e | 15

in court together with the consignation of the redemption price within the reglementary period. The hardly be gainsaid that they merely evidence a privilege granted by the State to qualified entities, and do
decision of respondent Court of Appeals is REVERSED and SET ASIDE. The decision of the Regional Trial not vest in the latter a permanent or irrevocable right to the particular concession area and the forest
Court of Malolos, Bulacan is REINSTATED. products therein. They may be validly amended, modified, replaced or rescinded by the Chief Executive
when national interests so require. Thus, they are not deemed contracts within the purview of the due
process of law clause.
Felipe Ysmael, Jr. & Co., Inc. vs The Deputy Executive Secretary, et al.
The Court expresses its concern regarding alleged irregularities in the issuance of timber license
FACTS: agreements to a number of logging concessionaires. Should the appropriate case be brought showing a
clear grave abuse of discretion on the part of concerned officials with respect to the implementation of this
In 1986, at the start of President Corazon Aquino’s administration, petitioner sent letters to the Office of public policy, the Court will not hesitate to step in. However, in this case, the Court finds no basis to issue
the President and to the Ministry of Natural Resources (MNR) seeking the reinstatement of its timber license a writ of certiorari and to grant any of the affirmative reliefs sought. Petition is dismissed.
agreement (TLA No. 87), which was cancelled in August 1983 along with nine other concessions, during
the Marcos administration. It alleged that after the its TLA was cancelled without being given the
opportunity to be heard, its logging area was re-awarded to other logging concessionaires without a formal SUNVILLE TIMBER PRODUCTS, INC. vs. HON. ALFONSO G. ABAD, COURT OF APPEALS, ISIDRO
award or license, as these entities were controlled or owned by relatives or cronies of deposed President GILBOLINGO AND ROBUSTIANO BUGTAI G.R. No. 85502 February 24, 1992 Exhaustion of
Marcos. Administrative Remedies
OCTOBER 18, 2017
The Ministry ruled that a timber license was not a contract within the due process clause of the Constitution,
but only a privilege which could be withdrawn whenever public interest or welfare so demands, and that
FACTS:
petitioner was not discriminated against in view of the fact that it was among ten concessionaires whose
The petitioner was granted a Timber License Agreement (TLA), authorizing it to cut, remove and utilize
licenses were revoked in 1983. It also emphasized the fact that there was currently a total log ban being
timber within the concession area covering 29,500 hectares of forest land in Zamboanga del Sur, for a
imposed on the subject areas. After the logging ban was lifted, petitioner appealed to the Office of the
period of ten years expiring on September 31, 1992. in 1987, herein private respondents filed a petition
President, but the petition was denied on the ground that the appeal was prematurely filed, the matter not
with the DENR for the cancellation of the TLA on the ground of serious violations of its conditions and the
having been terminated in the MNR. Hence, petitioner filed with the Supreme Court a petition for certiorari.
provisions of forestry laws and regulations. The same charges were subsequently made, also by the herein
ISSUE: private respondents, in a complaint for injunction with damages against the petitioner.

Whether public respondents acted with grave abuse of discretion amounting to lack or excess of jurisdiction The petitioner moved to dismiss this case on three grounds, to wit: 1) the court had no jurisdiction over
in refusing to overturn administrative orders issued by their predecessors. the complaint; 2) the plaintiffs had not yet exhausted administrative remedies; and 3) the injunction sought
was expressly prohibited by section 1 of PD 605. Judge Alfonso G. Abad denied the motion to dismiss and
RULING: the motion for reconsideration. The petitioner then elevated the matter to the CA, which sustained the trial
court .The CA held that the doctrine of exhaustion of administrative remedies was not without exception
The refusal of public respondents to reverse final and executory administrative orders does not constitute
and pointed to the several instances approved by this Court where it could be dispensed with. The
grave abuse of discretion amounting to lack or excess of jurisdiction. It is an established doctrine in this
respondent court found that in the case before it, the applicable exception was the urgent need for judicial
jurisdiction that the decisions and orders of administrative agencies have, upon their finality, the force and
binding effect of a final judgment within the purview of the doctrine of res judicata. These decisions and intervention. The decision also declared invalid Section 1 of PD 605, which provides:
orders are as conclusive upon the rights of the affected parties as though the same had been rendered by
a court of general jurisdiction. The rule of res judicata thus forbids the reopening of a matter once Sec. 1. No court of the Philippines shall have jurisdiction to issue any restraining order, preliminary
injunction or preliminary mandatory injunction in any case involving or growing out of the issuance,
determined by competent authority acting within their exclusive jurisdiction
approval or disapproval, revocation or suspension of, or any action whatsoever by the proper administrative
More importantly, the assailed orders of the MNR disclose public policy consideration, which effectively official or body on concessions, licenses, permits, patents, or public grants of any kind in connection with
forestall judicial interference. Public respondents, upon whose shoulders rests the task of implementing the disposition, exploitation, utilization, exploration and/or development of the natural resources of the
the policy to develop and conserve the country's natural resources, have indicated an ongoing department Philippines.
evaluation of all timber license agreements entered into, and permits or licenses issued, under the previous
dispensation. A long line of cases establish the basic rule that the courts will not interfere in matters which This was held to be an encroachment on the judicial power vested in the Supreme Court and the lower
are addressed to the sound discretion of government agencies entrusted with the regulation of activities courts by Article VIII, Section 1, of the Constitution. The respondent court cited Export Processing Zone
coming under their special technical knowledge and training. Authority v. Dulay, where several presidential decrees were declared unconstitutional for divesting the
courts of the judicial power to determine just compensation in expropriation cases.
Timber licenses, permits and license agreements are the principal instruments by which the State regulates
the utilization and disposition of forest resources to the end that public welfare is promoted. And it can ISSUES:
E n v i R e v i e w e r ( M I D T E R M S ) P a g e | 16

1. Whether the doctrine of exhaustion of administrative remedies was not correctly applied and that the (formerly the Bureau of Forest Development) the responsibility for the enforcement of the forestry laws
declaration of the unconstitutionality of Section 1 of PD 605 was improper. aid regulations here claimed to have been violated. This comprehensive conferment clearly implies at the
2. Whether the RTC is correct when it declared invalid Section 1 of PD 605. very least that the DENR should be allowed to rule in the first instance on any controversy coming under
its express powers before the courts of justice may intervene.
RULING:
1. The doctrine of exhaustion of administrative remedies calls for resort first to the appropriate The argument that the questions raised in the petition are purely legal is also not acceptable. The private
administrative authorities in the resolution of a controversy falling under their jurisdiction before the same respondents have charged, both in the administrative case before the DENR and in the civil case before
may be elevated to the courts of justice for review. Non-observance of the doctrine results in lack of a the RTC, that the petitioner has violated the terms and conditions of the TLA and the provisions of forestry
cause of action, which is one of the grounds allowed in the Rules of Court for the dismissal of the complaint. laws and regulations. The charge involves factual issues calling for the presentation of supporting
The deficiency is not jurisdictional. Failure to invoke it operates as a waiver of the objection as a ground evidence. Such evidence is best evaluated first by the administrative authorities, employing their specialized
for a motion to dismiss and the court may then proceed with the case as if the doctrine had been observed. knowledge of the agreement and the rules allegedly violated, before the courts may step in to exercise
their powers of review.
One of the reasons for the doctrine of exhaustion is the separation of powers, which enjoins upon the
Judiciary a becoming policy of non-interference with matters coming primarily (albeit not exclusively) within As for the alleged urgent necessity for judicial action and the claimed adverse impact of the case on the
the competence of the other departments. The theory is that the administrative authorities are in a better national interest, the record does not show that the petitioners have satisfactorily established these
position to resolve questions addressed to their particular expertise and that errors committed by extraordinary circumstances to justify deviation from the doctrine by exhaustion of administrative remedies
subordinates in their resolution may be rectified by their superiors if given a chance to do so. A no less and immediate resort to the courts of justice. In fact, this particular submission must fall flat against the
important consideration is that administrative decisions are usually questioned in the special civil actions petitioner’s uncontested contention that it has since 1988 stopped its operations under the TLA in
of certiorari, prohibition and mandamus, which are allowed only when there is no other plain, speedy and compliance with the order of the DENR.
adequate remedy available to the petitioner. It may be added that strict enforcement of the rule could also
relieve the courts of a considerable number of avoidable cases which otherwise would burden their heavily 2. There in no question that Civil Case No. 2732 comes within the jurisdiction of the respondent court.
loaded dockets. 9 Nevertheless, as the wrong alleged in the complaint was supposedly committed as a result of the unlawful
As correctly suggested by the respondent court, however, there are a number of instances when the logging activities of the petitioner, it will be necessary first to determine whether or not the TLA and the
doctrine may be dispensed with and judicial action validly resorted to immediately. forestry laws and regulations had indeed been violated. To repeat for emphasis, determination of this
question is the primary responsibility of the Forest Management Bureau of the DENR. The application of
Among these exceptional cases are: the expertise of the administrative agency in the resolution of the issue raised is a condition precedent for
1) when the question raised is purely legal; the eventual examination, if still necessary, of the same question by a court of justice. In view of the
2) when the administrative body is in estoppel; above observations, we find that there was no need for the respondent court to declare the
3) when the act complained of is patently illegal; unconstitutionality of Section 1 of PD 605. The rule is that a question of constitutionality must be avoided
4) when there is urgent need for judicial intervention; where the case can be decided on some other available ground, as we have done in the case before us.
5) when the claim involved is small; The resolution of this same question must await another case, where all the indispensable requisites of a
6) when irreparable damage will be suffered; judicial inquiry into a constitutional question are satisfactorily established. In such an event, it will be time
7) when there is no other plain, speedy and adequate remedy; for the Court “to make the hammer fall, and heavily,” in the words of Justice Laurel, if such action is
8) when strong public interest is involved; warranted. The petition is GRANTED.
9) when the subject of the controversy is private land; and
10) in quo warranto proceedings.
LAGUA VS CUSI, 160 SCRA 69 (1983)
The private respondents now submit that their complaint comes under the exceptions because forestry
laws do not require observance of the doctrine as a condition precedent to judicial action; the question FACTS: A memorandum was issued preventing the passage of Plaintiff Laguas' hauling trucks loaded with
they are raising is purely legal; application of the doctrine will cause great and irreparable damage; and logs for the Japanese vessel on the national highway loading towards where the vessel was berthed. In
public interest is involved. compliance with this directive, the security force of Defendant Eastcoast closed the road to the use by
plaintiffs trucks and other equipments and effectively prevented their passage thereof while the vehicles
We rule for the petitioner. Even if it be assumed that the forestry laws do not expressly require prior resort and trucks of other people were curiously not disturbed and were allowed passess on the same road. It
to administrative remedies, the reasons for the doctrine above given, if nothing else, would suffice to still resulted that the loading of logs on the M/S "Kyofuku Maru" was discontinued. When Plaintiffs Laguas
require its observance. Even if such reasons were disregarded, there would still be the explicit language of were already resuming the hauling operations of their logs towards the Japanese Vessel, again that same
pertinent laws vesting in the DENR the power and function “to regulate the development, disposition, road, only the day before ordered by the BFD to be opened for use and passage by plaintiffs, was closed
extraction, exploration and use of the country’s forests” and “to exercise exclusive jurisdiction” in the to them by Defendant Eastcoast's security men upon a radio message order of Defendant Maglana. Even
“management and disposition of all lands of the public domain,” and in the Forest Management Bureau
E n v i R e v i e w e r ( M I D T E R M S ) P a g e | 17

the vessel M/S "Kyofuku Maruwas" ordered by Defendant Maglana to untie her anchor contrary to existing military reservation that had been turned over to the Philippine government in 1965, was declared
laws, rules and regulations of the Bureau of Customs and the Philippine Coastguard. Given no recourse in disposable and alienable only in 1971. Hence there was non-compliance with the 30 year requirement.
the face of the blatant and illegal closure of the road in defiance of BFD orders to the contrary by the
Defendant East coast through the order of Defendant Maglana, Plaintiff Laguas had to depart posthaste to
Mati, Davao Oriental, from Baganga where the shipment and the road closure were made, to seek the SERAFIN B. YNGSON, vs. THE HON. SECRETARY OF AGRICULTURE and NATURAL RESOURCES,
assistance of the PC thereat. The private respondents filed a motion to dismiss arguing that petitioner ANITA V. DE GONZALES and JOSE M. LOPEZ
Daylinda Laguas has no capacity to sue as her name was not registered as an "agent" or "dealer" of logs
in the Bureau of Forestry. The Court agrees with the defendants that under the law, the Bureau of Forest FACTS:
Development has the exclusive power to regulate the use of logging road and to determine whether their
use is in violation of laws. The subject matter of the case at bar are the same mangrove swamps with an area of about 66 hectares,
more or less, situated in sitio Urbaso, barrio Mabini, municipality of Escalante, province of the Negros
ISSUE: Whether or not the Bureau of Forest Development has the jurisdiction to determine first the Occidental. On March 19, 1952, petitioner-appellant Serafin B. Yngson filed a similar application for
legality of closure of logging roads before the case be directed to the regular courts fishpond permit with the Bureau of Fisheries followed by those of the respondents-appellees, Anita de
Gonzales and Jose M. Lopez, who filed their respective applications with the same bureau on March 19 and
HELD: NO. P.D. No. 705 upon which the respondent court based its order does not vest any power in April 24, 1953.
the Bureau of Forest Development to determine whether or not the closure of a logging road is legal or On March 19, 1952, petitioner-appellant Serafin B. Yngson filed a similar application for fishpond permit
illegal and to make such determination a pre-requisite before an action for damages may be maintained. with the Bureau of Fisheries followed by those of the respondents-appellees, Anita de Gonzales and Jose
Moreover, the complaint instituted by the petitioners is clearly for damages based on the alleged illegal M. Lopez, who filed their respective applications with the same bureau on March 19 and April 24, 1953.
closure of the logging road. Whether or not such closure was illegal is a matter to be established on the
part of the petitioners and a matter to be disproved by the private respondents. This should appropriately When the applications were filed by the aforesaid parties in the instant case, said area was not yet available
be threshed out in a judicial proceeding. It is beyond the power and authority of the Bureau of Forest for fishpond purposes and the same was only released for said purpose on January 14, 1954. The
Development to determine the unlawful closure of a passage way, much less award or deny the payment conflicting claims of the aforesaid parties were brought to the attention of the Director of the Bureau of
of damages based on such closure. Not every activity inside a forest area is subject to the jurisdiction of Fisheries who issued an order on April 10, 1954 awarding the whole area in favor of the petitioner-appellant
the Bureau of Forest Development. and rejecting the claims of the respondents-appellees. Appellants Anita V. de Gonzales and Jose M. Lopez
appealed the order where DANR set aside the order of the Director of the Bureau of Fisheries and caused
the division of the area in question into three portions giving each party an area of one-third (1/3) of the
Spouses GEMINIANO and AMPARO DE OCAMPO and Spouses PEDRO and CRISANTA whole area covered by their respective applications.
SANTOS, vs. FEDERICO ARLOS, MARY ARLOS, TEOFILO OJERIO and BELLA OJERIO
Facts: Not satisfied with one-third of the 66 hectares, Mr. Yngson filed a petition for certiorari with the Court of
Respondent Federico S. Arlos and Teofilo D. Ojerio filed an application for registration wherein they seek First Instance. CFI dismissed. Hence this Petition.
judicial confirmation of their titles to parcels of land located at Cabcaben, Mariveles, Bataan. Petitioner
ISSUE:
Spouses Geminiano de Ocampo and Amparo De Ocampo opposed the application for registration, alleging
that they are the co-owners of the lot in question as evidenced by evidenced by Transfer Certificate of Title W/N the administrative agencies having jurisdiction over leases of public lands for development of
Nos. T-43298 and T-44205, and that they became owners of said lots by purchase from the government fishponds gravely abuse their discretion
through sales patents. The Regional Trial Court Ruled in favor of Respondents.
HELD:
Issue:
Whether or not respondent Arlos complied with the Public Land Act requiring possession of the land subject The mangrove swampland was released and made available for fishpond purposes only on January 14,
to judicial confirmation for a period of 30 years. 1954. It is clear, therefore, that all five applications were filed prematurely. There was no land available
for lease permits and conversion into fishponds at the time all five applicants filed their applications.
Held: It is elementary in the law governing the disposition of lands of the public domain that until timber or forest
No. A title may be judicially confirmed under Section 48 of the Public Land Act only if it pertains to alienable lands are released as disposable and alienable neither the Bureau of Lands nor the Bureau of Fisheries has
lands of the public domain. Unless such assets are reclassified and considered disposable and alienable, authority to lease, grant, sell, or otherwise dispose of these lands for homesteads, sales patents, leases
occupation thereof in the concept of owner, no matter how long cannot ripen into ownership and be for grazing or other purposes, fishpond leases, and other modes of utilization.
registered as a title. Verily, Presidential Decree No. 107314 clarified Section 48 (b) of the Public Land Act
by specifically declaring that the latter applied only to alienable and disposable lands of the public The Bureau of Fisheries has no jurisdiction to administer and dispose of swamplands or mangrove lands
domain.15cräläwvirtualibräry In the present case, the disputed land which was formerly a part of a US forming part of the public domain while such lands are still classified as forest land or timberland and not
released for fishery or other purposes.
E n v i R e v i e w e r ( M I D T E R M S ) P a g e | 18

DIRECTOR OF FORESTRY, FOREST STATION WARDEN, DISTRICT 13, BUREAU OF FORESTRY, Fulgencio S. Factoran, Jr., then Secretary of the Department of Environment and Natural Resources
BOARD OF DIRECTORS, NATIONAL WATERWORKS AND SEWERAGE AUTHORITY 1 and CHIEF (DENR), continued approval of the Timber License Agreements (TLAs) to numerous commercial logging
OF STAFF, ARMED FORCES OF THE PHILIPPINES vs. HON. EMMANUEL M. MUÑOZ, as Judge of companies to cut and deforest the remaining forests of the country. Petitioners request the defendant, his
the Court of First Instance of Bulacan, Branch I, the SHERIFF OF THE PROVINCE of BULACAN, agents, representatives and other persons acting in his behalf to:
and PINAGCAMALIGAN INDO-AGRO DEVELOPMENT CORPORATION, INC.
Cancel all existing timber license agreements in the country;
Facts: Cease and desist from receiving, accepting, processing, renewing or approving new timber license
agreements.
Piadeco claims to be the owner of Some 72,000 hectares of land 3 located in the municipalities of Angat,
Norzagaray and San Jose del Monte, province of Bulacan, and in Antipolo and Montalban, province of Rizal. Plaintiffs further assert that the adverse and detrimental consequences of continued and deforestation are
Piadeco's evidence of ownership consists of Titulo de Propiedad No. 4136, dated April 25, 1894,4 and a so capable of unquestionable demonstration that the same may be submitted as a matter of judicial notice.
deed of absolute sale of July 12, 1962, in its favor. Piadeco applied for registration as private woodland This act of defendant constitutes a misappropriation and/or impairment of the natural resource property
some 10,000 hectares of this land. The Bureau of Forestry, on December 4, 1963, issued in Piadeco's name he holds in trust for the benefit of plaintiff minors and succeeding generations. Plaintiff have exhausted all
Certificate of Private Woodland Registration No. PWR 2065-New, covering but a portion of the land an administrative remedies with the defendant’s office. On March 2, 1990, plaintiffs served upon defendant a
aggregate area of 4,400 hectares and an average stand of 87.20 cubic meters, situated in the municipalities final demand to cancel all logging permits in the country. Defendant, however, fails and refuses to cancel
of Angat, Norzagaray, and San Jose del Monte, all of the province of Bulacan, and Montalban, in Rizal. It the existing TLA’s to the continuing serious damage and extreme prejudice of plaintiffs.
was to expire on December 31, 1964. By virtue of the registration certificate, Piadeco conducted logging
operations.. Issues:

The controversy in the these cases began on April 11, 1964, when Acting Director of Forestry Apolonio F.  Whether or not the petitioners have the right to bring action to the judicial power of the Court.
Rivera issued an order cancelling PWR No. 2065-New. He required Piadeco to surrender the original  Whether or not the petitioners failed to allege in their complaint a specific legal right violated by
certificate to him. Ground for this cancellation was that Piadeco had violated forestry rules and regulations the respondent Secretary for which any relief is provided by law.
for cutting trees within the Angat and Marikina Watershed Reservations, expressly excluded from the said  Whether or not petitioners’ proposition to have all the TLAs indiscriminately cancelled without the
certificate.5 requisite hearing violates the requirements of due process.

Issue: Rulings:
W/N Piadeco owns the woodland in question privately In the resolution of the case, the Court held that:
Ruling:
The petitioners have the right to bring action to the judicial power of the Court.
Titulo Propriedad No. 4136 was the high point of controversy in a land claim involving several hectares of
land. Pinagcamaligan Indo-Agro Development Corporation (PIADECO) was claiming to be the owner of
1. The case at bar is subject to judicial review by the Court. Justice Davide, Jr. precisely identified in his
some 72,000 hectares of land located in municipalities of Angat, Norzagaray, and San Jose Del Monte,
opinion the requisites for a case to be subjected for the judicial review by the Court. According to
province of Bulacan, and in Antipolo and Montalban, province of Rizal. PIADECO relied on the
him, the subject matter of the complaint is of common interest, making this civil case a class suit and
aforementioned title as incontrovertible evidence of its ownership. Justice Sanchez noted the dubious
proving the existence of an actual controversy. He strengthens this conclusion by citing in the decision
validity of the title in his opinion, stating “Private ownership of land must be proved not only through the
Section 1, Article 7 of the 1987 Constitution.
genuineness of title but also with clear identity of the land claimed xxx no definite area seems to have
2. The petitioners can file a class suit because they represent their generation as well as generations
been mentioned in the title”
yet unborn. Their personality to sue in behalf of the succeeding generations can only be based on
the concept of intergenerational responsibility insofar as the right to a balanced and healthful ecology
is concerned. Such a right, as hereinafter expounded, considers the “rhythm and harmony of nature.”
Oposa et al. v. Factoran Jr. Nature means the created world in its entirety. Such rhythm and harmony indispensably include, inter
alia, the judicious disposition, utilization, management, renewal and conservation of the country’s
Facts: forest, mineral, land, waters, fisheries, wildlife, off-shore areas and other natural resources to the
end that their exploration, development and utilization be equitably accessible to the present as well
The principal petitioners, all minors duly represented and joined by their respective parents. Impleaded as as future generations.
an additional plaintiff is the Philippine Ecological Network, Inc. (PENI), a domestic, non-stock and non- 3. Every generation has a responsibility to the next to preserve that rhythm and harmony for the full
profit corporation organized for the purpose of, inter alia, engaging in concerted action geared for the enjoyment of a balanced and healthful ecology. Put a little differently, the minors’ assertion of their
protection of our environment and natural resources. The petitioners alleged the respondent, Honorable
E n v i R e v i e w e r ( M I D T E R M S ) P a g e | 19

right to a sound environment constitutes, at the same time, the performance of their obligation to
ensure the protection of that right for the generations to come. TAN V. DIRECTOR OF FORESTRY

Facts: The Bureau of Forestry issued Notice No. 2087, advertising for public bidding a certain track of
The Court does not agree with the trial court’s conclusions that the plaintiffs failed to allege with sufficient public forest situated in Olongapo, Zambales. The said land was located within the former US Naval
definiteness a specific legal right involved or a specific legal wrong committed, and that the complaint is Reservation which was turned over by the US Government to the Philippine Government. Petitioner
replete with vague assumptions and conclusions based on unverified data. Wenceslao Tan and nine other applicants submitted their applications of their offer before the deadline.
President Carlos Garcia issued a directive to the Director of the Bureau of Forestry on June 7, 1961 stating
that the area covered by the Naval Reservation be made a forest reserve for watershed purposes, and
4. The complaint focuses on one specific fundamental legal right — the right to a balanced and healthful
prepare and submit immediately a draft of proclamation. Also, the bids received by the Bureau of Forestry
ecology which, for the first time in our nation’s constitutional history, is solemnly incorporated in the
for the issuance of timber license be rejected. Out of the ten persons who submitted the bids, the said
fundamental law. Section 16, Article II of the 1987 Constitution explicitly provides:
area was awarded to the petitioner by the Bureau of Forestry. Against this award, Ravago Commercial
Company and Jorge Lao Happick filed motions for reconsideration which were denied by the Bureau of
Sec. 16. The State shall protect and advance the right of the people to a balanced and healthful Forestry. Secretary of Agriculture and Natural Resources Benjamin Gozon issued General Memorandum
ecology in accord with the rhythm and harmony of nature. Order No. 46 on May 30, 1963 delegating the authority to the Director of Forestry to grant ordinary timber
licenses. Thereafter, Jose Feliciano, being appointed as Acting Secretary of Agriculture and Natural
This right unites with the right to health which is provided for in the preceding section of the same article: Resources and replaced Secretary Gozon, promulgate immediately upon his assumption of office General
Memorandum Order No. 60, revoking the authority delegated to Bureau of Forestry to grant ordinary timber
licenses. It was also stated under the said memorandum that the issuance of new licenses, including
Sec. 15. The State shall protect and promote the right to health of the people and instill health
amendments, shall be signed by the Secretary of Agriculture and Natural Resources. On December 19,
consciousness among them.
1963, Ordinary Timber License No. 20-64, in the name of the petitioner, was signed by the Acting Director
of Forestry Estanislao Bernal without the approval of the Secretary of Agriculture and Natural Resources.
While the right to a balanced and healthful ecology is to be found under the Declaration of Principles and It was released on January 6, 1964 by the Office of the Director of Forestry but it was not signed by the
State Policies and not under the Bill of Rights, it does not follow that it is less important than any of the Secretary of Agriculture and Natural Resources as required by the Memorandum. Ravago Commercial
civil and political rights enumerated in the latter. Such a right belongs to a different category of rights Company wrote a letter on February 12, 1964 to the Secretary stating that OTL No. 20-64 issued to the
altogether for it concerns nothing less than self-preservation and self-perpetuation — aptly and fittingly petitioner should be cancelled and revoked on the ground that the grant was irregular, anomalous and
stressed by the petitioners — the advancement of which may even be said to predate all governments and contrary to existing forestry laws, rules and regulations. On March 9, 1964, said timber license was declared
constitutions. As a matter of fact, these basic rights need not even be written in the Constitution for they void ab initio by the Secretary of Agriculture and Natural Resources. Petitioner moved for a reconsideration
are assumed to exist from the inception of humankind. but the motion was denied. Hence, this petition.

 The Court are not persuaded by the trial court’s pronouncement. Issue: Whether or not Ordinary Timber License No. 20-64 is valid.
1. The respondent Secretary did not invoke in his motion to dismiss the non-impairment clause. If
he had done so, Justice Feliciano would have acted with utmost infidelity to the Government by Held: No. Petitioner-appellant’s timber license was signed and released without authority by then Acting
providing undue and unwarranted benefits and advantages to the timber license holders because Director Estanislao Bernal and is therefore void ab initio. The release of the license on January 6, 1964
he would have forever bound the Government to strictly respect the said licenses according to gives rise to the impression that it was ante-dated to December 19, 1963 on which date the authority of
their terms and conditions regardless of changes in policy and the demands of public interest and the Director of Forestry was revoked. But, what is of greatest importance is the date of the release or
welfare. He was aware that as correctly pointed out by the petitioners, into every timber license issuance, and not the date of the signing of the license. While the petitioner-appellant’s timber license
must be read Section 20 of the Forestry Reform Code (P.D. No. 705) which provides that when might have been signed on December 19, 1953 it was released only on January 6, 1964. Before its release,
the national interest so requires, the President may amend, modify, replace or rescind any no right is acquired by the licensee.
contract, concession, permit, licenses or any other form of privilege granted herein .
2. All licenses may thus be revoked or rescinded by executive action. It is not a contract, property A timber license is an instrument by which the State regulates the utilization and disposition of forest
or a property right protested by the due process clause of the Constitution. resources to the end that public welfare is promoted. It is not a contract within the purview of the due
process clause; it is only a license or privilege, which can be validly withdrawn whenever dictated by public
Hence, the instant Petition is hereby GRANTED, and the challenged Order of respondent Judge of 18 July interest or public welfare. Thus, the Court held that the granting of license does not create irrevocable
1991 dismissing Civil Case No. 90-777 was set aside. The petitioners amend their complaint to implead as rights, neither is it property or property rights.
defendants the holders or grantees of the questioned timber license agreements.
E n v i R e v i e w e r ( M I D T E R M S ) P a g e | 20

However, the penalty imposed by the CA was amended as to that imposed on qualified theft. The law
PEOPLE V WILSON QUE treats, cutting, gathering, collecting and possessing timber or other forest products without license as an
offense as grave as and equivalent to the felony of qualified theft.
Facts: Accused-appellant Wilson Que appeals from his conviction for violation of Sec. 68 of PD 705 as
amended by EO No. 277. He claimed that he acquired the 258 pieces of tanguile lumber from a legal
source. During the trial, he presented the private timber land permits (PLTP) issued by the DENR to Enrica DENR VS. DARAMAN
Cayosa and Elpidio Sabal. The PLTP authorizes its holder to cut, gather and dispose timber from the forest
covered by the permit. He alleged that the tanguile lumber which came from the forest are covered by the *** DENR HAS JURISDICTION OVER THE CONFISCATION OF FOREST PRODUCTS AND CONVEYANCES USED IN THE
said PLTP’s of Cayosa and Sabal. Appellant argues that he cannot be convicted for violation of Sec. 68 of COMMISSION OF THE OFFENSE. ***
PD 705 because EO 277 which amended section 68 to penalize the possession of timber or other forest THE private respondents herein named Gregorio Daraman and Narciso Lucenecio are charged with violation
products without the proper legal documents did not indicate the particular documents necessary to make of Section 68 of Presidential Decree No. 705. They were accused to gather, collect and possess seventy
the possession legal. Neither did the other forest laws and regulations existing at the time of its enactment. two (72) pieces of assorted sizes of lumber. The prosecution presented Pablo Opinion who testified that
Issue: WON appellant is guilty of violation of Section 68 of PD 705 as amended by EO 277. he is an employee of the Department of Environment and Natural Resources as a Forest Ranger. He is the
one responsible in stopping Daraman who is a driver of a vehicle that contains the said assorted lumber.
Held: Yes. There are two (2) separate and distinct offenses punished under Section 68 of PD 705, to wit: When he asked the driver daraman for some papers, the latter replied that he had none because they
were not his. Pablo then confiscated the said lumber and the vehicle.
(1) Cutting, gathering, collecting and removing timber or other forest products from any forest land, or
timber from any alienable or disposable public land, or from private land without any authority; and The RTC acquitted both accused on the ground of insufficiency of evidence and ORDERED THE RETURN
(2) Possession of timber or other forest products without the legal documents required under existing OF THE DISPUTED VEHICLE. On the other hand, the DENR- CENRO of Catbalogan, Samar conducted
forest laws and regulations. administrative confiscation proceedings on the seized lumber and vehicle. The respondents failed to
present documents SHOWING THE LEGALITY OF THEIR POSSESSION AND TRANSPORTATION OF THE
In the first offense, one can raise as a defense the legality of the acts of cutting, gathering, collecting or LUMBER SEIZED. Hence, the CENRO officer recommended the final confiscation of the seized lumber and
removing timber or other forest products by presenting the authorization issued by the DENR. In the conveyance which was approved by the chief of the Legal Division of the DENR.
second offense, however, it is immaterial whether the cutting, gathering, collecting, and removal of the
forest products is legal or not. Mere possession of forest products without the proper documents The DENR Region 8 moved for the reconsideration of the assailed ecision only insofar as it ordered the
consummates the crime. Whether or not the lumber comes from a legal source is immaterial because EO return of the said vehicle to the owner. It contended that the vehicle has already been administratively
277 considers the mere possession of timber or other forest products without the proper legal documents confiscated by the DENR. The issue is WON the RTC had jurisdiction to release the confiscated vehicle.
as malum prohibitum.
The SC ruled in the negative.

The Court agreed that the RTC overstepped its jurisdiction when it ordered the return of the disputed
Taopa vs. People vehicle, because the vehicle had already become government property by virtue of the forfeiture Order
issued by DENR on January 26, 1994. The DENR secretary or his duly authorized representative, under
Facts:
Section 68-A of PD 705 as amended by EO 277, may order the confiscation and disposition of all
A certain Cuison was arrested for driving a truck containing illegally-cut lumber. Upon investigation, Cuison conveyances—by land, water or air—used in illegally cutting, gathering, removing, possessing or
pointed to petitioner Taopa as owners of the said lumber. So, Taopa was arrested for violation of Section abandoning forest products.
68 of PD 705 or the Revised Forestry Code. Upon conviction by the Court of Appeals, he then seeked
Jurisdiction is conferred by substantive law. A comparison of the provisions of the two relevant sections
acquittal before the SC contending that he was not in the truck when the lumber was seized.
of PD 705, as amended, shows that the jurisdiction of the RTC covers the confiscation of the timber or
Issue: forest products as well as the machinery, equipment, implements and tools illegally used in the area where
the timber or forest products are found; it is the DENR that has jurisdiction over the confiscation of forest
Whether or not Taopa is guilty of violating Sec. 68 of PD 705. products and, to stress, all conveyances used in the commission of the offense. Section 68 reads:

Held: “Section 68. Cutting, Gathering and/or Collecting Timber, or Other Forest Products Without License.—Any
person who shall cut, gather, collect, remove timber or other forest products from any forest land, or
YES. Based on the factual circumstances, Taopa was proven to exercise dominion over the lumber loaded timber from alienable or disposable public land, or from private land, without any authority, or possess
in the truck. In fact, it was loaded in front of Taopa’s house and that he was accompanying the truck driven timber or other forest products without the legal documents as required under existing forest laws and
by Cuison. Furthermore, the fact that Taopa ran away at the mere sight of the police is clear indication of regulations, shall be punished with the penalties imposed under Articles 309 and 310 of the Revised Penal
guilt. Code.
E n v i R e v i e w e r ( M I D T E R M S ) P a g e | 21

Section 68-A, in contrast, provides: Issue:

“SEC. 68-A. Administrative Authority of the Department Head or His Duly Authorized Representative to Whether or not Judge Paderanga erred when it issued the said writ of replevin.
Order Confiscation.—In all cases of violations of this Code or other forest laws rules and regulations, the
Department Head or his duly authorized representative, may order the confiscation of any forest products Held:
illegally cut, gathered, removed, or possessed or abandoned, and all conveyances used either by land,
The Court held in the affirmative. Judge Paderanga should have dismissed the replevin suit outright for
water or air in the commission of the offense and to dispose of the same in accordance with pertinent
three reasons. First, under the doctrine of exhaustion of administrative remedies courts cannot take
laws, regulations or policies on the matter.”
cognizance of cases pending before administrative agencies. In this case, Edma did not resort to the same.
The implementing guidelines of Section 68-A define conveyance in a manner that includes “any type or He went straight to court and filed a complaint for replevin, and damages. Second, under the doctrine of
class of vehicle, craft, whether motorized or not, used either in land, water or air, or a combination thereof primary jurisdiction, courts cannot take cognizance of cases pending before administrative agencies of
or any mode of transport used in the movement of any forest product. special competence. The DENR is the agency responsible for the enforcement of forestry laws. The
complaint for replevin itself stated that members of DENR’s Task Force Sagip Kalikasan took over the forest
Hence, the original and exclusive jurisdiction over the confiscation of “all conveyances used either by products and brought them to the DENR CENRO. This should have alerted Judge Paderanga that the DENR
land, water or air in the commission of the offense and to dispose of the same” is vested in the had custody of the forest products, that administrative proceedings may have commenced, and that the
Department of Environment and Natural Resources (DENR) secretary or a duly authorized replevin suit had to be dismissed outright. Third, the forest products are already in custodia legis and thus
representative. The DENR secretary has supervision and control over the enforcement of forestry, cannot be subject of replevin.
reforestation, parks, game and wildlife laws, rules and regulations.

The validity and legality of the Order of Forfeiture falls outside the ambit of the review of the assailed
Atok Big-Wedge Mining Company v. Hon. Intermediate Appellate Court and Tuktukan Saingan
Decision and Order. The basis for the assailed Order to release the vehicle was private respondents’
acquittal of the charge of violating Section 68. On the other hand, the forfeiture Order issued by the DENR Facts:
was based on Section 68-A, which involved a distinct and separate matter cognizable by it.
Applicant Tuktukan Saingan seeks the registration of a parcel of land in Lucnab, Hogon, Benguet. His
The release of the vehicle to private respondents would defeat the purpose and undermine the evidence shows that he acquired the land from his father-in-law, Dongail, when he married his daughter.
implementation of forestry laws. The preamble of the amendment in EO 277 underscores the urgency to At the time of acquisition, it was planted with camotes, casava, langka, gabi, coffee and avocados; He
conserve the remaining forest resources of the country for the benefit of the present and future lived on the land since his marriage up to present, he paid taxes during the Japanese occupation and even
generations. Our forest resources may be effectively conserved and protected only through the vigilant before it; And that he was never disturbed in his possession.
enforcement and implementation of our forestry laws.29 Strong paramount public policy should not be
degraded by narrow constructions of the law that frustrate its clear intent or unreasonably restrict its scope. The Bureau of Lands and Bureau of Forestry, represented by the provincial fiscal oppose the application.
The Atok Big-Wedge Mining Company also oppose it claiming that the land in question is within its mineral
claims- Sally, Evelyn, and Ethel. Atok Big-Wedge Mining Company submitted evidence showing that the
Lt. Gen. Alfonso P. Dagudag (Ret.), complainant, v. Judge Maximo G.W. Paderanga, respondent. annual assessment work of these mineral claims were maintained and that it was recorded in the mining
recorder’s office.
Facts:
The Director of Lands thru the office of the solicitor General, opposed private respondents application on
On or about January 30, 2005, the Region VII Philippine National Police Regional Maritime Group (PNPRMG) the ground that the applicant did not have the title in fee simple over the questioned land and that he had
received an information that MV General Ricarte of NMC Container Lines, Inc. was shipping container vans not exercised continuous, exclusive and notorious possession and occupation over the said land for atleast
containing illegal forest products from CDO to Cebu. The shipments were falsely declared as cassava meal 30 years immediately preceding the filing of the application.
and corn grains to avoid inspection of DENR. A team composed of representatives from PNPRMG, DENR
and the Philippine Coast Guard inspected said container vans. The crew of MV General Ricarte failed to The lower court ruled in favor of Atok-Big-Wedge Mining Company. The mining claims were recorded ahead
produce the certificate of origin forms and other pertinent transport documents covering the forest of time when the applicant declared the land for taxation purposes. And so, the evidence of the applicant
products. Gen. Dagudag alleged that, since nobody claimed the forest products within reasonable period cannot prevail over the evidence of Atok-Big-Wedge Mining Company. Moreover, if applicant was already
of time, the DENR considered them abandoned, and Richard Abella, PENRO OIC issued a seizure receipt in possession and occupation of the land, it is strange that he did not oppose its survey when the mining
to NMC Container. Then, a complaint was filed before Judge Paderanga by a certain Roger Edma who company surveyed the area.
prayed that a writ of replevin be issued ordering defendants DENR, CENRO, Gen. Dagudag, and others to
The appellate court reversed its decision on the ground that private respond established his right over the
deliver the forest products to him and that judgment be rendered ordering the defendants to pay him
parcel of land. Mere payment of annual assessment fees is only proof of compliance and not a proof of
moral damages, attorney’s fees, and litigation expenses. Judge Paderanga issued a writ of replevin ordering
actual assessment. On the other hand, the applicant constructed various improvements on the land such
Sheriff Reynaldo L. Salceda to take possession of the forest products.
as 3 residential houses, fruit trees, rice fields and other permanent improvements. Also, it has been alleged
E n v i R e v i e w e r ( M I D T E R M S ) P a g e | 22

that Atok-Big-Wedge converted its application on mineral claims in question into mining lease only in HELD:
compliance with PD1214 which makes the mining company as a mere lesse of the mining claims.
The 1987 Constitution retained the Regalian Doctrine.
Issue:
The Regalian Doctrine extends not only to land but also to all natural wealth that may be found in the
Within the land had been vested on the Petitioner bowels of the earth. The Court ruled that the phrase “management or other form of assistance” in the
1973 Constitution was deleted in the 1987 Constitution, which allows only “technical or financial
Ruling: No.
assistance”. The management or operation of mining activities by foreign contractors which is the primary
(1) the rights under the Philippine Bill of 1902 of a mining claim holder over his claim has been made feature of service contracts, was precisely the evil that the drafters of 1987 Constitution sought to
subject by the said Bill itself on the strict requirement that he actually performs work or undertakes eradicate.
improvements on the mine every year and does not merely file his affidavit of annual assessment, which
requirement was correctly identified and declared in E.O No. 141; and (2) that the same rights have been The constitutional provision allowing the Pres. to enter into FTAAs with foreign-owned corporations is an
terminated by P.D No. 1214, a police power enactment, under which non-application for mining lease exception to the rule that participation in the nation’s natural resources is reserved exclusively to Filipinos.
amounts to waiver of all rights under the Philippine Bill of 1902 and application for mining lease amounts Accordingly, such provision must be construed strictly against their enjoyment by non-Filipinos.
to waiver of the right under said Bill to apply for patent. In the light of these substantial conditions upon
The Court finds that RA 7942 is invalid insofar as said Act authorizes service contracts. Although the
the rights of a mining claim holder under the Philippine Bill of 1902, there should remain no doubt now
statute employs the phrase “financial and technical agreements” in accordance with the 1987 Constitution,
that such rights were not, in the first place, absolute or in the nature of ownership, and neither were they
it actually treats these agreements as service contracts that grant beneficial ownership to foreign
intended to be so.
contractors contrary to the fundamental law. Since FTAA between the WMCP and the Philippine
Government is a service contract, the Court hereby declares it unconstitutional.
LA BUGAL B’LAAN TRIBAL ASSOCIATION V. RAMOS

FACTS: Comilang v. Buendia

La Bugal B’laan Tribal Association, Inc. is a farmers and indigenous people’s cooperative organized under Facts:
Phil.laws representing a community actually affected by the mining activities of WMCP
Nicolas Comilang staked a mining claim known as the "Bua Fraction Mineral Claim" over a parcel of land in
On March 3, 1995, Pres. Fidel V. Ramos approved Philippine Mining Act (R.A 7942) to govern the Tuding, Benguet, He stopped the exploration but continue to live in the house built on a portion of the
exploration, development, utilization & processing of all mineral resources. R.A 7942 defines the modes of land with his wife and other relatives. In 1918, Macario Comilang and his relatives also settled on a portion
mineral agreements for mining operations, outlines the procedures for their filing and approval, of the land with an area of about one (1) hectare, for residential and agricultural purposes. Surface rights
assignment/transfer and withdrawal, and fixes their terms. Before its effectivity, however, on March over the area embraced in the original Bua Fraction Mineral Claim of Nicolas Comilang soon became the
30,1995 the Pres.entered into a Financial and Technical Assistance Agreement with WMCP covering 99,387 subject of litigation in an action to quiet title filed in RTC of Baguio by the other heirs against Macario
hectares of land in South Cotabato, Sultan Kudarat, Davao del Sur and North Cotabato. claiming that they bought the rights and interest of Nicolas Comilang in the old mining claim.

On Aug. 15, 1995, DENR Secretary Victor O. Ramos issued DENR Administrative Order (DAO) No. 95-23, The court dismissed both claims of ownership of petitioner and respondent declaring the area as a public
otherwise known as the Implementing Rules and Regulations of RA No. 7942, this was later repealed by land, but recognized the possession of Macario Comilang over 1½ hectares which was declared for taxation
DAO No. 9640. On Jan. 10, 1997, Petitioners sent a letter to the DENR Sec. demanding that DENR to stop purposes but later on levied and sold at a public auction to satisfy a money judgment obtained by spouses
the implementation of R.A. No. 7942 & DAO No. 9640. Jose Coloma and Eugenia Rumbaoa against Macario filed in the RTC of Baguio. The spouses were the
purchasers in the auction and the certificate of sale was issued in favor of them.
Petitioners prayed that RA 7942 and its IRR be declared unconstitutional on the ground that they allow
fully foreign owned corporation to explore, develop, utilize and exploit mineral resources in a manner In the meantime, an application for lode patent covering the Bua Fraction Mineral Claim was filed with the
contrary to Sec. 2, par. 4, Art. 12 of the Constitution. On January 23, 2001, WMC sold all its shares in Bureau of Mines by Marcos Comilang. Abdon Delenela and his co-heirs filed their opposition to the
WMCP to Sagittarius Mines, 60 % is owned by Filipinos while the 40 % is owned by Indophil Resources, application. Pending application, Delenela filed an action for the determination of their rights on the land
an Australian Company. in the RTC of Baguio to which the said court awarded one-half in undivided share in the mineral claim in
favor of Marcos and the other half also in undivided share in favor of Abdon Delenela and co-heirs.
ISSUE: Delenela, with the knowledge and conformity of Marcos Comilang, redeemed and bought from the Coloma
spouses, the latter's rights, title, interest and claim to the 1-1/2 hectares of land acquired under the
Whether or not RA 7942 otherwise known as Philippine Mining Act of 1995, along with its IRR is certificate of sale. A writ of possession was issued in their favor. Delenela, from acquiring possession of
unconstitutional. the share of Comilang from the Coloma spouses, filed a petition for ejectment against Comilang.
E n v i R e v i e w e r ( M I D T E R M S ) P a g e | 23

Issue: Whether or not the issuance of a mineral lode patent over the Bua Mineral Claim by the Director Petitioner’s arguments have no merit. The mere filing of the expropriation proceeding means that the
of Mines (Patent issued on November 7, 1966) whereby full ownership not only of the minerals therein but property of the petitioner is no longer part of the public domain. The eminent domain is the power of the
also of the surface ground have been conveyed to the patentee government to take private property for public use. In the present case, the land where mineral claims
were located are needed for the construction of site for PMA.the fact that the location of a mining claim
Held: NO. Interest acquired under like certificates of sale alone has been described as more than a lien has been perfected doesn’t bar the government to exercise its power of eminent domain. The right of
on the property, more than an equitable estate, an inchoate legal title to the property. There is no room eminent domain of the government covers all forms of private property, whether tangible or intangible,
for doubt, therefore, that the right to possess or own the surface ground is separate and distinct from the and includes the rights which are attached to these lands.
mineral rights over the same land. And when the application for lode patent to the mineral claim was
prosecuted in the Bureau of Mines, the said application could not have legally included the surface ground
sold to another in the execution sale. Consequently, We have to declare that the patent procured
thereunder, at least with respect to the 1-½ hectares sold in execution pertains only to the mineral right SOUTHEAST MINDANAO GOLD MINING CORPORATION, petitioner, vs. BALITE PORTAL
and does not include the surface ground of the land in question. However, since Delenela acquired the MINING COOPERATIVE and others similarly situated; and THE HONORABLE ANTONIO
interest of Comilang from the Coloma spouses, she then owned both the surface rights and mineral rights CERILLES, in his capacity as Secretary of the Department of Environment and Natural
over the subject property. Resources (DENR), PROVINCIAL MINING REGULATORY BOARD OF DAVAO (PMRB-Davao),
respondents.

BENGUET CONSOLIDATED INCORPORATED vs REPUBLIC The instant case involves a rich tract of mineral land situated in the Agusan-Davao-Surigao Forest Reserve
known as the "Diwalwal Gold Rush Area." Marcopper Mining Corporation (Marcopper) was granted
FACTS Exploration Permit No. 133 (EP No. 133) which included the hotly-contested Diwalwal area.

In the year 1958, respondent Republic of the Philippines filed a complaint for expropriation against 10 Congress enacted Republic Act No. 7076, or the People's Small-Scale Mining Act. The law established a
defendants and among them is petitioner Benguet. Respondent Republic contended that they needed the People's Small-Scale Mining Program to be implemented by the Secretary of the DENR and created the
property for establishing and maintaining a site for the Philippine Military Academy and that it had occupied Provincial Mining Regulatory Board (PMRB) The statute also authorized the PMRB to declare and set aside
since 1950 these areas covered by mining claims and that they have installed permanent buildings and small-scale mining areas subject to review by the DENR Secretary and award mining contracts to small-
improvements with no less than 3 million in the belief that these areas are part of the public domain. scale miners under certain conditions. DENR Secretary Fulgencio S. Factoran issued Department
Petitioner Benguet filed their motion to dismiss stating that they have 4 mining claims which were located Administrative Order (DAO) No. 66, declaring 729 hectares of the Diwalwal area as non-forest land open
and perfected on 1933. They contended that respondent Republic has not occupied the areas covered to small-scale mining. Subsequently, a petition for the cancellation of EP No. 133 and the admission of a
neither have made any improvements therein, that expropriation proceeding refers only to privately owned Mineral Production Sharing Arrangement (MPSA) proposal over Diwalwal was filed before the DENR
mineral lands, mining interest and other private interests of private individuals, thus the expropriation of Regional Executive Director, docketed as RED Mines Case. While the RED Mines case was pending,
Benguet’s mineral claim is in violation of law. Marcopper assigned its EP No. 133 to petitioner Southeast Mindanao Gold Mining Corporation (SEM), 8 which
in turn applied for an integrated MPSA over the land covered by the permit.
CFI heard the motion to dismiss however, further proceedings were held in abeyance pending the result
of the ongoing negotiation between the parties for possible amicable settlement. However, the CFI in 1973 The DENR Secretary issued Memorandum Order No. 97-0310 which provided that the DENR shall study
promulgated its decision awarding various sums to defendants excluding petitioner Benguet. thoroughly and exhaustively the option of direct state utilization of the mineral resources in the Diwalwal
Gold-Rush Area. Such study shall include, but shall not be limited to, studying and weighing the feasibility
Benguet filed Motion to clarify the decision and a motion for new trial. Acting on these motions, CFI ordered, of entering into management agreements or operating agreements, or both, with the appropriate
fixing just compensation of the surface of 4 mining claims with interests, in the total amount of government instrumentalities or private entities, or both, in carrying out the declared policy of rationalizing
P128,051.82. Both Benguet and Reublic appealed the decision of the CFI. IAC reversed and set aside the the mining operations in the Diwalwal Gold Rush Area
decision of the court a quo awarding Benguet the amount of P7,532.46. Benguet filed the present petition
for review. Petitioner filed a special civil action before the Court of Appeals against PMRB-Davao, the DENR Secretary
and BCPMC, It prayed for the nullification of the above-quoted Memorandum on the ground that the "direct
ISSUE state utilization" espoused therein would effectively impair its vested rights under EP No. 133. The Court
WON the expropriation of petitioner Benguet’s mineral claims is in violation of law considering that the of Appeals ruled that the DENR Secretary did not abuse his discretion in issuing Memorandum Order No.
claims were already located and perfected since 1933, thus segregated from public domain. 97-03 since the same was merely a directive to conduct studies on the various options available to the
government for solving the Diwalwal conflict.
HELD
ISSUE: W/N the Court of Appeals erred in ruling that the assailed memorandum order did not
adopt the ‘direct state utilization scheme’ in resolving the Diwalwal dispute.
E n v i R e v i e w e r ( M I D T E R M S ) P a g e | 24

Miners Association of the PH vs. Factoran


The State may pursue the constitutional policy of full control and supervision of the exploration,
development and utilization of the country’s natural mineral resources, by either directly undertaking the Facts:
same or by entering into agreements with qualified entities. The DENR Secretary acted within his authority
The controversy occurred when the 1987 constitution was implemented, specifically the change introduced
when he ordered a study of the first option, which may be undertaken consistently in accordance with the
by Article XII, Section 2 of the 1987 Constitution on the system of exploration, development and utilization
constitutional policy enunciated above. Obviously, the State may not be precluded from considering a direct
of the country's natural resources. No longer is the utilization of inalienable lands of public domain through
takeover of the mines, if it is the only plausible remedy in sight to the gnawing complexities generated by
"license, concession or lease" under the 1935 and 1973 Constitutions1 allowed under the 1987 Constitution.
the gold rush. As implied earlier, the State need be guided only by the demands of public interest in settling
On July 10, 1987, President Corazon Aquino promulgated EO 211 prescribing interim procedures on the
for this option, as well as its material and logistic feasibility.
processing and approval of applications for the exploration, development and utilization of minerals. Here,
Apex Mining Corporation vs. Southeast Mindanao Gold Mining Corp. existing mining permits, licenses and leases were permitted to continue. Licensing is also governed by PD
463, without prejudice to any alterations the congress may adapt pursuant to Sec. 2 Art XII of the
Facts: This case involves "Diwalwal Gold Rush Area", a rick traxt of miner land located at Davao Del Norte. constitution
This has been stormed by conflicts by numerous mininh claimants. On Dec 12, 1983, Apex Mining
EO279 was promulgated on July 25,1987 authorizing the DENR Secretary to negotiate and conclude joint
Corporation and Balite Commjmal Portal Mining Cooperative entered into an agreement to operate mining
venture, co-production, or production-sharing agreements for the exploration, development and utilization
in the said area. On July 1, 1985, the Bureau of Forest Development issued a prospecting permit to
of mineral resources, and prescribing the guidelines for such agreements and those agreements involving
Marcopper Mining Corporation (MMC), and later, an Exploration Permit 113 (EP 113) was given to MMC.
technical or financial assistance by foreign-owned corporations for large-scale exploration, development,
On December 1991, DENR Secretary issued A.O. No. 66 declaring 729 hectares of the areas covered by
and utilization of minerals
the Agusan-Davao-Surigao Forest Reserve as non foresr lands and can be open for mining claims.
Also, MMC assigned EP 113 to the respondent SEM, allegedly, a corporation to be 100% owned by MMC. Pursuant to this mandate, the DENR Secreatry issued AO No. 57, the "Guidelines of Mineral Production
Several oppositors address rhe assignment of rights. The Panel Arbitrators of DENR ruled that the Sharing Agreement under Executive Order No. 279."6 Under this provision all existing mining leases or
assignment was valid. The CA also ruled the same. And it also ruled that the DENR Secretary has no power agreements which were granted after the effectivity of the 1987 Constitution pursuant to Executive Order
to withdraw a forest reserve since such power is lodged to the President. Hence, the petitioner elevated No. 211, except small scale mining leases and those pertaining to sand and gravel and quarry resources
the case to the SC. During the pendency of the case, Gloria Arroyo issued Proclamation No. 297 which covering an area of twenty (20) hectares or less, shall be converted into production-sharing agreements
excluded an are located in Compostella Valley and proclaimed the dame as mineral reservations and an within one (1) year from the effectivity of these guidelines.
environmentally critical area. The DENR Secretary, acting as an alter ego of the President, issued A.O. no.
2002-18 declaring an emergency situation in Diwalwal and declaring to stop all mining operations.
This was followed by Administrative Order No. 82 the "Procedural Guidelines on the Award of Mineral
Production Sharing Agreement (MPSA) through Negotiation."7Under the AO 82, existing mining lessees
Isssue:
were required to apply for permit. The Administrative orders were contested by the Mners Association of
a. W/n the CA erred in upholding the validity of EP 113 as well as its transfer to SEM. (Yes)
the Philippines. Petitioner contends that both orders were in excess of the rulemaking power granted by
b. W/n the CA erred in declaring that DENR Sec had no authority to issue A.O. no. 66. (No)
Eos 211 and 279. Both orders violate the non-impairment of contract provision since AO No. 57 unduly
c. W/n the subsequent orders of GMA and DENR Secretary outweighs the claim of Apex and Balite over
pre-terminates existing mining agreements and automatically converts them into production-sharing
Diealwal. (Yes)
agreements and AO 87 unduly preterminates existing agreemends, causing the abandonment of mining,
quarry and sand gravel permits.
Ruling:
a. SEM had not acquire any right to Diwalwal because the transfer of EP 113 was made without a prior
approval of the DENR Secretary. Under P.D. No. 463, it explicitly requires that the transfer/assignment Issue:
must be made with prior approval of DENR Sec. Are AOs 57 and 82 valid and does it not contravene Eos 211 and 279?

b. Commomwealth Act No. 37 vests solely to the President, with the concurrence of the National Assembly, Ruling:
the power to withdraw forest reserves and convert into non forest reserves. Thus A.O. 66 is null and void. The court ruled in favor of the validity of said AOs. The power of administrative officials to promulgate
rules and regulations in the implementation of a statute is necessarily limited only to carrying into effect
c. The issue of priority right has been undertaken by the issuance of Proclamation No. 297 and DAO No. what is provided in the legislative enactment
2002-18, covering that the mining operations in Diwalwal is within the full control of executive branch and
by declaring that the area as mineral reservation and as an environmentally critical area. PD 463 is no longer the governing law. It is the old system of exploration, development and utilization of
natural resources through "license, concession or lease" which, however, has been disallowed by Article
XII, Section 2 of the 1987 Constitution and is thus deemed repealed. However provisions not inconsistent
E n v i R e v i e w e r ( M I D T E R M S ) P a g e | 25

with EO 279 shall be retained. As mandated by the 1987 Constitution, the exploration, development and ownership. Furthermore, the claim and the location is perfected not only against 3rd persons but also
utilization of natural resources shall explicitly be under the full control and supervision of the State. against the Government.

There is thus no clear showing that the DENR secretary went beyond the authority given by EO 279.

The AOs do not automatically preterminate existing lease agreements since the Constitutional provision ZAMBALES CHROMITE MINING CO. v. LEIDO
was made to apply prospectively, thus respecting mining licenses or concessions granted before the 1987
FACTS: Petitioner Zambales is a mining corporation which is claiming to be the owner and holder of 60
constitution. However, it was noted that the non-impairment provision is not absolute. The State, in the
mineral claims which it acquired through purchase in good faith and for value 43 years ago. Said claims
exercise of its police power in this regard, may not be precluded by the constitutional restriction on non-
were registered in 1934 under Act of US Congress of July 1, 1902 or the Philippine Bill of 1902.
impairment of contract from altering, modifying and amending the mining leases or agreements granted.
The AOs do not also seek to convert mining lease agreements to production sharing agreements since On June 14, 1977, petitioner actually and duly filed its application for patent for each claim of said 60
production sharing agreements cant be declared/ordered unilaterally by the government; Production mineral claims which was approved and granted the rights under PD 463. Said claims became part of
sharing agreement requires a meeting of the minds after subsequent negotiation. petitioner’s private property following the doctrinal rules laid down in the cases of McDaniel and Gold Creek
Mining which already been segregated from public domain to which petitioner is entitled to the exclusive
possession and enjoyment against everyone. The issuance of PD 1214 which open to lease subsisting and
McDaniel v. Apacible and Cuisia valid patentable mining claims, lode or placer located under the provisions of the Act of US Congress of
FACTS July 1, 1902, as amended, already segregated from the public domain and held by it for over 43 yrs and
requiring it without fail and against their will to file a mining lease application with the Mines Regional
On June 17, 1916, in accordance with the Phil. Act of 1902, McDaniel located 3 petroleum placer mineral Office concerned within a period of 1 year is a deprivation of petitioner’s right to ownership without due
claims in Tayabas. Notices of their location were recorded in the office of the mining recorder. Furthermore, process of law nor just compensation and therefore, unconstitutional.
there was continuous possession and annual assessment work over the said claims.
Act No. 2932 which was approved on August 31, 1920 provided that “all public lands containing petroleum ISSUE: WoN the issuance of PD 1214 deprived the petitioner’s right.
or other mineral oils and gas, on which no patent, at the date this Act takes effect, has been issued, are
HELD: NEGATIVE. The court upheld the validity of PD 1214 for it is in accord with sec 8, Art 14 of the
hereby withdrawn from sale and are declared to be free and open to exploration, location and lease…” On
1973 Constitution and sec. 2, Art 12 of the 1987 Constitution. The Court also considered it as a valid
June 18, 1921, in accordance with Act No. 2932, Cuisia applied with the Secretary of Agriculture and
exercise of the sovereign power of the State over lands of public domain and the Decree does not cover
Natural Resources for a lease of a parcel of petroleum land that included McDaniel’s 3 claims.
all mining claims located under the Phil. Bill of 1902, but only those claims over which their locators failed
to obtain a patent.
However, Actd No. 2932 further provided that “parties having heretofore filed claims for any mineral lands
containing said minerals, shall be given preference to lease their respective claims, provided they file a Likewise, there is no showing that the petitioner has complied with all the terms and conditions prescribed
petition to that effect within 6 months from the date of the approval of this Act.” Therefore, all parties by law prior to Nov. 15, 1935 that there should be not only a valid and subsisting location of mineral land
having mineral claims prior to the approval of Act No. 2932 had until Feb. 28, 1921 to file a petition with but a continuous compliance with all the requirements of law such as the performance of annual
the Government to lease the corresponding public lands. Otherwise, their preference over other applicants assessment works and payment of real estate taxes.
shall be forfeited. McDaniel sought to prohibit the Government from granting Cuisia’s lease application
mainly on the argument that Act No. 2932 is unconstitutional since it deprives him of his property without
due process of law. MEDRANA vs. OFFICE OF THE PRESIDENT
ISSUE The controversy, which began in 1979, relates to Mining Lease Contract ("MLC") No. V-754 which the then
Whether or not the non issuance of patent reverts back the valid appropriation of public mineral land into Secretary of Agriculture and Natural Resources issued to private respondent Supreme Aggregates
property of public domain. Corporation ("Supreme Aggregates") on 30 June 1969. In 1976, within the period prescribed in Section
100 above, Supreme Aggregates filed with the Bureau of Mines an Application to Avail of Rights and
HELD Privileges under P.D. No. 463 which application was required by Sections 100 and 101, above, from all
The Supreme Court sustained McDaniel’s argument. It held that a perfected, valid appropriation of public claim owners and lessees desirous of maintaining their pre-existing rights under the regime inaugurated
mineral lands operates as a withdrawal of the tract from the body of the public domain, and so long as by that decree. On February 1979, the Director of the Bureau of Mines issued an order denying Supreme
such appropriation remains valid and subsisting, the land covered thereby is deemed private property. Aggregates’ Application for the same had failed to submit Affidavits of Annual Work Obligations. On June
Such perfected, valid and subsisting appropriation shall be deemed to have taken place when all the 1979, the Director of Mines issued Quarry Temporary Permits ("QTPs") to petitioner Teodoro Medrana.
requirements of the law in making the location of the mineral placer claims have been complied with and These permits covered areas within the territory leased to Supreme Aggregates. Supreme Aggregates filed
the claims were never abandoned or forfeited. This is notwithstanding the fact that no patent has been a petition with the Director of Mines praying for reinstatement of its rejected Application to Avail of Rights
issued since the right to a patent vests full equitable title with all the benefits, immunities, and burdens of and Privileges and for cancellation of Medrana’s QTPs. Medrana filed an answer to this petition. After
E n v i R e v i e w e r ( M I D T E R M S ) P a g e | 26

investigation, the Director of Mines ordered the reinstatement of Supreme Aggregates’ Application to Avail APEX MINING CO. INC. VS HONORABLE CANCIO GARCIA
of Rights and Privileges and the cancellation of Medrana’s QTPs since these covered areas within Supreme
Aggregates’ valid and subsisting MLC No. V-754. On appeal by Medrana, the Ministry of Natural Resources FACTS: The controversy in this case involves conflicting mining claims between herein petitioners Apex
reinstated Medrana’s QTPs and declared that Supreme Aggregates’ MLC No. V-754 had lapsed. In so Mining Co., Inc., et al. (Apex for short) and private respondent Marcopper Mining Corporation (MARCOPPER
deciding, former Minister of Natural Resources Teodoro Q. Peña reasoned that the order of the Bureau of for short). The disputed area is inside a timberland area located at Moncayo, Davao del Norte and Cateel,
Mines which had rejected Supreme Aggregates’ Application to Avail of Rights and Privileges had already Davao Oriental, consisting of 4,941.0 hectares.
become final when Supreme Aggregates filed its petition for reinstatement of its Application on 29 June
1979, that is, one hundred and twenty (120) days after its receipt of the Bureau of Mines’ order of denial. MARCOPPER was one of the first mining claimants in the disputed area, having registered its 16 claims on
Supreme Aggregates then filed an appeal with the Office of the President. The Office of the President, as January 19 and 20, 1984 through the filing of declarations of location pursuant to Presidential Decree No.
already noted in a decision dated 20 September 1988, reversed the decision of the Minister of Natural 463, otherwise known as the Mineral Resources Development Decree. MARCOPPER, allegedly, after
Resources and in essence held that the failure to submit Affidavits of Annual Work Obligations for two (2) registering its mining claim, learned from the central office of the Bureau of Forest Development (BFD)
consecutive years did not, by itself and standing alone, result in the automatic cancellation of MLC No. V- that the disputed area is within an existing forest reservation known as the "Agusan-Davao-Surigao Forest
754. Reserve" established by Proclamation No. 369 on February 27, 1931 by then Governor General Dwight F.
Davis, and realizing the invalidity of its mining claims for having availed of a wrong procedure, abandoned
Hence, the instant petition for certiorari. its 16 mining claims and applied for a prospecting permit instead with the BFD on April 11, 1984. The area
covered by its application consisted of 4,941.0 hectares overlapping its abandoned mining claims. On July
ISSUE: Whether or not the Executive Secretary (Office of the President) committed grave abuse of 1, 1985, the BFD issued to it a Permit to Prospect No. 755-123185. Discovering strong evidence of mineral
discretion. deposits in the area, it applied for a permit to explore with the then Bureau of Mines and Geo-Sciences
(BMGS). On March 10, 1986, it was issued Permit to Explore No. 133. However, upon verification from the
HELD: No. records of the BMGS, Davao City Mineral District Office, it found that the area covered by its Permit to
Careful reading of the Section 27 of the said law shows that abandonment of a mining claim or lease results Explore No. 133 is also the subject of several claims/declarations of APEX. Thus, on August 11, 1986,
from failure to comply with the annual work obligations on the area covered by a mining claim or lease for MARCOPPER filed with the BMGS a "Petition for Cancellation of Mining Claims and/or Small Scale Mining
two (2) consecutive years. The precipitating event of the lapse of a mining claim or lease contemplated in Permits" against APEX, alleging, among others and in substance, that the area covered by its Permit to
Section 27 is the failure to carry out actual work on a mining claim or lease, and not simply the failure Explore No. 133 and the declarations of locations/mining claims belonging to APEX are within an established
to submit in a timely manner the Affidavit of Annual Work Obligations. That Affidavit constitutes and existing forest reservation (Agusan-Davao-Surigao Forest Reserve) under Proclamation No. 369, dated
simply proof of compliance with the annual work obligations. Execution and submission of an Affidavit of February 27, 1931, that the said mining claims/declarations of location of APEX are invalid for being
Annual Work Obligations creates a presumption that the work obligation was indeed carried out. This violative of Presidential Decree No. 463 and its implementing rules and regulations since the acquisition of
presumption is by no means a conclusive one, but is, on the contrary, merely a prima facie one since mining rights within a forest reserve is through the filing of application for a permit to prospect with the
Section 27 expressly prescribes that "if it is found upon field verification that no such work was actually BFD and not through registration of declarations of location with the BMGS.
done on the mining claims, the claim owner/lessee shall likewise lose his rights thereto notwithstanding
submission of the aforesaid documents." It follows that, conversely, failure to submit the Affidavit of Annual APEX filed a Motion to Dismiss Marcopper's petition, alleging, in substance, that their mining claims are
Work Obligations raises the presumption that no work was actually done, but that this presumption too not within any established or proclaimed forest reserve, and as such, the acquisition of mining lights thereto
can be overturned by affirmative proof — e.g., by "field verification" — that the required annual work must be undertaken through the registration of declaration of location with the BMGS and not through the
obligations had in fact been carried out on the mining claim or leased area. To hold that the mere failure filing of an application for permit to prospect with the BFD; and that the permit to prospect and permit to
to submit the Affidavits resulted in automatic abandonment of MLC No. V-754 notwithstanding the actual explore issued to MARCOPPER are inoperative and of no legal force and effect.
performance of work obligations, would not only run counter to the express language of Section 27, but
would also be to exalt form over substance. BMGS issued an order declaring permit to explore 133 null and void. MARCOPPER appealed the said order
and the DENR after due hearing rendered decision reversing the order of BMGS declaring it null and void
Abandonment may be said to result where there is concurrence of two (2) elements: the first being the and the permit to explore of MArcopper was hereby declared valid and subsisting. APEx appealed the case
intent to abandon a right or claim and the second being the external act by which that to the Office of the President through Assistant Executive Secretary for Legal Affairs Cancio Garcia which
intention is expressed and carried into effect. There must, moreover, be an actual, as was also dismissed for lack of merit. The Court give due course to the petition.
distinguished from a merely projected, relinquishment of a claim or right; otherwise the right
or claim is not vacated or waived so as to be susceptible of being appropriated by the next ISSUE: W/N the disputed area is within an established and existing forest reservation?
owner. These two (2) requirements are clearly lacking in the case at bar. The Director of Mines and public
respondent Office of the President had found that, in point of fact, private respondent Supreme Aggregates Ruling: Affirmative.
had performed its annual work obligations. Sec 8 of Act No. 2874 empowers Governor General to reclassify lands of the public domain. The then
Governor General Dwight Davis in issuing Proclamation 369 withdrew from disposition the land described
E n v i R e v i e w e r ( M I D T E R M S ) P a g e | 27

in this case to establish a forest reserve. The disputed area is within an established and exiting forest Banahaw Mining an access/right of way to its mining claims. Banahaw Mining converted its mining
reserved. claims to applications for Mineral Production Sharing Agreements (MPSA for brevity).

The disputed areas, being clearly within a forest reserve, are not open to mining location. Sections 8 and While the MPSA were pending, Banahaw Mining, on December 18, 1996, decided to sell/assign its
13 of P.D. No. 463, as amended by P.D. No. 1385, provide: rights and interests over thirty-seven (37) mining claims in favor of private respondent Base Metals Mineral
Resources Corporation (Base Metals for brevity). The transfer included mining claims held by Banahaw
Sec. 8. Prospecting, Exploration and Exploitation of Minerals in Reserved Lands.1âwphi1 Prospecting, Mining in its own right as claim owner, as well as those covered by its mining operating agreement with
exploration and exploitation of minerals in reserved lands other than mineral reservations may be CMMCI.
undertaken by the proper Government agencies. In the event that said agencies Cannot undertake the
prospecting, exploration and exploitation of mineral in reserved lands, qualified persons may be permitted Upon being informed of the development, CMMCI, as claim owner, immediately approved the
to undertake such prospecting, exploration and exploitation in accordance with the rules and regulations assignment made by Banahaw Mining in favor of private respondent Base Metals, thereby recognizing
promulgated by the Secretary. The right to exploit the minerals found therein shag be awarded by the private respondent Base Metals as the new operator of its claims. On March 10, 1997, private
President under such terms and conditions as recommended by the Director and approved by the respondent Base Metals amended Banahaw Mining’s pending MPSA applications with the Bureau of
Secretary: Provided, That the party who undertook prospecting, exploration and exploitation of said area Mines to substitute itself as applicant and to submit additional documents in support of the application.
shall be given priority. Area clearances from the DENR Regional Director and Superintendent of the Agusan Marsh and
Wildlife Sanctuary were submitted, as required. On October 7, 1997, private respondent Base Metals‟
Notwithstanding the provisions of the preceding paragraph, a special permit may be issued by the Director amended MPSA applications were published in accordance with the requirements of the Mining Act of
to the exploration permittee to extract, remove and dispose of minerals in limited quantities as verified by 1995.
the Bureau of Mines.
On November 18, 1997, petitioner PICOP filed with the Mines Geo-Sciences Bureau (MGB), Caraga
Pursuant to P.D. No. 463, as amended, one can acquire mining rights within forest reserves by initially Regional Office No. XIII an Adverse Claim and/or Opposition to private respondent Base Metals‟
applying for a permit to prospect with the Bureau of Forest and Development (BFD) and subsequently for application.After the submission of their respective position paper, the Panel Arbitrator issued an
a permit to explore with the Bureau of Mines and Geo-Sciences (BMGS). Such procedural requisites were Order disapproving private respondent Base Metals‟ MPSA on the reasons that adverse claim was filed
complied with and undertaken by MARCOPPER after it had ascertained that its mining claims were found on time, that the granting of the MPSA application on area subject of an IFMA or PTLA which is
to be within the Agusan-Davao-Surigao Forest Reserve. On the other hand, the mining claims and SSMPs covered by a Presidential Warranty, the panel believes it cannot, unless the grantee consents thereto,
of APEX being located within said forest reserve, are in violation of the law and therefore result in a failure without the grantee’s consent, the area is considered closed to mining location (sec. 19) (b) (No. 2), DAO
to validly acquire mining rights. No. 96-40) and that the mining location in forest or timberland is allowed only if such forest or
timberland is not leased by the government to a qualified person or entity and if it is leased the consent
of the lessor is necessary, in addition to the area clearance to be issued by the agency concerned
PICOP RESOURCES, INC.,petitioner, versus BASE METALS MINERAL RESOURCES before it is subjected to mining operation.
CORPORATION and THE MINES ADJUDICATIONBOARD, respondents.
Plantation is considered closed to mining locations because it is off tangent to mining. Both are extremes.
FACTS: They cannot exist at the same time. The other must necessarily stop before the other operate. Private
Central Mindanao Mining and Development Corporation (CMMCI for brevity) entered into a Mines respondent Base Metals filed a Notice of Appeal with public respondent MAB, the latter rendered
Operating Agreement (Agreement for brevity) with Banahaw Mining and Development Corporation the assailed decision setting aside the Panel Arbitrator‟s order. The Court of Appeals upheld the decision
(Banahaw Mining for brevity) whereby the latter agreed to act as Mine Operator for the exploration, of the MAB. Hence this petition.
development, and eventual commercial operation of CMMCI‟s eighteen (18) mining claims located in
Agusan del Sur. ISSUE:

Pursuant to the terms of the Agreement, Banahaw Mining filed applications for Mining Lease Whether or not the area covered by Base Metals‟ MPSA is, by law, closed to mining activities
Contracts over the mining claims with the Bureau of Mines. So that Banahaw Mining was issued Whether or not the Presidential Warranty is a contract protected by the non-impairment clause of the 1987
a Mines Temporary Permit authorizing it to extract and dispose of precious minerals found within Constitution.
its mining claims. Upon its expiration, the temporary permit was subsequently renewed thrice by the
Bureau of Mines, the last being on June 28, 1991. HELD:
Anent the first issue, the Court ruled that the area covered by Base Metals‟ MPSA is, by law, not closed to
Since a portion of Banahaw Mining‟s mining claims was located in petitioner PICOP‟s logging concession mining activities. There is no evidence in this case that the area covered by Base Metals‟ MPSA has been
in Agusan del Sur, Banahaw Mining and petitioner PICOP entered into a Memorandum of Agreement, proclaimed as watershed forest reserves.
whereby, in mutual recognition of each other‟s right to the area concerned, petitioner PICOP allowed
E n v i R e v i e w e r ( M I D T E R M S ) P a g e | 28

Even granting that the area covered by the MPSA is part of the Agusan-Davao-Surigao Forest Reserve, (1) a prospecting permit from the agency that has jurisdiction over the area, in this case, the OEA;
such does not necessarily signify that the area is absolutely closed to mining activities. Contrary to (2) an exploration permit from the BMGS;
PICOP‟s obvious misreading of our decision in Apex Mining Co., Inc. v. Garcia, supra, to the effect that (3) if the exploration reveals the presence of commercial deposit, the permitee applies before the BMGS
mineral agreements are not allowed in the forest reserve established under Proclamation 369, the Court for the exclusion of the area from the reservation;
in that case actually ruled that pursuant to PD 463 as amended by PD 1385, one can acquire mining rights (4) granting by the president of the application to exclude the area from the reservation; and
within forest reserves, such as the Agusan-Davao-Surigao Forest Reserve, by initially applying for (5) a mining agreement approved by the DENR Secretary.
a permit to prospect with the Bureau of Forest and Development and subsequently for a permit to
explore with the Bureau of Mines and Geosciences. In this case, petitioner only obtained a prospecting permit from the OEA. In its correspondence with the
petitioner, the OEA, however, advised the petitioner on two separate occasions to obtain a "prospecting
Moreover, Sec. 18 RA 7942 allows mining even in timberland or forestty subject to existing rights permit" from the BMGS. The petitioner did not apply for an exploration permit with the BMGS, nor would
and reservations. Similarly, Sec. 47 of PD 705 permits mining operations in forest lands which include the the BMGS have granted petitioner an exploration permit because when petitioner wrote to the BMGS
public forest, the permanent forest or forest reserves, and forest reservations. informing the latter of its intention to enter into an MPSA with the DENR over Block 159, the BMGS informed
the petitioner that the respondent’s claim over Block 159 had already preceded that of the petitioner. The
With regard to the second issue, the Court do not subscribe to PICOP‟s argument that the Presidential advice given by the BMGS was justified since at that time, the respondent already had a pending application
Warranty dated September 25, 1968 is a contract protected by the non-impairment clause of the 1987 for the exclusion of Block 159 from the Malangas Coal Reservation. Thereafter, the petitioner filed his MPSA
Constitution. An examination of the Presidential Warranty at once reveals that it simply reassures application, without complying with the second, third and fourth requisites under . Since it ignored the
PICOP of the government’s commitment to uphold the terms and conditions of its timber license sound advice of the OEA and the BMGS, the government agencies concerned, and stubbornly insisted on
and guarantees PICOP‟s peaceful and adequate possession and enjoyment of the areas which are the its incorrect procedure, petitioner cannot complain now that its MPSA was revoked for failure to comply
basic sources of raw materials for its wood processing complex. The warranty covers only the right with the legal requirements.
to cut, collect, and remove timber in its concession area, and does not extend to the utilization of
other resources, such as mineral resources, occurring within the concession.
BENGUET CORPORATION v HON. OSCAR LEVISTE, and HELEN DIZON-REYES
The Presidential Warranty cannot be considered a contract distinct from PTLA No. 47 and IFMA No. 35.
It is merely a collateral undertaking which cannot amplify PICOP’s rights under its timber license. Since FACTS:
timber licenses are not contracts, the non-impairment clause cannot be invoked.
Private respondent alleged that she is the claimowner of 11 mining claims all located in the province of
Zambales. On January 15, 1967, she executed a Special Power of Attorney constituting her father, Celestino
M. Dizon, as her attorney-in-fact with full powers to "transfer, assign and dispose of her 11 mining claims.
PNOC-ENERGY DEVELOPMENT CORPORATION (PNOC-EDC), Petitioner, vs.
EMILIANO G. VENERACION, JR., Respondent. Celestino M. Dizon, acting as such attorney-in-fact for private respondent and other claimowners, entered
into an Agreement, with Dizon Mine whereby the latter was granted the right to explore, develop, exploit
Facts: and operate the 57 mining claims owned by the claimowners including the 11 claims of private respondent.

Respondent Veneracion applied for a Declaration of Location (DOL) over Block 159 of the Malangas Coal Seven (7) years later, on December 17, 1974, private respondent and the other claimowners executed a
Reservation. the DENR informed the respondent that his DOL cannot be registered since Block 159 was Deed of Ratification of Assignment, confirming the assignment, transfer and conveyance unto Dizon Mines
part of the Malangas Coal Reservation, as provided under Proclamation No. 284, issued by the President and its assigns and successors of the rights to possess, occupy, explore, develop and operate all the
on 19 July 1938. With the endorsement of the Office of Energy Affairs (OEA) and the DENR Secretary, the aforesaid mining claims. Almost three (3) months after the Deed of Ratification was executed, private
respondent petitioned the Office of the President for the withdrawal of Block 159 from the coal reservation respondent revoked Special Power of Attorney.
and its conversion into a mineral reservation. The Block was excluded by the President from the reservation. However, in spite of said notice, Dizon Mines and Benguet entered into an Operations Agreement (OA)
Petitioner PNOC-ENERGY DEVELOPMENT CORPORATION applied for a mineral prospecting permit over the whereby the former transferred to the latter the possession of the 57 mining claims for the purpose of
same Block 159 but it was denied for the reason that respondent already filed an application therefor. exploring, developing and operating them for production and marketing of marketable products under the
terms and conditions specified therein.
Issue:
Whether or not the petitioner acquired a preferential right on mining rights over Block 159. Private respondent prayed that the OA be declared null and void and inoperative insofar as it covers her
Held: 11 mining claims. Benguet Corp filed a Motion to dismiss raising issue that the court has no jurisdiction.
No. Petitioner cannot claim any mining rights over Block 159 for failure to comply with the legal
requirements. ISSUE:

The law enumerates the following requirements: W/N RTC has jurisdiction in cancelling mining contracts
E n v i R e v i e w e r ( M I D T E R M S ) P a g e | 29

HELD: HELD: The court rendered judgment in favor of the Green Valley.

Negative. Presidential Decree No. 1281 vests the Bureau of Mines with jurisdictional supervision and In upholding Green Valley's prior right over the mining areas subject of conflicting claims, the Office of the
control over all holders of mining claims or applicants for and/or grantees of mining licenses, permits, President rightly relied on the provisions of Section 28(a) of Commonwealth Act No. 137 (now Section
leases and/or operators thereof. 13(a), Presidential Decree No. 463). Under this provision, and under the regulations implementing it, it is
required that the lessor shall, first, secure a prospecting permit from the BFD and second, obtain an
Sec. 7. (c) of PD 1281 provides that Bureu of Mines shall have original and exclusive jurisdiction involving exploration permit in case of discovery of minerals in the area or when there is strong proof of
cancellation and/or enforcement of mining contracts due to the refusal of the claimowner/operator to abide mineralization. The records show that the petitioner's mining claims were backed up by no prospecting
by the terms and conditions thereof. permit.
In the case at bar, it is not disputed that the subject agreement is a mining contract and private respondent, On the other hand, Breen Valley had fully complied with such requirements, for which its claims should be
in seeking a judicial declaration of its nullity, does not wish to abide by its terms and conditions. These declared superior.
elements alone bring the action within the ambit of Section 7 of P.D. 1281.
As a general rule, the findings of government agencies with respect to the construction of statutes the
The reason underlying such refusal is indeed an irrelevant matter insofar as jurisdictional competence is implementation of which has been reposed in them, are controlling on the Court. The cases of McDaniel v.
concerned, for to make jurisdiction dependent thereon would not only be "ratifying two judicial bodies Apacible,[4] Gold Creek Mining Corporation v. Rodriguez,[5] and Salacot Mining Company v. Abadilla,[6]
exercising jurisdiction over an essentially the same subject matter—a situation analogous to split relied upon by the petitioner, and where we held that the appropriation of a mineral land pursuant to a
jurisdiction which is obnoxious to the orderly administration of justice" but also clearly ignoring the object valid claim segregates it from the public domain, are not in point. The petitioner assumes that the claims
of P.D. 1281 to make the adjudication of mining cases a purely administrative matter. of other claimants recorded in 1933 and 1934 were still valid when the Southern Zambales Forest
Reservation was established in 1956. According to the Office of the President, however, the original
claimowners had failed to perform annual development work on the claims in violation of the provisions of
GREENHILLS MINING COMPANY vs OFFICE OF THE PRESIDENT, GREEN VALLEY Section 36 of the Philippine Bill of 1902. As a consequence, the area became "open to relocation... as if no
location of the same had ever been made."[7] Conversely, assuming that the government lost the property
FACTS: when the petitioner, or the original claimowners staked their claims in 1933 and 1934, it reverted to the
Mining claims were previously located and registered with the office of the Mining recorder at Iba Zambles public dominion upon abandonment thereof. Accordingly, when President Magsaysay established the
under the provision of the Phil. Bill of 1992. However, it were abandoned due to the failure to pursue their Southern Zambales Forest Reserve in 1956, the areas covered by the said abandoned claims already
claims and to perform annual assessment works. 1956- President Magsaysay issued a Proclamation no. formed part of the public domain. The petitioner cannot, moreover, claim privity of title with the owners
245 establishing the Southern Zambales Forest Reserve covering the 37,000 hectares. of the prior locations. Such prior locations had been abandoned, or at most, forfeited, and the petitioner's
own location cannot be considered a continuation thereof.
Petitioner relocated the previously abandoned mining claims inside the reservation. Petitioner filed with the WHEREFORE, the petition is DISMISSED. The temporary restraining order issued on November 12, 1986
Bureau of Forest Development (BFD) an application for prospecting permit within the reservation which is hereby lifted. No pronouncement as to costs.
was granted. Private Respondent applied with BFD for a prospecting permit also within the reservation
which was also granted. Then applied BMGS for an exploration permit over the same area covered by its
prospecting permit.It was referred to the BMGS/MLTSD and upon its verification it found that the area
REPUBLIC V. ROSEMOOR MINING AND DEVELOPMENT CORPORATION
applied for by Private Respondent were in conflict with Petitioner group of claims, but still it was granted.
Facts: After having been granted permission to prospect for marble deposits in the mountains of Biak-na-
Petitioner filed protest with the BFD and BMGS asking for the cancellations of Private Responded
Bato, San Miguel, Bulacan, and succeeded in discovering marble deposits of high quality and in commercial
prospecting and exploration permit. The Directors of BFD and BFGS issued an order amending of Private
quantities, petitioners Dr. Lourdes Pascual, Dr. Pedro De La Concha, Alejandro De La Concha, and Rufo De
Respondent prospecting permit. Private Respondent filed an appeal to the MNR which was held that since
Guzman applied with the Bureau of Mines, now Mines and Geosciences Bureau, for the issuance of the
the case involved determination of the mining rights of the parties, the investigation and resolution was
corresponding license to exploit said marble deposits and License No. 33 was issued in favor of the
within the original jurisdiction of the Bureau of Mines and Geo-Science.
petitioners. Shortly after respondent Ernesto Maceda was appointed Minister of the DENR, petitioners’
License No. 33 was cancelled by him through a letter to Rosemoor Mining and Development Corporation.
Unsatisfied, Private Respondent filed an appeal with the OFFICE OF THE PRESIDENT which rendered the
An original petition was filed because of the aforesaid cancellation and later submitted by the petitioners’
decision affirming the order of MNR and that all the mining claims in violation of Section 28(c) of CA No.137
amended petition to assail the same. The trial court ruled on September 27, 1996 that the privilege granted
are declared null and void. Private Respondent was given the preferential right to possess, exploit ,explore,
under respondents’ license had already ripened into a property right, which was protected under the due
develop and operate the areas. Petitioner filed a MR but was DENIED. Hence , this Petition.
process clause of the Constitution. The CA sustained the trial court’s decision that the grant of the quarry
license covering 330.3062 hectares was authorized by law, because the license was embraced by four
ISSUE: determination of the mining rights of the parties concerned over the disputed area,
separate applications—each for an area of 81 hectares. Hence, this petition.
E n v i R e v i e w e r ( M I D T E R M S ) P a g e | 30

Issue: The propriety of the application to the Columbio FTAA of Republic Act No. 7942 or the Philippine
Issue: Whether or not the issuance of QLP No. 33 is valid. Mining Act of 1995, particularly Section 40 thereof requiring the approval of the President for the
assignment or transfer of financial or technical assistance agreements.
Held: The license in question is dated August 3, 1982, and it was issued in the name of Rosemoor Mining
Development Corporation. The terms of the license allowed the corporation to extract and dispose of Ruling: Columbio FTAA was entered into by the Philippine Government and WMC Philippines on 22 March
marbleized limestone from a 330.3062-hectare land in San Miguel, Bulacan. The license is, however, 1995, undoubtedly before the Philippine Mining Act of 1995 took effect on 14 April 1995. It is undisputed
subject to the terms and conditions of PD 463, the governing law at the time it was granted; as well as to that said FTAA was granted in accordance with Executive Order No. 279 and Department Administrative
the rules and regulations promulgated thereunder. By the same token, Proclamation No. 2204—which Order No. 63, Series of 1991, which does not contain any similar condition on the transfer or assignment
awarded to Rosemoor the right of development, exploitation, and utilization of the mineral site—expressly of financial or technical assistance agreements. What petitioner would want the Court to espouse is the
cautioned that the grant was subject to “existing policies, laws, rules and regulations.” retroactive application of the Philippine Mining Act of 1995 to the Columbio FTAA, a valid agreement
concluded prior to the naissance of said piece of legislation. This posture of petitioner would clearly
The license was thus subject to Section 69 of PD 463 providing that: “Notwithstanding the provisions of contradict the established legal doctrine that statutes are to be construed as having only a prospective
Section 14 hereof, a quarry license shall cover an area of not more than on hundred (100) hectares in any operation unless the contrary is expressly stated or necessarily implied from the language used in the law.
one province and not more than one thousand (1,000) hectares in the entire Philippines.” The language of In the case at bar, there is an absence of either an express declaration or an implication in the Philippine
PD 463 is clear. It states that in categorical and mandatory terms that a quarry license, like that of Mining Act of 1995 that the provisions of said law shall be made to apply retroactively, therefore, any
respondents, should cover a maximum of 100 hectares in any given province. This law neither provides section of said law must be made to apply only prospectively.
any exception nor makes any reference to the number of applications for a license. Section 69 of PD 463
Be that as it may, assuming for the sake of argument that the Philippine Mining Act of 1995 may be applied
must be taken to mean exactly what it says. Clearly, the intent of the law would be brazenly circumvented
retrospectively to the Columbio FTAA, the lack of presidential approval will not be fatal as to render the
by ruling that a license may cover an area exceeding the maximum by the mere expediency of filing several
transfer illegal, especially since, as in the instant case, the alleged lack of presidential approval has been
applications. Such ruling would indirectly permit an act that is directly prohibited by the law.
remedied when petitioner appealed the matter to the Office of the President which approved the Order of
the DENR Secretary. In the La Bugal-B'Laan Tribal Association, Inc. v. Ramos case, involving the same
Like timber permits, mining exploration permits do not vest in the grantee any permanent or irrevocable
FTAA subject of the instant case: when the transferee of an FTAA is another foreign corporation, there is
right within the purview of the non-impairment of contract and due process clauses of the Constitution,
a logical application of the requirement of prior approval by the President of the Republic and notification
since the State, under its all-encompassing police power, may alter, modify or amend the same, in
to Congress in the event of assignment or transfer of an FTAA. On the other hand, when the transferee
accordance with the demands of the general welfare.
of the FTAA happens to be a Filipino corporation, the need for such safeguard is not critical.

Section 40 of the Philippine Mining Act of 1995 requiring the approval of the President with respect to
LEPANTO CONSOLIDATED MINING CO. V. WMC RESOURCES INT'L. PTY. LTD., assignment or transfer of FTAAs, if made applicable retroactively to the Columbio FTAA, would be
tantamount to an impairment of the obligations under said contract as it would effectively restrict the right
Facts: The Philippine Government and WMC Philippines, the local wholly-owned subsidiary of WMC of the parties thereto to assign or transfer their interests in the said FTAA.
Resources International Pty. Ltd. (WMC Resources) executed a Financial and Technical Assistance
Agreement, denominated as the Columbio FTAA for the purpose of large scale exploration, development,
and commercial exploration of possible mineral resources in an initial contract area of 99,387 hectares Republic v. Marcopper Mining Corporation
located in the provinces of South Cotabato, Sultan Kudarat, Davao del Sur, and North Cotabato in
accordance with Executive Order No. 279 and Department Administrative Order No. 63, Series of 1991. Facts: Respondent Marcopper Mining Corporation (MMC) was issued a temporary permit to operate a
WMC Resources subsequently divested itself of its rights and interests in the Columbio FTAA, and on 12 tailings sea disposal system from a period of October 31, 1985 to October 21, 1986. On September 20,
July 2000 executed a Sale and Purchase Agreement with petitioner Lepanto over its entire shareholdings 1986, MMC received a telegraphic order from the NPCC directing the former to "(i)mmediately cease and
in WMC Philippines, subject to the exercise of the Tampakan Companies' exercise of their right of first desist from discharging mine tailings into Calancan Bay." The directive was brought about through the
refusal to purchase the subject shares. In an Agreement, however, the Tampakan Companies sought to efforts of certain religious groups which had been protesting MMC’s tailings sea disposal system. MMC
exercise its right of first refusal. Petitioner assailed the Tampakan Companies' exercise of its right of first requested the NPCC to refrain from implementing the aforesaid directive until its adoption of an alternative
refusal. On 10 January 2001, WMC Resources and the Tampakan Companies executed another Sale and tailings disposal system
Purchase Agreement, where Sagittarius Mines, Inc. was designated assignee and corporate vehicle which
would acquire the shareholdings and undertake the Columbio FTAA activities. After due consideration and Upon appeal before the Office of the President, a restraining order was issued which required respondent-
evaluation of the financial and technical qualifications of Sagittarius Mines, Inc., the DENR Secretary appelant to undertake a cost of not less than P30,000.00 a day fo the building of artificial reefs and planting
approved the transfer of the Columbio FTAA from WMC Philippines to Sagittarius Mines, Inc. Aggrieved by of sea grass, mangroves and vegetation on the causeway of Calancan Bay under the supervision of the
the transfer of the Columbio FTAA in favor of Sagittarius Mines, Inc., petitioner filed a Petition for Review Pollution Adjudication Board and subject to such guidelines as the Board may impose.
of the Order of the DENR Secretary with the Office of the President.
In line with the directive from the Office of the President, the Calancan Bay Rehabilitation Project (CBRP)
was created, and MMC remitted the amount of P30,000.00 a day, starting from May 13, 1988 to the Ecology
E n v i R e v i e w e r ( M I D T E R M S ) P a g e | 31

Trust Fund (ETF) thereof. However, on June 30, 1991, MMC stopped discharging its tailings in the Bay, The purpose of the law is to prevent trespass on private property. The importance of the written permission
hence, it likewise ceased from making further deposits to the ETF. is also apparent from the forms prescribed by the Bureau of Mines for the declaration of location of a
mining claim which REQUIRE THE LOCATOR TO STATE THAT THE LANDOWNER HAD GRANTED WRITTEN
On February 5, 1993, the Office of the President rendered a decision affirming the cease and desist Order PERMISSION FOR THE PROSPECTING AND LOCATION OF THE MINING CLAIM if the latter is located on
issued by the PAB; Respondent-appellant argues that the cease and desist orders were issued by the PAB private property.
ex-parte, in violation of its procedural and substantive rights provided for under Section 7 (a) of P.D. No.
984 requiring a public hearing before any order or decision for the discontinuance of discharge of a sewage SMPI argues that Sec 27 is inapplicable because it never entered the private land in question for the
or industrial wastes into the water, air or land could be issued by the PAB. Furthermore, it argues that PAB purpose of prospecting because the limestone deposits were prominently visible and exposed. The Court
did not jurisdiction over the said case because RA 7942 repealed the provisions of RA 3931 as amended said that entering has to precede prospecting; prospecting necessarily precedes discovery; and a valid
by PD 984. Therefore, the Mines regional director has jurisdiction and not the PAB. discovery is essential for the location OF A MINING CLAIM.

Issue: Whether or not RA 7942 repealed RA 3931 as amended by PD 984. SMPI also averred that its constitutional and statutory rights to use and exploit mineral resources
discovered and located by it are being unduly curtailed. Court said that the prospecting, exploration,
HELD: NO. Implied repeals are not favored and the law must be interpreted as to achieve a uniform discovery, and location must be done in accordance with the law. SMPI’s right to use and exploit never
system of jurisprudence. Both laws do not contain irreconcilable conflict and in fact complementary in matured because of its omission to comply with a condition precedent. To allow SMPI its claim for surface
nature. RA 7942 did not vest adjudicative powers upon the Mines Regional Director and it only concerns rights and right of way would be to countenance illegal trespass into private property.
promoting safety measures in the practice of mining which are administrative and regulatory powers only.
Therefore, MMC was correctly adjudged before the Pollution Adjusication Board.
Pyro Copper Mining Corporation v. Mines Adjudication Board-Department of Environment and
Natural Resources, et al.
STANDARD MINERAL PRODUCTS INC VS. CA ET AL
Facts:
Petitioner SMPI claims that it is the locator of a mining claims containing limestone in Antipolo, Rizal which
were duly registered in the Office of the Mining Recorder of Rizal in April 1959. Petitioner’s application for Mineral Production Sharing Agreement (MPSA) with Mines and Geo-Science
Bureau (MGB) of the DENR, Regional Office No. 1 in San Fernando City, La Union, for the exploration,
The mining claims cover about 15 hectares of 120 hectares of land registered in nthe name of the development and commercial utilization of certain pyrite ore and other mineral deposits in a 4,360.71-
respondents Rufino Deeunhong et. al, referred hereinafter as the “Landowners.” After locating the claims, hectare land in Dasol, Pangasinan, was approved and MPSA No. 153-2000-1 was issued in its favor. Three
SMPI applied for a mining lease FROM THE Bureau of Mines. The landowners opposed the application on (3) years after, Private Respondent Montague Resources Philippine Corporation filed an application for
the ground that SMPI had entered their land and filed its mining lease application WITHOUT THEIR exploration permit with MGB covering the same land of MPSA No. 153-2000-1. Thus, petitioner filed a
PERMISSION. The Bureau of Mines held their application in abeyance pending submission of the permission verified protest/opposition (V P/O) to the said application, with the Panel of Arbitrators (POA). Prior to said
of the surface owners. filing of V P/O, MPSA No. 153-2000-1 was cancelled per DENR Memorandum Order No. 2005-03 by DENR
Secretary Michael Defensor. Petitioner moved for reconsideration, but denied. MGB issued Exploration
No agreement was reached between the parties prompting SMPI to file an action against the Landowners
Permint No. 05-001 to Private Respondent. POA, then, dismissed motu proprio the said V P/O on the
praying that it be granted surface rights for mining purposes over 15 hectares of the Landowners property
ground, among others, that it has no jurisdiction over the same. MAB, on appeal, dismissed the appeal.
and a right of way of 5 hectares. The Landowners traversed the Complaint by averring that SMPI is not
The CA, dismissed also the petition.
entitles to the relief demanded because the prospecting was accomplished without previously securing the
landowner’s WRITTEN PERMISSION as surface owners as required by Section 27 of the Mining Act. Issues:
The Trial Court finding that the mineral claims were not located in accordance with law dismissed the 1. Whether or not the V P/O was filed out of times
complaint. The Appellate Court affirmed the decision. Hence, the petition for Review on Certiorari. 2. Whether or not POA has jurisdiction to cancel exploration permits.
The issue is WON SMPI is entitled to surface rights and a right of way to a 15-hecatre portion of the Held:
landowner’s property covered by SMPI’s mining claims for mining purposes.
1. Yes. Under Dao No. 96-40, the 30-day reglementary period for filing any adverse claim, protest, or
The SC held in the negative. opposition is reckoned from the last day of the posting of the announcement for application for exploration
permit. In this case, it was on August 27, 2005. The latter date was a Saturday, hence, the next working
SMPI failed to comply with the requisite of PRIOR WRITTEN permission by the landowner before entering
day which is August 30, 2005 (August 29, 2005 is a holiday). Further, the law states that an adverse claim,
the private land in question.
protest, or opposition be accompanied by the payment of the prescribed docket fee for the same to be
Sec. 27 of the Mining Act provides that “before entering private lands, the prospector shall first apply accepted for filing. However, in this case, the docket fee was only paid on September 6, 2005, which is
in writing for written permission of the private owner, claimant, or holder thereof…” beyond the said last day of filing.
E n v i R e v i e w e r ( M I D T E R M S ) P a g e | 32

2. POA has no jurisdiction to do the same. Under Section 28 of DAO No. 96-40, it is the Director/Concerned Whether or not the facts charged in the information constitute an offense.
Regional Director who may cancel the exploration permit. The “Director” means the Director of the MGB
Central Office, while “Regional Director” means the Regional Director of any MGB Regional Office. Ruling:

The elements of the offense, theft of minerals are that; (1) the accused extracted, removed and/ or
disposed of minerals; (2) these minerals belongs to the Government or have been taken from a mining
FLORO CEMENT CORPORATION VS. GOROSPE claim or claims leased, held or owned by other persons; and (3) the accused did not possess a mining
lease or a temporary permit or any other permit to mine granted by the Secretary or the Director under
Facts: The Municipality of Lugait filed a complaint against Floro, a corporation engaged in the existing mining decrees, laws and regulations.
manufacturing and selling of cement, for the collection of taxes. The taxes sought to be collected refers to
manufacturers and exporters taxes based on Municipal Ordinance No. 5 and 10. Floro denied liability for Evidently, the information filed in the court includes all the foregoing elements. Thus, it alleged (1) that
the payment of taxes on the ground that under P.D. No. 463 x a municipality chas no power to levy taxes the accused, conspiring and mutually helping one another, willfully and feloniously extracted, removed
on mines, mining operations and mineral products, it also contended that cement is a mineral product and and/or disposed of minerals or material aggregates like sand and gravel; (2) the minerals were taken from
that the manufacturing of cement is indirectly connected with mininh operations. Sumigar Quarry, Banawe, Ifugao, which is covered by a commercial permit issued by the Bureau of Mines,
Baguio City in favor of complaining witness Felix De Castro; and (3) the extracting was done without any
Issue: W/n Floro can be validly taxed by the Mun. of Lugait mining lease or permit of their own pursuant to law.
Ruling: Yes.

Cement is not a mineral product, sinve a cement is already a product/result of a process. It had undergone CRUZ VS SECRETARY OF ENVIRONMENT & NATURAL RESOURCES
chemical changes through manufacturing process. The business of manufacturing and exporting cement
cement does nkt partake of the exploration, development nor exploitation of mineral resources thus outside FACTS:
the scope of PD 463.
Petitioners Isagani Cruz and Cesar Europa assailed the constitutionality of certain provisions of RA 8371,
otherwise known as the Indigenous Peoples Rights act of 1997 (IPRA), and its Implementing Rules and
Regulations.
People of the Philippines v. Abad

Facts: HELD:

The Director of Mines issued a commercial lease permit to Felix de Castro granting him exclusive right to Justice Puno and Justice Kapunan filed a separate opinion sustaining the challenged provisions of the law.
quarry, extract and carry away sand and gravel from the Sumigar Quarry at Banawe, Ifugao.
According to J.Puno, the provisions of the IPRA do not contravene the Constitution.
Felix de Castro filed a case charging private respondents with the crime of “Theft of Minerals”
Ancestral domains and ancestral lands are the private property of indigenous peoples and do not constitute
That on or about March 1978 up to September of same year, the accused without any permit of their own part of the land of the public domain.
pursuant to law, did then and there willfully, unlawfully and feloniously extract, gather, remove, take and/or
dispose of the minerals like sand, gravel, stones and boulders by the use of threat, force and intimidation The Congress, may by law, allow small-scale utilization of natural resources by Filipino citizens, as well as
against the permittee and his laborers to the extent of 40,592.38. cooperative fish farming, with priority to subsistence fishermen and fishworkers in rivers, lakes, bays and
lagoons. For the large-scale exploration of these resources, specifically minerals, petroleum and other
Respondents accused filed a motion to quash claiming that they had paid sand and gravel tax and therefore mineral oils, the State, through the Pres. may enter into technical and financial agreements with foreign-
it was with the consent of the government. They also invoked LOI No. 243 which allows persons to extract owned corporations.
sand and gravel even leased area if it is to be used in government infrastructure. The petitioner opposed
the quashal, arguing that the taxes paid to the Municipal Government are not the fees required by Bureau Under the Philippine Mining Act of 1995 (RA 7942) and the People’s Small-Scale Mining Act of 1991 (RA
of Mines and that LOI 243 grants the right to extract sand and gravel to be used for government projects 7076), the three types of agreements, i.e., co-production, joint venture, or production-sharing, may apply
and not on private person or entity. to both large-scale and small-scale mining. Small-scale mining refers to mining activities which rely heavily
on manual labor using simple implements and methods and do not use explosives or heavy mining
The respondent judge quashed the information on the ground that P.D No. 463 is limited to an
equipment.
administrative violation and that the crime of Theft under the RPC was not committed since malice which
is an essential element of a crime is lacking. Petitioner prayed for reconsideration but it was denied thus According to J. Kapunan, No ancestral may be declared as a people’s small-scale mining area without
this certiorari petition alleging grave abuse of discretion on the part of the respondent judge. the prior consent of the cultural communities concerned as declared by Sec. 16 of RA 7942. If Ancestral
Issue: lands are declared as people’s small-scale mining area, the members of the cultural communities living
E n v i R e v i e w e r ( M I D T E R M S ) P a g e | 33

within said area, shall be given priority in the awarding of small-scale mining contracts as expressly provide contain coals, mapping, core drilling and trenching, sinking, tunneling, extraction and utilization of coal
in Sec. 7 of RA 7076. deposits, among others.

Sec. 17 of RA 7942 states that in the event that the members of such indigenous cultural community give Based on the foregoing provisions, it’s obvious that a scrutiny of the coal operating contract of Bacaltos
their consent to mining operations within their ancestral land, royalties, shall be paid to them by the parties Coal Mines would have provided SMC knowledge of the activities which are germane, related and incident
to the mining contract. tp the power to use it. However, they did not even required Savellon to produce the same.

Lastly, the rights given to the indigenous people regarding the exploitation of natural resources under SMC was negligent as it failed to verify first if indeed petitioner Bacaltos owned a vessel. It merely satisfied
Sec.7(b) and 57 of RA 8371 or the “ Indigenous Peoples Right Act of 1997” (IPRA), amplify what has been itself with the claim of Savellon that the vessel it was leasing is owned by Bacaltos and relied on the
granted to them under existing laws, such as the Philippine Mining Act of 1995 (RA 7942) and the People’s presentation of the Authorization. The Authorization does not also state that petitioner owns any vessel
Small-Scale Mining Act of 1991 (RA 7076). and in fact, petitioner themselves declared that they have not leased any vessel, they do not need it and
they do not even own one. The Court ruled that SMC was the author of its own damage for its failure to
ascertain the extent and limits of the authority granted to Savellon. Petitioners are therefore free from an
y liability. The decision of the CA has been set aside and reversed and the decision of the court a quo was
BACALTOS COAL MINES vs CA and San Miguel Corp.
modified declaring Rene Savellon as solely liable to SMC for the amounts adjudged.
FACTS

Petitioner seek the reversal of the decision of the CA affirming the decision of the RTC holding petitioner
Bacaltos Coal Mines, German Bacaltos and co-dependant Rene Savellon jointly and severally liable to FIRST PHILIPPINE INDUSTRIAL CORPORATION vs.CA G.R. No. 125948
private respondent, San Miguel Corporation under a Trip Charter Party. German Bacaltos, the proprietor of
Bacaltos Coal Mines executed an Authorization in favor of Rene Savellon, authorizing the latter to use the Facts:
coal operating contract of Bacaltos Coal Mines for any legitimate purpose. Petitioner is a grantee of a pipeline concession under Republic Act No. 387, as amended, to contract, install
Rene Savellon, through the Authorization granted to him, entered into a contract with San Miguel Corp, and operate oil pipelines. In January 1995, petitioner applied for a mayor's permit with the Office of the
particularly, the Trip Charter Party wherein Savellon claimed that Bacaltos is the owner of the vessel M/V Mayor of Batangas. However, the respondent City Treasurer required petitioner to pay a local tax based
Premship II and that it is lending the said vessel to the private respondent as cargo carrier for 3 round on its gross receipts for the fiscal year 1993 pursuant to the Local Government Code .. The assessed tax
trips to Davao. SMC issued a check in the name of Rene Savellon with the amount of P650,000.00 as the amounted to P956,076.04 payable in four installments based on the gross receipts for the fiscal year 1993
payment they agreed upon. However, the vessel was only used by private respondent for only one trip. which amounted to P181,681,151.00. Petitioner paid the tax under protest in the amount of P239,019.01
Hence they filed a complaint against Bacaltos Coal Mines and Rene Savellon for specific performance and for the first quarter of 1993.
damages. Petitioner Bacaltos alleged they are not liable considering that the power granted to Rene Petitioner filed a letter-protest addressed to the City Treasurer claiming that it is a holder of a government
Savellon are only those clearly expressed in the Authorization which do not include any power to enter into concession under the Petroleum act and is engaged the business of transporting petroleum. As such, it is
contract with SMC. exempt from paying tax on gross receipts under Sec. 133 of the LGC of 1991. The protest was denied by
RTC rendered the assailed decision in favor of SMC ordering petitioners jointly and severally liable to SMC. the City Treasurer, who reasoned that the petitioner cant be considered engaged in the transportation
In appeal, CA affirmed in toto the decision of the trial court, hence this present petition. business. Thus a complaint and request for tax refund was filed with the RTC. It ruled that the petitioner
cant be exempted since the exemption only applies to common carriers so as not to overburden the riding
ISSUE: public. Petitioner, it contends, is only a special carrier extending its facilities to a single special customer.

WON Rene Savellon was authorized to enter into a Trip Charter Party Contract with SMC although such Issues:
authority cannot be found on the four corners of the Authorization.
Whether FPIC can be considered a common carrier and whether refund should be applied.
HELD;
Ruling:
Based on the Authorization executed by German Bacaltos in favor of Rene Savellon, the extent and scope
of his powers is exclusive only to “use the coal operating contract” of Bacaltos Coal Mines. Since the FPIC is a common carrier. Art. 1732 of the Civil Code defines a "common carrier" as "any person,
principal subject of the Authorization is the ‘coal operating contract’, SMC should have required its corporation, firm or association engaged in the business of carrying or transporting passengers or goods
presentation to determine what is it and how it may be used by Rene Savellon. The nature of coal operating or both, by land, water, or air, for compensation, offering their services to the public. The law does not
contract is found in the provisions of PD No. 972 or the Coal Development Act of 1976 specifically Section distinguish despite FPIC’s having a limited clientele. A common carrier is engaged in public service. Republic
9 thereof which provides the obligations of the contractor, like, to examine, investigate lands supposed to Act 387 also regards petroleum operation as a public utility. Everything relating to the exploration for and
exploitation of petroleum and everything relating to the manufacture, refining, storage, or transportation
by special methods of petroleum, is hereby declared to be a public utility.
E n v i R e v i e w e r ( M I D T E R M S ) P a g e | 34

It was also found that the legislative intent in excluding from the taxing power of the local government accorded an equal significance. Or its importance could be nil. In light of this uncertainty, we rule that the
unit the imposition of business tax against common carriers is to prevent a duplication of the so-called early deregulation under E.O. No. 392 constitutes a misapplication of R.A. No. 8180.
"common carrier's tax, which amounts to 3%. The refund is thus granted.
Again, we underline in scarlet that the fundamental principle espoused by section 19, Article XII of the
Constitution is competition for it alone can release the creative forces of the market. But the competition
that can unleash these creative forces is competition that is fighting yet is fair. Ideally, this kind of
FRANCISCO S. TATAD vs. THE SECRETARY OF THE DEPARTMENT OF ENERGY AND THE
competition requires the presence of not one, not just a few but several players. A market controlled by
SECRETARY OF THE DEPARTMENT OF FINANCE.
one player (monopoly) or dominated by a handful of players (oligopoly) is hardly the market where honest-
EDCEL C. LAGMAN, JOKER P. ARROYO, ENRIQUE GARCIA, WIGBERTO TAÑADA, FLAG HUMAN RIGHTS to-goodness competition will prevail. Monopolistic or oligopolistic markets deserve our careful scrutiny and
FOUNDATION, INC., FREEDOM FROM DEBT COALITION (FDC), SANLAKAS vs. HON. RUBEN TORRES in his laws which barricade the entry points of new players in the market should be viewed with suspicion.
capacity as the Executive Secretary, HON. FRANCISCO VIRAY, in his capacity as the Secretary of Energy,
CALTEX Philippines, Inc., PETRON Corporation and PILIPINAS SHELL Corporation. In the cases at bar, it cannot be denied that our downstream oil industry is operated and controlled by an
oligopoly, a foreign oligopoly at that. Petron, Shell and Caltex stand as the only major league players in
Facts: In March 1996, Congress took the audacious step of deregulating the downstream oil industry. It the oil market. All other players belong to the lilliputian league. As the dominant players, Petron, Shell and
enacted R.A. No. 8180, entitled the "Downstream Oil Industry Deregulation Act of 1996." Under the Caltex boast of existing refineries of various capacities. The tariff differential of 4% therefore works to their
deregulated environment, "any person or entity may import or purchase any quantity of crude oil and immense benefit. Yet, this is only one edge of the tariff differential. The other edge cuts and cuts deep in
petroleum products from a foreign or domestic source, lease or own and operate refineries and other the heart of their competitors. It erects a high barrier to the entry of new players. New players that intend
downstream oil facilities and market such crude oil or use the same for his own requirement," subject only to equalize the market power of Petron, Shell and Caltex by building refineries of their own will have to
to monitoring by the Department of Energy. spend billions of pesos. Those who will not build refineries but compete with them will suffer the huge
disadvantage of increasing their product cost by 4%. They will be competing on an uneven field. The
On February 8, 1997, the President implemented the full deregulation of the Downstream Oil Industry argument that the 4% tariff differential is desirable because it will induce prospective players to invest in
through E.O. No. 372. The petitions at bar assail the constitutionality of various provisions of R.A No. 8180 refineries puts the cart before the horse. The first need is to attract new players and they cannot be
and E.O. No. 372 on the grounds that E.O. No. 392 implementing the full deregulation of the downstream attracted by burdening them with heavy disincentives. Without new players belonging to the league of
oil industry is arbitrary and unreasonable because it was enacted due to the alleged depletion of the OPSF Petron, Shell and Caltex, competition in our downstream oil industry is an idle dream.
fund — a condition not found in R.A. No. 8180 and section 15 of R.A. No. 8180 and E.O. No. 392 allow
the formation of a de facto cartel among the three existing oil companies — Petron, Caltex and Shell — in The provision on inventory widens the balance of advantage of Petron, Shell and Caltex against prospective
violation of the constitutional prohibition against monopolies, combinations in restraint of trade and unfair new players. Petron, Shell and Caltex can easily comply with the inventory requirement of R.A. No. 8180
competition. Respondents, on the other hand, fervently defend the constitutionality of R.A. No. 8180 and in view of their existing storage facilities. Prospective competitors again will find compliance with this
E.O. No. 392. requirement difficult as it will entail a prohibitive cost. The construction cost of storage facilities and the
cost of inventory can thus scare prospective players. Their net effect is to further occlude the entry points
Issue; W/N RA 8180 violates the constitutional prohibition against monopolies, combination in restraint of of new players, dampen competition and enhance the control of the market by the three (3) existing oil
trade and unfair completion. companies.

Ruling: Section 19 of Article XII of the Constitution allegedly violated by the aforestated provisions of R.A.
No. 8180 mandates: "The State shall regulate or prohibit monopolies when the public interest so requires. PEREZ v. LPG REFILLERS ASSOCIATION OF THE PHILIPPINES, INC.
No combinations in restraint of trade or unfair competition shall be allowed."
FACTS
The Executive misapplied R.A. No. 8180 when it considered the depletion of the OPSF fund as a factor in Batasang Pambansa Blg. 33 as amended, penalizes illegal trading, hoarding, overpricing, adulteration,
fully deregulating the downstream oil industry in February 1997. A perusal of section 15 of R.A. No. 8180 underdelivery, and underfilling of petroleum products, as well as possession for trade of adulterated
will readily reveal that it only enumerated two factors to be considered by the Department of Energy and petroleum products and of underfilled liquefied petroleum gas (LPG) cylinders. The said law imposes a
the Office of the President, viz.: (1) the time when the prices of crude oil and petroleum products in the penalty that is monetary for violators. The respondent association asked the DOE to set aside the circular
world market are declining, and (2) the time when the exchange rate of the peso in relation to the US for being contrary to law. However, the said department denied the request for lack of merit. The
dollar is stable. Section 15 did not mention the depletion of the OPSF fund as a factor to be given weight respondent then filed before the trial court a petition and the trial court nullified the said circular on the
by the Executive before ordering full deregulation. ground that it introduced new offenses not included in the law.

the Executive co-mingled the factor of depletion of the OPSF fund with the factors of decline of the price ISSUE
of crude oil in the world market and the stability of the peso to the US dollar. On the basis of the text of Whether or not the Circular issued by DOE is valid
E.O. No. 392, it is impossible to determine the weight given by the Executive department to the depletion
of the OPSF fund. It could well be the principal consideration for the early deregulation. It could have been RULING
E n v i R e v i e w e r ( M I D T E R M S ) P a g e | 35

Yes, For an administrative regulation, such as the Circular in this case, to have the force of penal law, (1) customers in relation to the sale of energy. These charges cannot be imposed without the sale of energy.
the violation of the administrative regulation must be made a crime by the delegating statute itself; and Indeed, a consideration in fixing rates is the purpose for which the penalties are constituted: the regulation
(2) the penalty for such violation must be provided by the statute itself. of the system loads of transmission lines,51 so as to ensure the continuous operation of the public utility or
to cover part of its operating expenses.
The Circular satisfies the first requirement. B.P. Blg. 33, as amended, criminalizes illegal trading,
adulteration, underfilling, hoarding, and overpricing of petroleum products. These specific acts and The power to determine, fix and prescribe rates being charged customers is vested in the ERB. Therefore,
omissions are obviously within the contemplation of the law, which seeks to curb the pernicious practices unless it gives prior approval, the penalties cannot be imposed by the NPC. Without that authority, the
of some petroleum merchants. challenged provisions in the "Rules on the Sale of Electricity" cannot be imposed on the electric plant
operators that PEPOA represents. While petitioner may issue rules and regulations consistent with its
As for the second requirement, we find that the Circular is in accord with the law. Under B.P. Blg. 33, as corporate objectives, provisions that have a bearing on the impositions of rates must be approved by the
amended, the monetary penalty for any person who commits any of the acts Aforestated is limited to a ERB.
minimum of P20,000 and a maximum of P50,000. Under the Circular, the maximum pecuniary penalty for
retail outlets is P20,000, an amount within the range allowed by law. However, for the refillers, marketers,
and dealers, the Circular is silent as to any maximum monetary penalty. This mere silence, nonetheless,
FREEDOM FROM DEBT COALITION V ERC
does not amount to violation of theaforesaid statutory maximum limit. Further, the mere fact that the
Circular provides penalties on a per cylinder basis does not in itself run counter to the law since all that FACTS:
B.P. Blg. 33 prescribes are the minimum and the maximum limits of penalties.
On October 10, 2003, MERALCO filed with the ERC an Application for an increase in rates. National
Clearly, it is B.P. Blg. 33, as amended, which defines what constitute punishable acts involving petroleum Association of Electricity Consumers for Reforms, Inc. (NASECORE), Mr. Genaro Lualhati (Lualhati), and
products and which set the minimum and maximum limits for the corresponding penalties. The Circular petitioner Freedom from Debt Coalition (FDC) expressed their intention to file an opposition to MERALCO’s
merely implements the said law, albeit it is silent on the maximum pecuniary penalty for refillers, marketers, Application. NASECORE filed a Motion for Production of Documents to enable it to evaluate MERALCO’s
and dealers. Nothing in the Circular contravenes the law. Application however, November 27, 2003, the ERC, without first resolving the Motions for Production of
Documents of NASECORE and FDC and apparently without considering Lualhati’s Opposition, issued an
Order provisionally approving MERALCO’s ex parte application for rate increases.
NATIONAL POWER CORPORATION, Petitioner, v. PHILIPPINE ELECTRIC PLANT OWNERS FDC argues that the order of ERC is void for having been issued without legal/statutory authority and for
ASSOCIATION (PEPOA), INC., Respondent. having been issued with grave abuse of dicretion.

Facts: ISSUE:

W/N ERC has legal/ statutory authority to grant provisional rate adjustments under RA 9136/ EPIRA
PEPOA filed before the ERB a Complaint8 against the NPC for alleged unauthorized collection of rates in
the guise of penalty for 1) excess consumption, double or triple the existing rates; or 2) unused HELD:
consumption, as if fully availed of.9 The penalties were being charged pursuant to the NPC's Rules on the
Sale of Electricity, specifically Nos. 5 (Minimum Charges) and 6 (Penalty for Consumption in Excess of the YES. ERC authority is found in Secs. 44 and 80 of the EPIRA The ERC is endowed with the statutory
Allowable Limit of the Contract Demand/Energy) of the Schedule of Charges. On December 20, 1995, the authority to approve provisional rate adjustments under the aegis of Sections 44 and 80 of the EPIRA. The
ERB issued an Order12 directing the NPC to cease and desist from collecting the penalties, pending sections read, thus:
resolution of the case. On May 12, 1998, the ERB rendered its Decision declaring as final the cease and
desist order directed against petitioner. The ERB decision was appealed to the Court of Appeals but it was SEC. 44. Transfer of Powers and Functions. — The powers and functions of the Energy Regulatory Board
affirmed stating the the authorization of the ERB was required for the implementation of the penalties in not inconsistent with the provisions of this Act are hereby transferred to the ERC. The foregoing transfer
question. Hence, this petition. of powers and functions shall include all applicable funds and appropriations, records, equipment, property
and personnel as may be necessary.
Issue: What is clear from Sections 80 and 44 is that the legislators saw the superfluity or needlessness of carrying
Whether or not the imposition of the penalties is an increase in power rates that requires authorization of over in the EPIRA the same provision found in the previous laws. The power to approve provisional rate
the Energy Regulatory Board. increases is included among the powers transferred to the ERC by virtue of Section 44 since the grant of
that authority is not inconsistent with the EPIRA; rather, it is in full harmony with the thrust of the law
Held: which is to strengthen the ERC as the new regulatory body.
Yes. The penalties imposed by the NPC in its "Rules on the Sale of Electricity" are covered by the definition
of rate. "Minimum Charges" and "Penalties for Consumption in Excess of Allowable Limit" are exacted from
E n v i R e v i e w e r ( M I D T E R M S ) P a g e | 36

KAPISANAN NG MGA KAWANI NG ENERGY REGULATORY BOARD v. COMMISSIONER FE B. (1) the issuance of a formal letter related to the date of filing of job applications, including the use of Civil
BARIN, DEPUTY COMMISSIONERS CARLOS R. ALINDADA, LETICIA V. IBAY, OLIVER B. Service application form no. 212; (2) the creation of a placement/recruitment committee and setting
BUTALID, and MARY ANNE B. COLAYCO, of the ENERGY REGULATORY COMMISSION. guidelines relative to its functions, without prejudice to existing Civil Service rules and regulations; and (3)
copies of the plantilla positions and their corresponding qualification standards duly approved by either the
The Facts President of the Philippines or the Civil Service Commission (CSC).
RA 9136, popularly known as EPIRA (for Electric Power Industry Reform Act of 2001), was enacted on 8
June 2001 and took effect on 26 June 2001. Section 38 of RA 9136 provides for the abolition of the ERB Issue:
and the creation of the ERC. The pertinent portions of Section 38 read: W/N Sec. 38 of R.A. 9136 abolished ERB and established the ERC, or did it only expand the ERB

Creation of the Energy Regulatory Commission. ' There is hereby created an independent, quasi-judicial RULING:
regulatory board to be named the Energy Regulatory Commission (ERC). For this purpose, the existing After comparing the functions of the ERB and the ERC, we find that the ERC indeed assumed the functions
Energy Regulatory Board (ERB) created under Executive Order No. 172, as amended, is hereby abolished. of the ERB. However, the overlap in the functions of the ERB and of the ERC does not mean that there is
The Commission shall be composed of a Chairman and four (4) members to be appointed by the President no valid abolition of the ERB. The ERC has new and expanded functions which are intended to meet
of the Philippines. x x x the specific needs of a deregulated power industry. Indeed, National Land Titles and Deeds Registration
Administration v. Civil Service Commission stated that:
Within three (3) months from the creation of the ERC, the Chairman shall submit for the approval of the
President of the Philippines the new organizational structure and plantilla positions necessary to carry out [I]f the newly created office has substantially new, different or additional functions, duties or powers, so
the powers and functions of the ERC. The Chairman and members of the Commission shall assume office that it may be said in fact to create an office different from the one abolished, even though it embraces all
at the beginning of their terms: Provided, That, if upon the effectivity of this Act, the Commission has not or some of the duties of the old office it will be considered as an abolition of one office and the creation of
been constituted and the new staffing pattern and plantilla positions have not been approved and filled- a new or different one. The same is true if one office is abolished and its duties, for reasons of economy
up, the current Board and existing personnel of ERB shall continue to hold office. are given to an existing officer or office.13

The existing personnel of the ERB, if qualified, shall be given preference in the filling up of plantilla positions Throughout the years, the scope of the regulation has gradually narrowed from that of public services in
created in the ERC, subject to existing civil service rules and regulations. At the time of the filing of this 1902 to the electricity industry and water resources in 1972 to the electric power industry and oil industry
petition, the ERC was composed of Commissioner Fe B. Barin and Deputy Commissioners Carlos R. in 1977 to the electric industry alone in 1998. The ERC retains the ERB's traditional rate and service
Alindada, Leticia V. Ibay, Oliver B. Butalid, and Mary Anne B. Colayco (collectively, Commissioners). The regulation functions. However, the ERC now also has to promote competitive operations in the electricity
Commissioners assumed office on 15 August 2001. Pursuant to Section 38 of RA 9136, the Commissioners market. RA 9136 expanded the ERC's concerns to encompass both the consumers and the utility investors.
issued the proposed Table of Organization, Staffing Pattern, and Salary Structure on 25 September 2001 Thus, the EPIRA provides a framework for the restructuring of the industry, including the privatization of
which the President of the Philippines approved on 13 November 2001. Meanwhile, KERB submitted to the the assets of the National Power Corporation (NPC), the transition to a competitive structure, and the
Commissioners its Resolution No. 2001-02 on 13 September 2001. Resolution No. 2001-02 requested the delineation of the roles of various government agencies and the private entities. The law ordains the
Commissioners for an opportunity to be informed on the proposed plantilla positions with their equivalent division of the industry into four (4) distinct sectors, namely: generation, transmission, distribution and
qualification standards. supply. Corollarily, the NPC generating plants have to privatized and its transmission business spun off and
privatized thereafter.
On 17 October 2001, the Commissioners issued the guidelines for the selection and hiring of ERC
employees. A portion of the guidelines reflects the Commissioners' view on the selection and hiring of the In tandem with the restructuring of the industry is the establishment of "a strong and purely independent
ERC employees vis-a-vis Civil Service rules, thus: regulatory body." Thus, the law created the ERC in place of the Energy Regulatory Board (ERB). To achieve
its aforestated goal, the law has reconfigured the organization of the regulatory body. x x x 15 There is no
Since R.A. 9136 has abolished the Energy Regulatory Board (ERB), it is the view of the Commission that question in our minds that, because of the expansion of the ERC's functions and concerns, there was a
the provisions of Republic Act No. 6656 (An Act to Protect the Security of [Tenure of] Civil Service Officers valid abolition of the ERB. Thus, there is no merit to KERB's allegation that there is an impairment of the
and Employees in the Implementation of Government Reorganization) will not directly apply to ERC's security of tenure of the ERB's employees.
current efforts to establish a new organization. Civil Service laws, rules and regulations, however, will have
suppletory application to the extent possible in regard to the selection and placement of employees in the
ERC. On 5 November 2005, KERB sent a letter to the Commissioners stating the KERB members' objection
to the Commissioners' stand that Civil Service laws, rules and regulations have suppletory application in
the selection and placement of the ERC employees. KERB asserted that RA 9136 did not abolish the ERB
or change the ERB's character as an economic regulator of the electric power industry. KERB insisted that
RA 9136 merely changed the ERB's name to the ERC and expanded the ERB's functions and objectives.
KERB sent the Commissioners yet another letter on 13 November 2001. KERB made a number of requests:
E n v i R e v i e w e r ( M I D T E R M S ) P a g e | 37

REPUBLIC OF THE PHILIPPINES, REPRESENTED BY ENERGY REGULATORY BOARD petitioner, additional burden to small consumers constituting around 52% of petitioner’s customers. On May 18, 1970,
vs. MANILA ELECTRIC COMPANY, respondent. the Republic and other oppositors argued that the increase in rate sought is excessive and unreasonable
and will bring about greater hardship to the people, as well as directly cause increase in the cost of
Facts: production which will have to be unduly borne by the consuming public. The respondent Commission,
through Commissioner Enrique Medina, issued an order on May 27, 1970 directing the Auditor General to
MERALCO filed with petitioner ERB an application for the revision of its rate schedules to reflect an average conduct an examination of MERALCO’s books of account and to submit a report thereof within 15 days.
increase in its distribution charge. ERB granted a provisional increase subject to the condition that should The Office of the Auditor General requested the respondent through a letter on June 9, 1970 to allow it
the COA thru its audit report find MERALCO is entitled to a lesser increase, all excess amounts collected sufficient time until June 30, 1970 to submit its report. However, in view of his impending retirement on
from the latter’s customers shall either be refunded to them or correspondingly credited in their favor. The July 2, 1970, the Commissioner replied on June 16, 1970 that the report be submitted on or before June
COA report found that MERALCO is entitled to a lesser increase, thus ERB ordered the refund or crediting 20, 1970. By return indorsement, the Office of the Auditor General informed the Commissioner on June
of the excess amounts. On appeal, the CA set aside the ERB decision. MRs were denied. 19, 1970 that although the examination could be finished that day, yet it would take about a week or so
to prepare and write the report. The Office of the Auditor General submitted its report on June 24, 1970
Issue:
to the respondent Commission, without the supporting documents and for lack of material time, without
Whether or not the regulation of ERB as to the adjustment of rates of MERALCO is valid. being able to delve “into as much detail as would ordinarily be done in a rate audit, considering the
magnitude of the utility’s operations, so that only the bit items were test-checked.” After hearing on the
Ruling: YES. merits of the petition, the respondent Commission promulgated a decision on June 30, 1970 finding that
the proposed rates reasonable and justified with minor adjustments. Respondent Commission issued an
The regulation of rates to be charged by public utilities is founded upon the police powers of the State and order on July 1, 1970 clarifying and supplementing its decision that in case the decision should be appealed
statutes prescribing rules for the control and regulation of public utilities are a valid exercise thereof. When the provisional rates should apply in the meantime. The Republic filed a motion for reconsideration on July
private property is used for a public purpose and is affected with public interest, it ceases to be juris privati 20, 1970 of the decision and order with the Commission en banc, requesting the Secretary to include in
only and becomes subject to regulation. The regulation is to promote the common good. Submission to the calendar of the Commission en banc on August 6, 1970. The oppositors filed with the respondent
regulation may be withdrawn by the owner by discontinuing use; but as long as use of the property is Commission on August 10, 1970 a notice of appeal from the decision dated June 30, 1970 and
continued, the same is subject to public regulation. supplementary order dated July 1, 1970. For lack of quorum to enable the respondent Commission to sit
In regulating rates charged by public utilities, the State protects the public against arbitrary and excessive en banc, owing to the retirement from service of Commissioner Medina on July 2, 1970, the existence of
a vacancy among the five Associate Commissioners, thereby leaving only four incumbent Associate
rates while maintaining the efficiency and quality of services rendered. However, the power to regulate
rates does not give the State the right to prescribe rates which are so low as to deprive the public utility Commissioners, and it being the provision in Section 3 of the Public Service Act, the petitioner’s motion for
reconsideration could not be heard and resolved by the respondent Commission. For this reason, the
of a reasonable return on investment. Thus, the rates prescribed by the State must be one that yields a
Republic filed a petition for review without awaiting the resolution of the respondent Commission on the
fair return on the public utility upon the value of the property performing the service and one that is
petitioner’s motion for reconsideration, the filing of which is not a condition precedent to enable the
reasonable to the public for the services rendered. The fixing of just and reasonable rates involves a
petitioner to appeal from said decision of the respondent Commission. MERALCO in turn filed a motion for
balancing of the investor and the consumer interests.
reconsideration, being unacted upon, for lack of quorum, resorted to this Court.
WHEREFORE, in view of the foregoing, the instant petitions are GRANTED and the decision of the Court
of Appeals in C.A. G.R. SP No. 46888 is REVERSED. Respondent MERALCO is authorized to adopt a rate Issue: Whether or not the order made by the Commission authorizing the provisional rates is valid.
adjustment in the amount of P0.017 per kilowatthour, effective with respect to MERALCO's billing cycles
beginning February 1994. Further, in accordance with the decision of the ERB dated February 16, 1998, Held: Yes. Considering that when the hearings were begun the notice had already been published six days
the excess average amount of P0.167 per kilwatthour starting with the applicant's billing cycles beginning in succession, and moreover, Section 16(c) of the Public Service Act expressly prescribes—that the
February 1998 is ordered to be refunded to MERALCO's customers or correspondingly credited in their Commission may, in its discretion, approve rates proposed by public services provisionally and without
favor for future consumption. necessity of any hearing; but it shall call a hearing thereon within 30 days thereafter, upon publication and
notice to the concerns operating in the territory affected. If the Commission is empowered to approve
provisional rates even without a hearing, a fortiori it may act on such rates upon a six-day notice to persons
REPUBLIC V. MEDINA concerned. In fact, when the provisional rates were approved on May 20, the full 10-day notice had been
published. The Public Service Act makes no distinction; it speaks of rates proposed by public services; and
Facts: Manila Electric Company (MERALCO) filed an application with the Public Service Commission seeking whether initial or revised, these rates are necessarily proposed merely, until the Commission approves
approval of revised rate schedules, with increased charges, claiming that the floating exchange rate and them. Moreover, the Public Service Commission practice is to hear and approve revised rates without
economic conditions resulting therefrom increased its operating and maintenance expenses by more than published notices of hearing. Thea reason is easily discerned: the provisional rates are by their nature
40%, and likewise increased the peso cost of servicing its foreign debts, causing it to incur an operational temporary and subject to adjustment in conformity with the definitive rates approved, and in the case at
deficit and net loss of over one million pesos a month. The proposed new rates would give it a reasonable bar, the Public Service Commission order of May 20, 1970 expressly so provided.
return of below 12% of the present value of its properties devoted to the public service, and implicated no
E n v i R e v i e w e r ( M I D T E R M S ) P a g e | 38

Thus, it is held that a regulatory commission’s field of inquiry, however, is not confined to the computation HELD: No. The State, in its exercise of police power, can regulate the rates imposed by a public utility
of the cost of service or capital nor to a mere prognostication of the future behavior of the money and such as SURNECO
capital markets. It must also balance investor and consumer expectations in such a way that broad
requirements of public interest may be meaningfully realized. It would hence appear in keeping with its The ERC was merely implementing the system loss caps in R.A. No. 7832 when it reviewed and confirmed
public duty if a regulatory body is allowed wide discretion in the choice of methods rationally related to the SURNECOS PPA charges, and ordered the refund of the amount collected in excess of the allowable system
achievement of this end. loss caps through its continued use of the multiplier scheme. The Commission deemed it appropriate to
clarify its PPA confirmation process particularly on the treatment of the Prompt Payment Discount (PPD)
granted to distribution utilities (DUs) by their power suppliers. The foregoing clarification was intended to
SURIGAO DEL NORTE ELECTRIC COOPERATIVE, INC. (SURNECO), PETITIONER,V. ENERGY ensure that only the actual costs of purchased power are recovered by the DUs.
REGULATORY COMMISSION, RESPONDENT.
In directing SURNECO to refund its over-recoveries based on PPA policies, which only ensured that the PPA
FACTS: mechanism remains a purely cost-recovery mechanism and not a revenue-generating scheme for the
electric cooperatives, the ERC merely exercised its authority to regulate and approve the rates imposed by
The Association of Mindanao Rural Electric Cooperatives, as representative of SURNECO and of the other the electric cooperatives on their consumers. The ERC simply performed its mandate to protect the public
33 rural electric cooperatives in Mindanao, filed a petition before the then Energy Regulatory Board (ERB) interest imbued in those rates.
for the approval of the formula for automatic cost adjustment and adoption of the National Power
Corporation (NPC) restructured rate adjustment to comply with Republic Act (R.A.) No. 7832. As held in the case of Republic v. Manila Electric Company, the regulation of rates to be charged by public
utilities is founded upon the police powers of the State and statutes prescribing rules for the control and
The ERB granted SURNECO and other rural electric cooperatives provisional authority to use and implement regulation of public utilities are a valid exercise thereof. When private property is used for a public purpose
the Purchased Power Adjustment (PPA). In the meantime, the passage of R.A. No. 9136 led to the creation and is affected with public interest, it ceases to be juris privati only and becomes subject to regulation.
of the Energy Regulatory Commission (ERC), replacing and succeeding the ERB. All pending cases before The regulation is to promote the common good. Submission to regulation may be withdrawn by the owner
the ERB were transferred to the ERC. Thereafter, the ERC continued its review, verification, and by discontinuing use; but as long as use of the property is continued, the same is subject to public
confirmation of the electric cooperatives implementation of the PPA formula based on the available data regulation. Likewise, SURNECO cannot validly assert that the caps set by R.A. No. 7832 are arbitrary, or
and information submitted by the latter. that they violate the non-impairment clause of the Constitution for allegedly traversing the loan agreement
between NEA and ADB. Striking down a legislative enactment, or any of its provisions, can be done only
The ERC issued its assailed Order, mandating that the discounts earned by SURNECO from its power by way of a direct action, not through a collateral attack, and more so, not for the first time on appeal in
supplier should be deducted from the computation of the power cost. SURNECO filed a motion for order to avoid compliance. The challenge to the laws constitutionality should also be raised at the earliest
reconsideration, but it was denied. Aggrieved, SURNECO filed a petition for review to the CA but the same opportunity. Even assuming, merely for arguments sake, that the ERC issuances violated the NEA and ADB
was denied. Upon denial of the motion for reconsideration, SURNECO files the instant petition. covenant, the contract had to yield to the greater authority of the States exercise of police power. It has
long been settled that police power legislation, adopted by the State to promote the health, morals, peace,
ISSUE: Whether or not the CA erred in affirming the ERC Decision education, good order, safety, and general welfare of the people prevail not only over future contracts but
even over those already in existence, for all private contracts must yield to the superior and legitimate
measures taken by the State to promote public welfare

Вам также может понравиться